Вы находитесь на странице: 1из 139

Answers Book / Contents:

A total of 17 chapters containing 713 Answers


distributed as;
1- Cardiology; 55 answers.
2- Pulmonary medicine; 51 answers.
3- Gastroenterology; 50 answers.
4- Hepato-biliary system; 30 answers.
5- Nephrology; 51answers.
6- Electrolytes and Acid-Base Disturbances; 20
answers.
7- Endocrinology; 50 answers.
8- Diabetes Mellitus; 20 answers.
9- Hematology; 40 answers.
10- Rheumatology; 50 answers.
11- Neurology; 176 answers.
12- Infectious diseases; 40 answers.
13- Immunology; 10 answers.
14- Psychiatry 10 answers.
15- Dermatology; 35 answers.
16- Genetics; 5 answers.
17- Toxicology; 20 answers.
NB: Questions regarding basic medical sciences are
distributed throughout chapters.
This is the Answers Book
Please see questions in the Question Book
Preface:
The art of medicine involves questions. It involves
questions asked when taking a medical history, when
forming a differential diagnosis, and when planning a
diagnostic or therapeutic plan. MRCP candidates,
regardless of their level of training, are constantly
confronted with questions posed from past papers, from
patients, from mentors, and from within themselves.
The time-honored, question-based, Socratic approach
of teaching is alive and well in academic and clinical
world of internal medicine. This book is intended to
provide the reader with many of the questions and
answers commonly encountered during their MRCP
study and period of preparation. This book is not meant
to replace textbooks. Rather it intended to focus on the
lead-in questions and topics commonly seen in the
MRCP examination. Please read textbooks to boost
your level of knowledge. Some of the 1
st
chapters were
launched in www.aippg.net website; but these are now
modified and updated.
I'm greatly thankful to my direct board supervisor
Professor Doctor Khalil Al-Shaikhly (MRCP UK,
FRCP Glasgow) for his continuous support, to our dear
patients, and to my dear friends and colleagues.
Dr. Osama Amin
All Rights Reserved. January 2006.
http://neurology4mrcp.orgfree.com/
http://neurology4mrcp.bravejournal.com/
mrcpfrcp@gmail.com
Chapter I / Cardiology Answers
Q1:
Answer: e
All other options are true, and also: hypertrophic obstructive cardiomyopathy, and
right ventricular hypertrophy. Note that the causes are not that many and they are easy
to be remembered.
Remember: TRUE and isolated posterior wall MI is very rare, usually associated with
inferior wall myocardial infarction, so look also at lead II, III, and aVF (i.e. right
coronary artery occlusion).
NB: Mirror image dextrocardia and wrong lead connection (so called limb lead
reversal) are usually forgotten as a cause of prominent R wave in lead V1.
Q2:
Answer: e
All other options are associated with ST segment depression.
Causes of ST elevation:
A- Full thickness myocardial infarction ( so called ST segment elevation myocardial
infarction or STEMI).
B- Early repolarization after an attack of angina.
C- Acute pericarditis.
D- Ventricular aneurysm.
E- Transiently during cardiac and coronary angiography.
F- Prinzmetal's angina.
Again, these are easy to be remembered as they are few in number.
Q3:
Answer: e
Remember, ventricular arrhythmias in WPW (apart from ventricular fibrillation
degenerated from atrial fibrillation) are highly atypical, and suggest either an
alternative diagnosis or co-existent pathology ( e.g. amiodaron given in this patient in
the long term to prevent arrhythmias, causing long QT interval and then torsades de
pointes ventricular tachycardia).
Q4:
Answer: e
Left sided cardiac lesions are not part of Noonan's syndrome and suggest an acquired
defect e.g. mitral stenosis following a rheumatic fever attack.
Remember: short stature may be seen ( the phenotype is usually mistaken for that of
Turner's syndrome), so be ware of a FEMALE Turner's with right sided cardiac
lesions, actually she may be Noonan's as it is an autosomal disease (male=female).
Q5:
Answer: d
Remember, long QT Syndrome is a risk for torsades de pointes ventricular
tachycardia. Long QT syndrome can be caused by:
1-Inherited syndromes (congenital long QT syndromes)
2-Electrolyte imbalance (see above)
3-Mitral valve prolapse.
4-Rheumatic carditis
5-Drugs (like Class Ic and III anti-arrhythmics).
6-Bardycardia associated: any cause of bradycardia, hence the use of isoprenalin
infusion in such cases (be ware, it is contraindicated in congenital cases).
Q6:
Answer: c
a- False, sarcomeric contractile proteins gene mutation. Many types of mutations had
been detected, and certain gene mutations per se predict a poor prognosis.
b- False, 50% of cases. 25% of Idiopathic dilated cardiomyopathy patients have a
positive family history.
c- True, but the presence of obstructive element per se does not predict a poor
prognosis.
d- False, many cases are totally asymptomatic and detected by doing
echocardiography for some reason or another.
e- False, the asymmetric septal type is the commonest one, but the apical variety is the
predominant one in the Far East. Note that there are certain variants which do not
have cardiac hypertophy at all.
Note that the commonest mutations are seen in:
1- Beta myosin heavy chain gene, associated with elaborate ventricular
hypertrophy.
2- Troponin gene, little or no ventricular hypertrophy, abnormal vascular
responses upon exercise, and there is a high risk of sudden death.
3- Myosin binding protein- C gene, usually manifested later in life, often
associated with prominent cardiac dysrrhythmias and systemic hypertension.
Q7:
Answer: b
a- True, the usually seen type with no cause that can be identified, although a viral
etiology is supposed to be the culprit..
b- False, rarely progresses, and most of cases follow a benign course.
c- True, with a risk a hemorrhagic component. The presence uremic pericarditis in
those on dialysis indicates an inadequate dialysis regime, or if the patient was not on
dialysis, then it is an indication to start dialysis.
d- Be ware of this, a disappearance of the rub indicates either:
1- A transient phenomenon, which is very common in clinical practice.
2- A resolution of the process.
3- A fluid collection in the pericardial sac.
e- True, usually presents either as a progressive collection of a large effusion or a
chronic constrictive picture.
Q8:
Answer: a
a- True, type A aortic dissection presenting as an inferior wall myocardial infarction
due to involvement of the ostium of the right coronary artery, and an acute aortic
regurgitation.
b- Pregnancy now is considered to be a relative contraindication.
c- Proliferative diabetic retinopathy is a relative contraindication.
d- Severe but easily controllable hypertension is a relative contraindication.
e- Actually, thrombolysis is STRONGLY indicated here.
Q9:
Answer: a
Hydralazine and minoxidil, both have a pure arteriolar dilating effect. Others are
having arterilolar and veno-dilating effects
Q10:
Answer: e
The followings indicate a high risk unstable angina:
1- Post-infarct angina.
2- Recurrent chest pain at rest.
3- Development of heart failure.
4- Cardiac dysrhythmias.
5- The presence of transient ST segment elevation.
6- The presence of ST segment depression
7- The persistence of deep T wave inversion.
8- Cardiac troponin level of more than 0.1 microgram / L.
Q11:
Answer: c
Option A and B are true and indicates the predominance of the right coronary arterial(
RCA) system in cases of RCA occlusion causing AV blocks in inferior wall
myocardial infarction.
Option c is false. Although used by many PTCA centers, such an approach should be
done only by those who are highly experienced, otherwise CABG should be used.
Q12:
Answer: d
The corneal deposits are usually reversible. Other side effects: pulmonary fibrosis,
slate grey skin pigmentation, hepatotoxicity. Always, be ware of drug interactions in
any combinations. 40% of the drug is iodine, hence the risk of either hypo or
hyperthyroidism.
Q13:
Answer: d
a- Superior vena cava obstruction causing FIXED J VP elevation.
b- Pericardical constriction.
c- Right ventricular infarction.
d- False. There is hypovolemia causing low J VP (unless there is a coexistent
pathology like heavy alcoholism causing cirrhosis and cardiomyopathy. Also the J VP
is useful in differentiating cirrhosis from pericardial constriction).
e-Ebstein anomaly and tricuspid regurgitation.
Q14:
Answer: c
a- True, the usual story (but presents as heart failure in infancy).
b- True, in certain series may reach 50%(remember, both the site of the coarctation
and the bicuspid aortic valve are high risk factors for infective endocarditis).
c-False, usually seen after the age of 6 years.
d- and e are true, hence in any young patient with SAH and previous "high" blood
pressure you must exclude coarctation of the aorta.
Q15:
Answer: e
Atenolol had not been studied in these trials.
Other drugs that improve the survival figure in chronic congestive heart failure are
ACE inhibitors.
Remember: Diuretics (apart from spironolactone) and digoxin do not improve the
survival figure in chronic congestive heart failure.
Q16:
Answer: d
Causes of culture negative endocarditis:
1-The commonest is prior antibiotic treatment.
2-Fastidious organism like HACEK group or a difficulty in culturing like brucella
species.
3-Fungal endocarditis and Q fever.
4-Marantic and Libman Sacks endocarditis.
5- Poor lab techniques and errors in collection of the blood samples (staph
epidermidis colonies FREQUENTLY reported by the lab as a NORMAL skin
commensal).
Q17:
Answer: e
Any cardiac lesion that is associated with a "J ET" lesion due to high flow indicates a
high risk, mitral stenosis is actually associated with a SLOW flow across the valve so
it has a low risk. Remember the highest risk is seen in: a-previous history of infective
endocarditis whatever the original cause was, and b-Prosthetic valves.
Q18:
Answer: c
Always GUESS the "best treatment" based on risk factors and history e.g. staph or
fungal endocarditis in IV drug addicts with a tricuspid valve endocarditis, and give the
treatment accordingly, pending the culture results.
Always remember that persistent FEVER may indicate failure of medical treatment
and persistent infection which is an indication for surgery. Other causes should
always be excluded:
1- Resistant organism .e.g. staph aureus to benzyl penicillin.
2- A non-bacterial cause like fungal organisms.
3- Superficial thrombophlebitis, as these drugs are given by an iv route.
d- "Drug fever ".
e- Coexistent pathology like wound infection or a visceral abscess that is difficult to
be treated which may be actually the cause of the endocarditis e.g. staph abscess
disseminating to produce metastatic lesions including involvement of the heart valves.
Q19:
Answer: d
In general, S3 indicates rapid early ventricular filling and /or high flow across the
mitral valve (or tricuspid valve in right sided S3) or a systolic dysfunction, so it is not
seen in PURE mitral stenosis as the flow is already impeded across the valve. If you
are sure that it is mitral stenosis and you heard a left ventricular S3 sound, this either
indicates MIXED mitral valve disease with the regurgitation is the predominant lesion
or an associated aortic regurgitation or simply a an LV dysfunction from other cause (
always remember that rheumatic heart disease is frequently associated with multiple
valvular lesions. Be sure it is not a right ventricular S3 as this may be seen in PURE
mitral stenosis due to right sided heart failure.
Q20:
Answer: c
It is supravalvular in William's (with mental retardation, hypercalcaemia, and elfin
faces).
A thrill is commonly felt at the neck (carotid shudder) and once symptoms appear you
have to intervene, usually ending with valve replacement.
Remember: always see the age of the patient as this may indicate the cause and
always see features of HOCM as the management is totally different. Aortic stenosis
per se is a risk factor for infective endocarditis before and after surgery.
Q21:
Answer: e
Always LEAVE some leg edema (i.e. one plus edema) as total DRYNESS means
profound hypovolemia and this would result in many complications e.g. prerenal
failure and electrolyte imbalance.
Q22
Answer: b
The objectives of treatment in this diastolic heart failure differ from that of congestive
heart failure states with systolic dysfunction. Targets of treatment in general are:
1- Control rate: to give time for the ventricle to fill properly, hence tachycardias have
a deleterious effect.
2- Control edema: with diuretics.
3- Control hypertension.
4- Control the original disease: e.g. aortic stenosis.
Till now and unfortunately there is no general consensus about the optimal medical
treatment. ACE inhibitors have NO effect on the overall mortality figure (cf systolic
congestive heart failure).
Aortic stenosis, not regurgitation, is a cause.
Q23:
Answer: e
Secondary hypertension may develop in the way of long standing essential
hypertension like the development of an atherosclerotic renal artery stenosis and
causing deterioration in the in the overall hypertension control.
Remember: renal artery stenosis is both a cause and an effect of hypertension.
Remember: renal failure is both a cause and an effect of hypertension.
Strong family history of hypertension usually goes with essential variety.
Q24:
Answer: b
Poorly controlled hypertension is a very powerful risk factor for intracerebral
bleeding, Diuretics (and non-dihydropyridin calcium channel blockers) are the first
line agents in old people with essential hypertension. In diabetic nephropathy with
persistent proteinuria the target should be below 120 / 75 mmHg.
Q25:
Answer: 5
Option e is a protective one.
Q26:
Answer: 5
Isolated systolic heart failure is highly unusual in long standing hypertension and
suggests a coexistent pathology like toxic causes.
Q28:
Answer: a
Metoprolol, acebutolol, and atenolol are cardio selective.
Q27:
Answer: c
High incidence of recurrence following surgery in seen in familial cases.
Remember: the commonest site is the fossa ovalis at the left side of the interatrial
septum (70%). Atypical sites usually indicate and occur in familial cases.
Q29:
Answer: e
The J VP is a good "window" to see what is happening in the heart.
In heart block the cannon "a" waves are irregular; if regular they may indicate a nodal
rhythm.
Q30:
Answer: d
Short PR interval occurs in:
1- Any tachycardia state (theophyllin may cause tachycardia).
2- Congenital short PR interval (WPW and LGL syndromes).
3- Ventricular ectopic occuring immediately after a sinus "p" wave.
4- Nodal rhythms.
Q31
Answer: b
WPW has many diverse and atypical ECG manifestations like psudo- MI pattern,
pseudo-LVH pattern. Posterior wall MI has a prominent R in lead V1. Other causes of
pathological Q wave: HOCM and errors in leading and calibration of the ECG
machine.
Q32:
Answer: a
Ebstien anomaly is not commonly associated with aortic coarctation. All other options
are true.
Q33:
Answer: e
Morphine should be given IV as there is a risk of hematoma formation in SC route
(remember the patient will be on a thrombolytic, aspirin and heparin)
Aspirin per se enhances the effect of the thrombolytic therapy and improves the
mortality figure. .
Q34:
Answer: e
No place for Digoxin in secondary prophylaxis of MI. Remember, the 4 drugs in this
topic:
Aspirin, ACE inhibitors, beta blockers, and statins.
Q35:
Answer: e
Option "e" may interfere with the performance and interpretation of the test. There are
many contraindications to exercise testing, and these are:
1-any significant left ventricular out flow obstruction eg severe aortic stenosis and
hypertrophic cardiomyopathy.
2-suspected or documented left main stem stenosis.
3-untreated congestive heart failure.
4-poorly controlled severe systemic hypertension.
5-suspected aortic dissection.
6-during the course of acute coronary syndromes eg unstable angina.
7-aquired complete heart block.
8-any febrile illness in general.
9-acute pericarditis or myocarditis.
Q36:
Answer: e
Such changes in middle aged woman with chest pain are seen commonly and
unfortunately affect the interpretation of the test as "false positive".
Options a, b, c and d and also development of such changes at an early stage of Bruce
protocol (ie low threshold for ischemia) indicate a STRONG positive test and a high
risk for subsequent coronary events.
Q37:
Answer d
Option "d" actually indicates a strong positive exercise ECG testing and should be
followed by angiography as this indicates a high risk patient for subsequent coronary
events.
Others are truly a cause of such a false positive results, and also electrolyte
disturbances like hypokalemia, anemia, beta blockers, left bundle branch block, and
hyperventilation.
Q38:
Answer: e
This is a pulseless electrical activity (PEA) previously called electromechanical
dissociation.
So dont jump to DC the MI patient in the CCU, "cardiac arrest" is not always due to
VF / VT causing death like appearance and no pulse. So always check the monitor
first and be calm , this is the objective of the CCU monitors!!! Causes of PEA:
Hypoxia, hypokalemia / hyperkalemia, hypovolemia, hypothermia, tension
pneumothorax, tamponade, toxic / therapeutic disturbances, thromboembolic /
mechanical obstruction.
Q39:
Answer: e
ACE inhibitors in option "e" had been shown to be greatly effective and of great
benefit. Dont be afraid of the "RENAL effect of ACE inhibitors" here, actually they
are the drug of choice in uremia with hypertension and post MI secondary prophylaxis
states.
Q40:
Answer: a
Thiazides are the first line agents in hypertension for many reasons. Loop diuretics are
the first line agents in heart failure. So loop diuretics are preferred in heart failure with
hypertension.
Be ware of long term complications of diuretic use, like acid base and electrolyte
disturbance, hyperlipdemia, hyperurecemia ...etc.
Q41:
Answer: d
1 out of 10 cases is familial and an autosomal inheritance has been suggested.
Remember the definitive treatment is heart lung transplantation and medical treatment
is only weakly effective and mainly used in the way of awaiting surgery.
Medications used:
Prostacyclin infusion, nitrous oxide inhalation (directly to the pulmonary vascular
bed), warfarine (as in situ thromosis has been seen in some cases), nifidipine.
Dont forget treatment of right sided heart failure!!!!
Medial hypertrophy and fibrinoid necrosis are seen in all branches of the pulmonary
arterial tree and result in pulmonary vascular obstruction.
Q42
Answer: e
Cor pulmonale is the dilatation and or hypertrophy of the right ventricular chamber in
response to an increase in the pulmonary vascular resistance, not necessarily
associated with heart failure (a common wrong belief, cor pulmonale=right sided
heart failure).
If you have a patient who is at risk of cor pumonale (eg bronchiectasis, the first thing
to see during follow ups is the J VP, it is the first sign, even before the leg edema.
Post polio syndrome is an MND like picture, so respiratory muscles may be involved
with chronic hypoxia and hypercapnia.
Q43:
Answer: b
The idea of this question is that this COPD patient is cyanosed with somnolence .so
this indicates CO2 narcosis, so option b seems reasonable trying to detect
papillodema. In COPD with cor pulmonale:
1- Do fundoscope : looking for papillodema (CO2 narcosis produces raised
intracranial pressure due to vasodilatation).
2- Examine his hands looking for:
1-flapping tremor.
2-cyanosis.
3-warmness.
4-bounding rapid pulse.
5- ? Clubbing has been suggested by some series.
Remember, cor pumonale in the context of COPD is under diagnosed in clinical
practice unfortunately.
Echo study will show greatly dilated and or hypertrophied right ventricle WITH small
sized compressed left ventricles (hence the presence of a low cardiac out put state and
deterioration of the chest problem).
Q44:
Answer: e
Up to 70-80 % of all lung emboli came from the venous system of the legs, yet only
10-15 % came from the pelvic veins.
Dont forget the rare causes of lung emboli : amniotic fluid, placenta, air , fat , tumor (
especially choriocarcinoms ) ,parasites ( especially schistosomes ) and septic emboli
from right sided endocarditis.
Q45:
Answer: c
a- True, the usual story especially in acute massive ones.
2- True, and they need a good film quality to be seen by an expert!!
3- False, although highly characteristic in the appropriate clinical settings,
unfortunately they are uncommon.
4- True, due to segmental collapse.
5- Secondary infection of an infarcted area.
Other signs: linear horizontal opacities (usually lower zones) and pleural effusion
(may be bloody).
Q46:
Answer: d
Although S1Q3T3 pattern is highly suggestive, in clinical practice it is unfortunately
uncommon. T wave inversion in lead V1 and V2 can be very useful instead.
Remember, the ECG might be totally normal.
Q47:
Answer: d
Heparin therapy per se substantially REDUCES the mortality by preventing further
embolic events and also reduces mediator induced pulmonary vasoconstriction and
bronchospasm from thrombin activation and platelet aggregation.
Remember, oral anticoagulants do not act immediately.
Q48:
Answer: b
The use of inotropic agents is of LITTLE value as the hypoxic dilated right ventricle
is already MAXIMALLY stimulated by endogenous catecholamines.
Transcatheter suction embolectomy has been used with good results when compared
with option "e" above.
Q49:
Answer: d
Difficult and tricky one, but you have to know the incidence of the common
anomalies.
Tetralogy of Fallot -6%.
Others:
Pulmonic stenosis 7%.
Aortic stenosis 6%.
Complete transposition of great arteries 4%.
Q50:
Answer: c
a- True, due to loss of support of the right coronary cusp.
b- True, there is already an oligemic lung.
c-False, ASD secondum usually presents in middle age with cardiac arrhythmias
,usually atrial fibrillation. ASD primum, because of the associated mitral and or
tricuspid regurgitations, tends to present earlier.

Q51:
Answer: e
a- Lutembacher syndrome.
b- Lithium usage (3% incidence of Epstein anomaly).
c- True, also ASD.
d- True, Valproate sodium can cause this.
e- False, there is right sided cardiac lesions like pulmonic stenosis.
Q52:
Answer: e
a- ?Neurosyphilis- Aortitis.
b- ?Ankylosing spondylitis.
c- ?Pseudoxanthoma elasticum-plucked chicken skin.
d- False, may reveal rheumatoid arthritis.
e- ?mucopolysacharidosis, cloudy cornea.
Q53
Answer: c
a- True, due to pulmonary hypertension and secondary tricuspid regurgitation.
b- True, also deterioration due to atrial fibrillation which does not respond to medical
treatment.
c- Such a degree of a gradient indicates a severe and a significant stenosis.
d- True. Also, limitation of physical activity despite optimal medical treatment.
e- True.
Remember: the commonest cause of Mitral stenosis is rheumatic heart disease and
usually there is an associated other valvular abnormalities (usually aortic) so before
assuming that it a mitral stenosis which DOES NOT need antibiotic prophylaxis see
other valves like aortic regurgitation which will CHANGE the picture and definitely
the patient will need such a prophylaxis (for the aortic lesion not the mitral stenosis)
Remember: antibiotic prophylaxis when given against infective endocarditis, will
NOT be against rheumatic fever (and vise versa) .i.e. 17 year old male with
significant mixed aortic valve disease and mitral stenosis due to rheumatic fever
needing a dental help for tooth extraction, he should be on antibiotic prophylaxis for
the dental work +usual daily prophylaxis for the rheumatic fever (remember he
may be also on warfarin ..So anticipate bleeding upon tooth extraction!! ).
Q54:
Answer: d
a- True, with right axis deviation.
b- True, with right bundle branch block and prolonged PR interval.
c- True, due to volume overload.
d- False, in Ebstien anaomaly there is: 1-TALL peaked P waves in standard lead II
2-RBBB with small amplitude QRS complexes. 3-WPW syndrome type B, i.e. the
QRS complex is negative in the right precordial leads.
4-paroxysmal supra-ventricular tachycardia
e- True.
Q55:
Answer: d
a- False, many changes will then be seen by this time, like cardiomegally, large
pulmonary arteryetc.
b- False, seen in Fallot's tetralogy, with oligemic lung.
c- False the REVERSED sign of 3. Notice that the rib nothing is seen from the 3
rd
to
the 9
th
rib by the age of 6 years. We may see also a left ventricular hypertrophy, and
cardiomegally.
d- True, with peripheral PRUNNING of the pulmonary vasculature, indicating severe
pulmonary hypertension.
e- False, can be seen on angiography.
Chapter II / Pulmonary Medicine Answers
Q1:
Answer: c
The visceral pleura is insensitive to pain and the pleural pain is due to involvement of
the parietal pleura.
Q2:
Answer: e
Option "e" causes tachypnia, and hence Co2 wash out and hypocapnia. All others are
the cause of hypoventilation and Co2 retention and cyanosis that can be corrected by
giving oxygen.
Q3:
Answer: a
Other causes of this type of hypoxemia that is not corrected by O2 therapy
1-Any right to left shunt (at cardiac or lung level).
2-Decreased O2 carrying capacity of blood: anemia and inactivated hemoglobins
Q4:
Answer: c
V/Q scan is contraindicated in ASD because the macro aggregates of albumin loge in
the cerebral and renal circulations with devastating effects.
MRI is usually used in mediastinal lesions and lung apical lesions and vertebral
lesions, CT scan is mainly used for parenchyma lung lesions.
Bronchoscope is useful in central tumors.
Ultrasound is very sensitive of the detection of small pleural effusions.

Q5:
Answer: e
Fibrosing alveolitis causes reduction in both TLCO and KCO. Isolated reduction in
the KCO with normal pulmonary function tests usually indicates severe anemia. All
other options cause a low TLCO with a normal or high KCO.
Q5:
Answer: c
Wegner's, as well as bronchogenic cysts, pulmonary sequestration, lymphoma, benigh
tumors, apspergilloma and rheumatoid nodule all are uncommon causes of a solitary
pulmonary nodule or the so-called coin lesion.
Q6:
Answer: e
The indications are:
1- PaO2 of less than 7.3 kPa with any level of PaCO2, and an FEV1 less than 1.5
liters.
2- PaO2 between 7.3 to 8 kPa PLUS either pulmonary hypertension, peripheral
edema, or nocturnal hypoxemia.
Q7:
Answer: e
Fever may be due to many things, so dont assume that the patient is having an
exacerbation simply because of fever.
Q8:
Answer: c
Steroids have no effect on the progression of the disease. Other indications:
a- if the exacerbation was the presenting feature.
b- if there is a poor response to bronchodilator therapy.
Remember: inhaled low dose steroids MAY be used in severe COPD with history of
severe recurrent exacerbations requiring hospitalization, but they dont affect the rate
of decline of FEV1 over time.
Q9:
Answer: d
Surprisingly, atopy patients have a better survival, and to date, no drug treatment
(aside from LTOT) has been shown to affect the disease outcome.
Q10:
Answer: d
Diuretics should be strongly considered if there edema or raised J VP. Other options
are applicable and true.
Q11:
Answer: e
Hypercapnia is a "relative" contraindication to air plane travel, as well as a PaO2 of
less than 6.7 kPa on air .Option d is true as ALL patients would develop profound
reduction in their PaO2.
Q12:
Answer: c
Weight loss is common and is due to inflammatory mediators from the bronchial
epithelium like TNF alpha, this usually stimulates unnecessary investigations for
weight loss for a fear of malignancy.
Q14:
Answer: c
Thickening of the basement membrane contributes to chronic disability and poor
response in chronic asthma cases.
All of the changes gradually in the long term become irreversible.
Q15:
Answer: c
IV steroids have no role in such a prophylaxis. All others can be used effectively.
Q16:
Answer: d
Hyperlucency is very common during an acute attack and acutally may be the only
finding. In chronic cases, the chest X ray may be like that of an emphysema.
Q17:
Answer: e
Options a, b, c and d are HIGHLY efficacious measures to prevent asthma attacks.
Option "e" has a low efficacy and preventive measures "against mites and pollens"
have a LOW to uncertain efficacy, unlike the popular belief. Option "e" is a powerful
measure in eczema, not asthma.
Q18:
Answer: d
a-this should be maintained in all cases, with O2 saturation above 93%.
2-you are not dealing with COPD with chronic hypercapnia!!! So use it.
3-oral steroids are given, but if the patient has vomiting or unable to swallow, then
give it IV.
4-should be done in all cases in the A&E department to assess the severity and
subsequent and follow up management plan.
5-true.
Q19:
Answer: c
Aminophyllin should be avoided in those who are already on oral theophyllin for a
high risk of toxicity is likelihood, besides; many randomized controlled trials showed
an increased mortality with its use.
Q20:
Answer: d
Such a PEFR indicates a severe attack, but not necessarily an indication for
mechanical ventilation. Other indications: PaO2 less than 8 kPa and falling, blood PH
is low and falling. i.e. deteriorating blood gases despite optimal therapy .
Q21:
Answer: d
Clubbing and hemoptysis are both common, and absence of large amount of sputum is
not against the diagnosis (bronchiectasis sicca).
Q22:
Answer: a
It is on chromosome 7. Many mutations had been detected till now.
Q23:
Answer: d
Actually treating other causes of bronchiectasis with recombinant human DNAase has
been shown to produce deleterious effects, so it is only used in bronchiectasis of
cystic fibrosis .Other mode of treatment in cystic fibrosis: continuous oral high dose
ibuprifin, and gene therapy.
Q24:
Answer: d
Actually, many are having overlapping picture and it is really difficult to differentiate
between typical and atypical pneumonias in clinical practice.
Q25:
Answer: e
The development of complications may and usually adversely affect the outcome and
should be looked for in all cases. Other options definitely applicable.
Q26:
Answer: c
Pleural fluid should be aspirated ONLY when present for more than a trivial amount
(more than 10 mm thickness on lateral decubitus films) and preferably under
ultrasound guide. Other lab tests should be ordered to diagnose the causative agent.
Unfortunately, in at least 20% of cases, no infectious agent can be isolated.
Q27:
Answer: C
Blood urea more than 7 mmol/L.
Others: WBC below 4000 or above 20000, hypoxemia with PaO2 below 8 kPa, age
above 60 years, multiple lobe involvement and the presence of underlying co-morbid
disease.
Q28:
Answer: e
Remember: atypical pulmonary edema may be unilateral and / or localized and may
be extremely difficult to differentiate it from pneumonias, however absence of fever
and presence of underlying heart disease are in favor of pulmonary edema.
Q29:
Answer: c
It occurs in up to 2-5 % of hospital admissions and predominantly of gram negative
type. Aggressive treatment is indicated as the majority of patients are already
debilitated by another illness, eg immune suppression, malignancy...etc.
Q30:
Answer: e
Chest x ray is usually normal in such cases and usually presents as pyrexia of
unknown origin, hepatosplenomegally weight loss and malaise.
Q31:
Answer: c
Usually it becomes falsely negative LATELY in the course of TB meningitis in 25-50
% of cases only. Other causes of false negative testing: sarcoidosis, childhood
exanthems, uremia, and systemic malignancy.
Q32:
Answer: a
a- True, it is static and hence contraindicated.
b- False, usually caused by ethambutol, and rarely by isoniazide.
c- False, a rare complication, be careful as it might cause jaundice and hence easily
confused as hepatitis.
d- False, the reverse is true.
e- False, only to high risk groups, e.g. HIV patients, malnutrition, chronic diarrhea,
alcoholics. It should not be given routinely.
Q33:
Answer: a
a- False, it is weakly positive, and the total IgE is greatly raised.
b- True, a hypersensitivity reaction.
c- True and may be very high.
d- True, aspirgillus clavatus is a cause of malt worker's lung and cheese worker's lung.
e- True, like asthma and cystic fibrosis.
Q34:
Answer: d
90% of lung cancers are associated with cigarette smoking. So PREVENTION is
easier than treatment. The reverse is true in option"d'.
Q35:
Answer: b
4% of cancer deaths in women and 8% cancer deaths in men. Difficult one I agree,
but you have to know these facts as this is an aggressive and a very common cancer.
Q36:
Answer: a
Some tumors are truly endobronchial and may not produce any changes in the plain x
ray films, especially early in the course, hence bronchoscope is important in high risk
clinical settings.
Q37:
Answer: b
In general, the contraindications are:
Any T4 (options a, c).
Any N3 (option e).
FEV1 less than 0.8 L.
Any severe or unstable cardiac or other medical conditions.
Q39:
Answer d
Option "d" is an anterior mediastinal mass.
Q40:
Answer: 4
This is a tricky question, although hypercalcemia is usually seen around 20-30 % of
cases but as a presenting complaint it is 1 % or less.
Q41:
Answer d
Other indications:
1- Rapidly worsening stage II/III with deteriorating lung function test.
2- Hypercalciuria.
3- Lupus perinio per se.
4- Any major organ involvement.
Q42:
Answer: 5
Surprisingly, LDH is elevated in the MAJ ORITY of patients. There is a query
association with antidepressants and EBV infection. Excess neutrophils in
bronchoalveolar lavage (BAL) portend a bad prognosis (neutrophilic BAL is also seen
in pneumoconiosis, in sarcoidosis and extrinsic allergic alveolitis it is lymphocytic).
ANA and RF are seen up to 50% of cases, and not necessarily indicate an association
with an underlying collagen vascular disease.
Q43:
Answer: c
Byssinosis-textile industries with cotton, flax and hemp dust.
Malt worker's lung-aspirgillus clavatus.
Cheese worker's lung- aspirgillus clavatus and penicillium cassie.
Bird's lung-avian serum proteins.
Q44:
Answer: d
a- True, up to 90%.
b- True, and demolition, ship breaking, break-pads, pipe and boiler lagging.
c- True, a strong association.
d- False, smoking has no effect in asbestos-related mesotheliomas.
e- Multiplicative rather than additive (may reach up to 50 folds).
Q45:
Answer: e
Church-Strauss vasculities may be the cause, but not Wegner's granulomatosis.
Also seen in :
1- acute and chronic eosinophilic pneumonia.
2- hypereosinophilic syndrome.
3- fungal and parasitic infections.
4- other drugs like PAS and sulphasalazine.
Q46:
Answer: c
Up to 50% remission occurs after whole lung lavage. Cough and dyspnea are
common.
Q47:
Answer: d
Hormonal ablation therapy and progestins are of doubtful value. The only treatment is
lung transplantation.
Q48:
Answer d
Subphrenic abscess is considered to be a common cause .Other uncommon causes:
Uremia, asbestos related effusion, and Dressler's syndrome.
Q49:
Answer: e
Chest tightness is very common, yet severe dyspnea is an indication. Besides
excessive coughing during simple aspiration, all other options are true.
Q50:
Answer: e
Phrenic nerve palsy is a cause. Other causes:
1- Excess gas in the stomach or colon.
2- Large tumors or cysts in the liver.
3- Any cause of reduction in the volume of the lung e.g. lobectomy or lung fibrosis.
Chapter III / Gastroenterology Answers
Q1:
Answer: c
Somatostatin decreases gastrin secretion, inhibits gastric acid secretion, inhibits
insulin secretion and inhibits gall bladder contraction. It is secreted from D cells
(present throughout GIT tract) in response to fat ingestion.
Q2:
Answer: e
Barium enemas usually miss polyps less than 1 cm in size. Remember: GIT contrast
studies are very useful in many GIT diseases including functional studies like gastric
emptying studies.
Q3:
Answer: e
Remember: the contraindications are;
1-severe shock. 2-recent MI, unstable angina and cardiac arrhythmias. 3-severe
respiratory disease. 4-atlanto-axial subluxation. 5-possible visceral perforation.
Items 2, 3 and 4 are relative contraindications and can be safely performed by an
expert. Careful patient selection is very important.
Q4:
Answer: e
Certain high risk groups (like cardiac prosthetic valves) should be given antibiotic
prophylaxis when undergoing upper GIT endoscope. All other options are true but
rare if the patients were carefully selected, and the operator was an efficient one.
Q5:
Answer: c
75SeHCAT test is used in bile salt malabsorption and diarrhea. Notice that this
question has long options, and was made only for teaching purposes, for such
investigations to be kept in mind. It is a difficult one, I agree,
Q6:
Answer: e
a- True, in peptic ulcer disease, but both tests differs at certain aspects (13C uses
radioactivity, and 14C requires an expensive mass spectrometer).
b- True, in obscure lower GIT bleeding in children.
c- True, and in inflammatory bowel disease.
d- True, in protein losing enteropathy.
e- 99Tc-sulphur is used as a test for gastric emptying studies e.g. in gastro paresis.
Notice that the majority of these tests are used at certain centers only and not that
widely available, ?teaching purposes.
Q7:
Answer: b
It is asymptomatic and discovered during endoscope done for certain reason or
another. Note that proton pump inhibitors are not effective at reversing the
histological abnormalities. Till now, there is no consensus about the optimal
management of it, because we may have mild, moderate, or a severe degree of
dysplasia. There many options, like LASER therapy, total esophagectomy, repeated
endoscope and biopsy samplingetc. Notice the risk of adenocarcinoma is very hight
compared to the general population. The incidence of esophageal adenocarcinoma is
rising at a global level.
Q8:
Answer: e
Esophageal adenocarcinoma is the result of barrette's esophagus. Other options are
true, and the patient may present with one of these complications rather than simply
with heart burn and regurgitation.
Q10:
Answer: c
The tight lower esophageal sphincter PROTECTS against acid reflux and hence NO
heart burn occurs. It is a pre-malignant condition, and the risk persists even in those
who had received successful treatment.
Q11:
Answer: e
Other etiologies: Barrette's esophagus, post-cricoid web, Post caustic stricture,
achalasia, chewing betel nuts.
Q12:
Answer: e
Pernicious anemia causes chronic atrophic gastritis (remember, no acid, no ulcer).
Other causes of acute gastritis: NSAIDS and aspirin, herpes simplex (with CMV are
seen in HIV), alcohol, severe stress like after burns or CNS trauma.
Q13:
Answer: e
Also no (or very weak) association with Non-ulcer dyspepsia and hence in both
(gastro-esophageal reflux disease and non-ulcer dyspepsia) there is NO need for H
pylori eradication therapy.
Q14:
Answer: c
What are these!? Long options, yes these were made only for teaching purposes.
Rapid urease test on an antral biopsy specimen has a LOW sensitivity but a HIGH
specificity. You should know these investigations and their limitations and benefits
very WELL.
Q15:
Answer: d
Age above 60 years is a risk factor. Remember; the most ulcerogenic NSAID is
azapropazone, and the least is low dose ibuprofen. For treatment or prevention of
NSAID-induced peptic ulceration :you can use either prostaglandin analogues like
misopristol OR proton pumps inhibitors , both have the same efficacy BUT the side
effects are more prominent with misopristol (like diarrhea and abdominal pain ), so
clinically proton pump inhibitors are preferred.
Q16:
Answer: a
Diarrhea is very common and is seen up to 30-50% of cases. Other options are true
and common. You have to know these well.
Q17:
Answer: c
You should be familiar with anti H pylori side effects ,as failure to notice them in a
truly peptic ulcer patient may make you feel that your diagnosis is wrong and you
may stop the treatment and subject the patient to unnecessary investigations.
a- True, also diarrhea, and potentiates warfarin.
b- True, and diarrhea.
c- False, binds these drugs and lessens their effect.
d- Unfortunately the main limitation. Also it is abortifacient and contraindicated in
woman of child bearing age.
e- Also prolonged treatment may produce bismuth toxicity.
Q18:
Answer: e
20-60 % of cases are part of MEN type I. Difficult options I guess, but it is one of the
commonest topics seen in the MRCP examination.
Q19:
Answer: d
Barium meal may reveal thick mucosal folds, due to the hypertrophic effect of gastrin
on the gastric mucosa. The presence of ulcers at atypical sites should always prompt a
search for ZE syndrome.
Q20:
Answer: d
Luckily, the overall 5 year survival is 60-75% which is a good figure. Remember:
high dose PPIs will produce profound symptomatic relief, heal ulcers and alleviate
diarrhea.
Q21:
Answer: e
Tylosis (palmoplanter thickening) predisposes to esophageal squamous carcinomas.
Other risk factors: smoking, alcohol, H pylori, dietary association with smoked or
pickled food, Menetrier's disease.
Q22:
Answer: a
Its incidence is falling in UK; the disease is extremely common in China, J apan and
south East Asia.
Q23:
Answer: e
The disease is much more common in northern Europe with a prevalence of 1/300.
The disease may be seen in up to 10% of first degree relatives.
Q24:
Answer: c
Rice, maize and potatoes are given and are satisfactory source of complex
carbohydrates. Remember: the histological changes in the small bowel are
REVERSIBLE, but the sensitivity is LIFELONG.
Q25:
Answer: b
Esophageal squamous cell carcinoma and intestinal lymphomas are seen. Item d is
dermatitis herpetiformis.
Q26:
Answer: e
In option "e", IgA anti-endomysium will be falsely negative, but IgG anti-
endomysium will be positive. Other options are true.
Q27:
Answer: a
Type I diabetes is an association (2-8%), not type II. Remember: although these
associations are relatively uncommon, but when present will confuse the picture and
will complicate the diagnostic approach or may make you feel that the patient is not
adherent on a gluten free diet or the diagnosis is wrong from the start. ALWAYS
think of a coexistent pathology in such diseases. Other association: Down's syndrome,
sarcoidosis, selective IgA deficiency (2%), autoimmune hepatitis, microscopic colitis,
and thyroid disease (5%).
Q28:
Answer: e
It is commonly seen there as well as southern India. It may be seen in a western
citizen (e.g. UK) returning from these areas. Other options are true.
Q29:
Answer: c
a- And long term use of PPIs, partial gastrectomy, hypogammaglobulinema
scleroderma, diabetic autonomic neuropathy, intestinal strictures (eg in Crohn's
disease), gastric surgery (blind loop after Billroth II operation, jejunal diverticulosis,
and enterocolic fistulas eg in Crohn;s disease.
b- True, duodenal and jejunal aspirate and cultures actually are ACADEMIC
methods, and rarely done in clinical practice.
c- Serum folate is normal or high and is a good clue. Serum B12 is low.
d- Although many patient may require extension to 28 days or a continuous rotational
courses of antibiotics.
e- True, with features of B12 deficiency.
Q30:
Answer: a
a- False, gram positive T whipelii.
b- True, and the presentation depends on the organ involved.
c- True, like CNS, lung, musculoskeletal.
d- True, following treatment, profound symptomatic improvement is seen in a week,
but the histological changes may take few weeks to revert to normal.
e- True, usually the relapsed site is the CNS.
Q31
Answer: d
Anti-diarrheal agents are useful. The commonest causes in adults are: Crohn's disease,
mesenteric infarction, radiation enteritis and volvulous.
Q32:
Answer: e
Cholestyramin is used when bile salt diarrhea due to terminal ileal disease is present.
Q33:
Answer: e
Many GIT diseases can cause protein losing enteropathy, like celiac, Crohn's,
lymphoma, Whipple's, tropical sprue, eosinophilic gastroenteritis, radiation enteritis.
Q34:
Answer: b
Lymphocytopenia is characteristic, and although hypogammaglobulinema is seen, but
the susceptibility to infections is that increased.
Q35:
Answer: e
The prognosis is good and almost all will respond to corticosteroids and or disodium
cromoglycate. Dietary manipulations are rarely effective. The disease is characterized
by inflammatory eosinophilc infiltration of the gut wall in the absence of parasitic
infestation or eosinophilia of other tissues. Exclusion of other causes of eosinophilia
must be part of the management. Eosinophilic ascites may occur when there is serosal
involvement.
Q36:
Answer: e
Crohn's disease is usually seen in active smokers and ulcerative colitis in non smokers
and ex-smokers. The above facts should be kept in mind.
Q37:
Answer: c
Daily bowel motions more than 6. Others features indicating a severe attack: the
presence of frank blood in stool (+++), pulse rate>90 beats/minutes, temperature >
37.8 C 2 days out of 4, frank blood seen in the lumen by sigmoidoscopy, and dilated
bowel loops and or mucosal islands on plain X ray films.
Q39:
Answer: e
Sacroiliitis, ankylosing spondylitis like picture, and primary sclerorsing cholangitis
are independent of the disease activity. These should kept in mind.
Q40:
Answer: e
Remember: Ulcerative colitis can be CURED by surgery (eg. panproctocolectomy
with permanent iliostomy), unlike Crohn's disease in which surgery should
conservative as much as possible.
Q41:
Answer: c
Up to 50 % of cases will meet criteria of a psychiatric disease ranging from anxiety
and depression to frank panic attacks.
Q42:
Answer: e
Metaplastic (or hyperplasic), hamartomatous and inflammatory polyps have no
malignant potential. Remember: up to 50 % of western population above the age of 60
years has been shown to have polyps, of those, 50% are multiple, and they are more
common in the distal colon.
Q43:
Answer: e
a- True, an uncommon Autosomal Dominant disease, responsible for about 1% of all
colorectal carcinomas.
b- True, and may undergo malignant changes which are usually the cause of death in
those who underwent total colectomy.
c- True, a good screening target, seen as dark round pigmented retinal lesions.
d- True, Gardener Syndrome is characterized by predominance of the extraintestinal
manifestations especially osteomas. Turcot's main clinical picture is malignant brain
tumors.
e- Desmoid tumors are BENIGN tumors, usually found in the mesentery or abdominal
wall.
Q44:
Answer: b
J uvenile polyposis is an autosomal dominant in 20% of cases and carries a risk of
colorectal carcinoma which usually develops before the age of 40 years.
Q45:
Answer: e
Other "Protective" factors:Diet rich in vegetables, folic acid and calcium. Other risk
factors: Obesity and sedentary life style, colorectal adenomas, and long standing
extensive ulcerative colitis.
Q46:
Answer: 5
Viral infections like mumps and coxachie are considered to be rare causes. Other rare
causes: abdominal trauma, surgery, cardiopulmonary bypass, prolonged hypothermia,
and drugs like azathioprine and valproate.
Q47:
Answer: d
Although serum amylase is used in the diagnosis when it is very high, it is level has
no prognostic value at all. Other factors:
WBC more than 15000/mm3.
Blood urea more than 16 mmol/L (after re-hydration).
ALT more than 200 u/ L.
LDH more than 600 u/L.
Q48:
Answer: e
a- True, because the kidneys very efficiently excrete amylase.
b- True, seen as failure of the inflamed pancreas to enhance after IV contrast CT
scanning.
c- True, and surgical drainage.
d- True, due to disruption of the pancreatic ducts.
e- False, serial measurements of the CRP is very important indicator of the over all
progression of the disease.
Q49:
Answer: e
Unfortunately, despite counseling and education, almost all patients continue to drink
alcohol (which actually produces pain relief) and up to 20% are opiates dependent.
Wight loss is common and is due to anorexia, avoidance of food because of post
prandial pain, diabetes and malabsorption.
Q50:
Answer: e
Pancreatic enzymes supplements are also useful as an analgesic by RESTING the
pancreas.
Chapter IV / Hepatobiliary Diseases Answers
Q1
Answer: 4
1- True, like Kupffer cells, Ito cells.
2- True, also removal of IgG complexes and cytokine production.
3- True, and also synthesis of extra-cellular matrix and synthesis and release of
collagenases and metalloprotease inhibitors.
4- False, vitamin K and folic acids are stored in small amounts and hence deficiency
occurs relatively rapidly when there is NO supply of them.
5- True, and during fasting also, the liver releases glucose.
Q2:
Answer: 5
Item 5 is in favor hepato-cellular damage. The reverse is true in biliary obstruction.
Q3:
Answer: 5
Cimetidin and ACUTE alcohol binge both are enzyme blockers or inhibitors.
Other inducing enzymes: meprobamate, INH, phenytoin, primidon and
phenobarbitone.
Q4:
Answer: 5
1- True, and also less useful in diffuse parenchymal diseases.
2- True, and also CT scan can be combined with splenoportography.
3- True, the MRCP (magnetic resonance cholangiopancreaticography ).
4- True, PTC or ERCP.
5- False, with radioisotope, both are rarely used now days.
Q5:
Answer: 4
1- True, can you do in an agitated patient?!
2- True, and it is contraindicated if the platelets less than 100 000/ mm3.
3- True, as well as severe anemia, hepatic hydatid cyst, hepatic hemangioma, and bile
duct obstruction.
4- False, is a contraindication.
5- True, as in lumbar puncture.
Q6:
Answer: 4
Up to 4 mg of urobilinogen is passed outside in urine every day. Other options are
true.
Q7:
Answer: 4
1- True, due to ineffective erythropoiesis.
2- True, may be hemolytic (or hepatocellular).
3- True, and Crigler Najjar syndrome type I and II.
4- False, it is a DIRECT hyperbilirubinemia, also in Dubin J ohnson syndrome.
5- True, and other causes of intravascular hemolysis.
Q8:
Answer: 5
So we are dealing with a direct hyperbilirubinemia:
1- ? pancreatic carcinoma
2- ? Bile duct stricture from a previous biliary surgery.
3- ? metastasis
4- ? any tumor including lymphoma and choledochal cyst
5- False, this and together with polished nails, just indicate itching from
cholestasis BUT with out any additional information for the cause.
Q9:
Answer: 5
A tricky question, all are used to stop the bleeding, but terlipressin is given
intravenously (ie systemic measure not a local measure). Other drugs used in the acute
setting: vasopressin, glypressin and octreotide. Remember: propranolol is used in the
primary and secondary prophylaxis against bleeding, not in the acute setting.
Q10:
Answer: 4
Esophageal transection is used to arrest acute bleeding episodes in certain situations.
It has no place in secondary prophylaxis.
Q11:
Answer: 5
Esophageal transaction although has a good result in experienced hands, it should be
done only when there is failure of endoscopic hemostasis and TIPSS is not available
or cannot be carried out. It carries a risk of future esophageal stricture. It is done by a
stapling gun.
Q12:
Answer: 5
1- True, should be done by an expert.
2- True, he results are excellent.
3- True, and all coagulation defects should be corrected also beforehand.
4- True, unfortunately yes, and necessitates iatrogenic shunt narrowing to lessen the
shunt.
5-False, this may indicate IN-STENT narrowing and hence venography and
angioplasty should be carried out.
Q13:
Answer: 3
1- True, and hence may present with fever only or as an acute sudden hepatic
encephalopathy in a previously well managed and compensated patient.
2- True, and hence polymicrobial infection should cast a doubt on the diagnosis and
regarded as visceral perforation until proved otherwise.
3- False, can be prevented by giving daily norfloxacin 400 mg / day.
4- True, and hence the name "SPONTANEOUS".
5- True, culturing the ascetic fluid in a blood culture bottle gives a high diagnostic
yield.
Remember: It carries a high mortality and hence should be treated aggressively.
Q14:
Answer: 4
Oral neomycin is used as a bowel decontaminant in these cases, and is not a
precipitating factor. Other precipitating factors:
a- Uremia: which may be spontaneous or diuretic induced.
b- GIT bleeding (upper or lower).
c- Surgery and trauma.
d- Surgical or spontaneous porto-systemic shunts.
e- Large volume abdominal paracentesis.
f- Hypokalemia.
g- Sedatives and hypnotics.
Q15:
Answer: 5
Remember: a patient with liver cirrhosis who presents with vague symptoms of
drowsiness, poor concentration...etc, dont assume that he is decompensating , but
first you should exclude many diseases which may resemble hepatic encephalopathy,
and then look for any precipitating cause for the encephalopathy, so you should be
patient and think well. Don't forget DOUBLE PATHOLOGIES .eg Cirrhosis patient
,who is alcoholic, and intoxicated after an alcohol binge, sustained a head injury with
subdural hematoma formation, and with alcoholic gastritis causing hematamesis, and
hypoglycemia may be present as well!
Q16:
Answer: 3
1- True, like hepatitis E infection in pregnancy.
2- True, hence you may FORGET the liver problem!
3- FALSE, it is absent in Rye's syndrome, but remember, the patient may DIE before
the development of jaundice!
4- True, hence you should be cleaver and anticipate them and correct rapidly them if
possible.
5- And early contact the hepatic transplant unit.
Q17:
Answer: 2
Although it is an intrinsic renal disease, the fractional Na excretion is less than 1 and
the Na concentration is less than 10 mmol / L in urine. i.e. pre-renal failure like
picture! Renal dose dopamine has a role in the management, yet the improvement in
renal function depends entirely on improvement of the liver function.
Q18:
Answer 5
Remember, MACRO-vesicular steataosis is much more common in is a benign
condition, yet the MICRO-vesicular type usually occurs in grave diseases like fatty
liver of pregnancy.
Q19:
Answer: 5
Hemochromatosis causes chronic liver disease, cirrhosis, and hepatocelluar
carcinoma. Wilson's disease can cause acute hepatitis, chronic active hepatitis, and
cirrhosis.
Q20:
Answer: 5
EBV causes acute hepatitis that never progresses to cirrhosis.
Q21:
Answer: 4
1- True, so anemia may be MISSED easily.
2- Apart from pregnancy (which are commonly seen in the 2
nd
and 3
rd
trimesters), if
these are seen in a healthy person, it is better to investigate the liver; they are seen in
up to 2% of healthy adults.
3- True, as well as prominent gynecomastia.
4- False, indicates an advanced disease, and some regards ascites per se is an
indication for hepatic transplantation in cirrhosis.
5-True, as well as low grade fever.
Q22:
Answer: 5
Unfortunately, hepatitis E in pregnancy carries a mortality rate up to 20%. Remember:
up to 80 % of hepatitis C infections will become chronic, and acute infection is
usually asymptomatic. The question's options are "long"; this is for teaching purposes
only and better not to be put in the answer explanations! In reality, there are no such
LONG options!
Q23:
Answer: 5
The 1
st
4 items predict a poor response to INF alpha treatment. Criteria for initiating
INF alpha treatment in chronic hepatitis B infections are:
1- Raised serum aminotransaminases.
2- Chronic active hepatitis picture on liver biopsy.
3- The infection was not acquired trans-placentally.
4- HIV negative status.
Q24:
Answer: 5
Hepatocellular carcinoma as a complication is surprisingly uncommon despite the
changes in liver histology. The disease occurs in exacerbations and remissions and
eventually will end with cirrhosis. Corticosteroids are effective in the treatment of
acuteattacks and at prevention of future attacks but they do not prevent the
progression to frank cirrhosis.
There are many associations, especially in type I, like ulcerative colitis, nephrotic
syndrome, Coombs positive hemolytic anemia, autoimmune thyroid disease etc.
So you should be careful when facing these complications as these will complicate
and confuse the picture. Type I is ANA and anti-smooth muscle antibodies positive;
while type II is anti-LKM antibodies positive.
Q25:
Answer: 5
There is proliferation of the endoplasmic reticulum, and together with mitochondrial
swelling, are seen only with the aid of electron microscope.
Q26:
Answer: 3
Although xanthelasmas are seen and Hypercholestremia is common, it is
predominantly of HDL type and hence is cardio protective! Liver transplantation is
the only curative step.
Q27:
Answer: 5
Corticosteroids and immune-suppressants are of no value at all .Liver transplantation
is the only curative step.
Q28:
Answer: 5
The fibrolammellar variant has a good prognosis and is NOT associated with hepatitis
B or C infection or cirrhosis and serum alpha fetoprotein is usually normal.
Q29:
Answer: 5
The patient should be ASYMPTOMATIC or at best having very mild symptoms.
Q30:
Answer: 5
Pregnancy and oral CCP are risk factors for cholesterol stone formation.
Chapter V / Nephrology Answers
Q1:
Answer: b
Hydroxylates 25 - hydroxycholechalciferol to its active form. Remember this
CONVERSION is impaired EARLY in chronic renal failure (not late), so that renal
bone disease is almost always seen in established uremias.
Q2:
Answer: c
a- True, you should know the size when READING the ultrasound report.
b- True, but clinically may not be that apparent.
c- False, about 1 million in number.
d- True, hence in hypovolemia, there is rapid activation of the renin angiotensin
system.
e- True, because of the site and size of the liver. This is important when SEEING the
nephgrogram phase of IVU.
Q3:
Answer: e
Smoking produces SIADH (and makes you urinate less). Remember: polyuria is not a
synonym to frequency. Frequency is the passage of FRQUENT yet small amounts of
urine, but not more than 2 liters / day. Polyuria is the passage of more than 3-4 liters
per day of DILUTE urine. Other causes of polyuria : nephrogenic and cranial diabetes
insipidus. Remember: one of the earliest abnormalities of early chronic failure is
polyuria and NOCTURIA, due to increased osmotic load per nephron and
hyperactivity of the remaining normal nephrons.
Q4:
Answer: c
a- True, other disadvantage is that the PRINTED images convey only a fraction of the
information gained by performing the investigation in real time.
b- True, in clinical practice this is applicable.
c- True, and differentiates renal cysts from solid tumors.
d- False, the density is INCREASED a tricky one!
e- It is the ratio of peak systolic and diastolic ratio, and influenced by the resistance to
flow through small intra-renal arteries, and elevated in many intrinsic renal diseases
like acute glomerulonephritis and renal transplant rejection. Also, the Doppler study
may be used in the detection of renal vein thrombosis and renal artery stenosis.
Q5:
Answer: d
You should avoid diuretics before hand. Remember contrast nephropathy is
unfortunately a common IATROGENIC mistake. Knowing the risk, careful patient
selection, with application of certain precautions (like good hydration, avoidance of
diuretics, and stopping metformin in diabetics), is important in reducing the risk of
contrast nephropathy.
Q6:
Answer: e
There is an excellent definition of the collecting system and ureters on the AP films.
Remember: in any imaging investigation, you should know the availability, cost,
advantages, disadvantages, complications.
Q7:
Answer: C
a- False, through the skin directly into the kidney.
b- False, under ultrasound guide.
c- TRUE, you need a large dilated kidney to insert your needle with ease.
d- False, used in obstructive uropathy.
e- False, an excellent view can be obtained.
Q8:
Answer: b
a- False, this is the main indication, especially in children with recurrent UTIs.
b- True, also used in the assessment of urinary bladder emptying and urethral
abnormalities.
c- False, it is done by directly and retrogradely injecting the dye through a uretheral
catheter.
d- False, especially in children.
e- False, but it is rarely used here.
Q10:
Answer: d
There is high risk of contrast nephropathy in certain patients (long standing diabetes,
multiple myoloma...etc), and risks of intra-arterial "accessing" like local hematoma or
bleeding at the femoral entry site, and intra-arterial manipulation like cholesterol
atheroembolic disease. Therapeutic intervention may be undertaken at the same time
of doing renal angiography like dilatation and stenting of renal artery stenosis and
occluding an AV fistula.
Q11:
Answer: d
Isolated hematuria with DEFORMED RBCs (ie RENAL hematuria). All other options
are true.
a- For example, looking for an evidence vasculitis causing a rapidly progressive
glomerunephritis which may have an excellent response to immune suppressive
therapy.
b- Chronic renal failure with small sized kidney is the usual picture, but when the size
is normal, then we have to biopsy the kidney because the etiology might be reversible
upon receiving an appropriate therap.
c- For example, the presence of hematuria, hypertension ,poor response to steroids.
d- False.
e- True, nephrotic syndromes in adults are not commonly caused by minimal change
nphropathy, so we have to know the cause because the management and prognosis are
totally different from that of minimal change nephropathy.
Q12:
Answer: d
Predicted renal size less than 60% is a contraindication. Remember: transplant
rejection is not a contraindication to renal biopsy (it may direct you to what to do
next). Biopsy from a single kidney is a relative contraindication and can be done
safely by an experienced operator.
Q13:
Answer: a
a- False, not all cases, some types dont not discolor urine.
b- True, alkaptonuria.
c- True, he is on L-dopa.
d- True, he is on rifampicin.
e- True, myoglobinuira.
Other causes: beetroot, hemoglobinuria, hematuria, and other drugs like senna.
Q14:
Answer: d
A/C ratio of less than 2.5 in males and 3.5 in females
Remember: minor leaks of albumin in urine may occur in: After heavy exercise,
fever, heart failure, exposure to extreme cold or heat weather, after general (especially
abdominal) surgery, extensive burns.
Remember: Bence J ohn's protein is POSITIVELY charged but albumin is
NEGATIVLEY charged, and the dipsticks detect negatively charged proteins. An
example of a clue to BJ proteinuria is the presence of urine protein of 2 g/ d with
negative protein dipsticks!
Q15:
Answer: b
a- False, the commonest causes.
b- TRUE, toxic and ischemic types.
c- False, almost always reversible causes.
d- False, uncommon.
e- 5% only for acute glomerulonephritis, and 10% for acute interstitial diseases.
Q16:
Answer: d
Complicated acute renal failure may have a mortality approaching 50-70% even at
best centers. Other statements all are true.
Q17:
Answer: e
Also by impaired consciousness causing aspiration. Dont simply assume that the
respiratory rater is rapid because of uremic acidosis; always look for the above causes
in the appropriate clinical setting.
Q18:
Answer: e
Hypophosphatemia may cause hemolysis and anemia and usually seen in those with
aggressive dialysis whether hemo- or peritoneal. Notice that hyperphosphatmeia is
more common than hypophosphatemia in renal failures in genrral.

Q19:
Answer d
Dipstick analysis of urine is done every day in labs for protein, glucose, bilirubin ,
ketones etc; however, many doctors dont know the FACTORS that influence the
interpretation of such tests. A difficult question I agree, just written for teaching
purposes. Significant glycosuria and contrast media in urine can cause abnormally
HIGH specific gravity.
Q20:
Answer: e
In the appropriate clinical setting suspecting UTI, a rapid and relatively cheap way of
confirming UTI is the dipsticks for leukocyte esterase (from WBCs lysis) and nitrite (
by the action of nitrate reductase on nitrate forming nitrite), yet many factors
unfortunately affect the interpretation of these tests. Medications which discolor urine
will give false POSITIVE results for nitrite.
Q21:
Answer: e
Calorimetric reagent strips are used for the detection of urinary PROTEINS.
Q22:
Answer: d
It is very important in the follow up of both types of diabetes. It is defined as either"
persistent proteinuria between 30-300 mg / day or between 20-200 microgram /
minute on 2 or more occasion, at least 6 months apart ". ACE inhibitors are very
important in the management, even in those with normal blood pressure. It is a very
powerful predictor for the future development of overt diabetic nephropathy and
atherosclerosis. Remember: neither the mechanism of the microalbuminuria nor an
explanation for these associations has been found. Remember: urinary protein of more
than 300-500 mg/ day is a FRANK proteinuria and is dipstick positive.
Q23:
Answer: e
Between 0.5-2 grams / day is considered as "source equivocal " and may be
glomerular or tubular, so further investigations are required.
Q24:
Answer: e
Lab features suggestive of pre-renal failure:
1-high specific gravity, usually more than 1.018.
2-urine osmolality more than 500 mosm / Kg (usually more than 600).
3-urinary sodium less than 20 meq / L (usually less than 10).
4-fractional sodium excretion less 1.
5-urine sediment: normal clear hyaline casts.
While in established acute tubular necrosis:
1-less than 1.010.
2-less than 320(usually around 280).
3-more than 20.
4-more than 1.
5-muddy brown granular casts with tubular epithelial cellsrespectively.
Q25:
Answer: b
a- True, both are responsible for up to 65% of cases.
b- True, although useful, unfortunately it s a late sign.
c- True, raised PTH per se, high calcium by phosphate product, and "unknown
toxins".
d- True, due to diarrhoea, vomiting, and salt loosing nephropathy.
e- True, unfortunately.
Q25:
Answers: e
These factors should always be looked for as their removal may profoundly slow the
progression of the renal failure .These includes:
1-urinary tract infection.
2-urinary tract obstruction.
3-nephrotoxic medications.
4-any infection (hyper catabolic state).
5-hypertension.
6-reduced renal perfusion: due to drug induced hypotension, renal artery stenosis,
sodium and water depletion, and cardiac failure.
Q26:
Answer: b
The half life of insulin is prolonged and hence the total daily doses should be reduced
for fear of hypoglycemia.
Q27:
Answer: d
Option "d" is wrong. Those with tubulo-interstital diseases, renal cystic disease,
obstructive uropathy and reflux nephropathy have what is called salt loosing
nephropathy; hence water and salt loss may be profound and may further deteriorate
the picture, so these should be replaced carefully.
Q28:
Answer: e
Difficult question, but you should know these figures as these reflect the advances in
renal replacement regimens and the overall outcome is not that gloomy as it was in
the past and many patients with CRF now have a good quality of life for many years.
Q29:
Answer: e
It may be done every day (usually in those with severe hypercatabolic state) or every
other day.
Q30:
Answer: e
Remember: there are 3 options in the treatment of renal artery stenosis :
1-medical with anti-hypertensive medications, low dose aspirin and lipid lowering
drugs.
2-angioplasty with or without stenting.
3-surgical resection of the stenotic segment.
Remember: At present there is no conclusive data to indicate the overall superiority of
one approach over another. Each patient should be managed individually. MRA is
now the screening method of choice for renal artery stenosis.
Q31:
Answer: b
It is usually X linked recessive and most cases are due to abnormalities in tissue
specific type IV collage alpha 5 chain due to mutations in the gene COL4A5 at Xq22.
Q32:
Answer: d
10% will develop subarachnoid hemorrhage although 15-40 % are having Berry
aneurysms. The risk may increase to 20% in those with a positive family history of
ruptured Berry's aneurysm.
Q33:
Answer: e
Although hypertension is common and should be treated aggressively, yet
hypotension due to slat loosing or wasting state may occur and necessitates fluid and
salt replacement.
Q34:
Answer: c
The cysts are usually confined to the papillary collecting ducts. Remember: the
commonest causes of nephrocalcinosis are:
1-primary hyperparathyroidism.
2-medullary sponge kidney.
3-renal tubular acidosis type I (not II).Others are rare, like old healed TB and
sarcoidosis.
Q35:
Answer: d
Hyperphosphaturia is found here with profound hypophosphatemia. Hypercalciuria is
a feature of type I (distal), not type II. Other causes of this syndrome: cystinosis and
hereditary fructose intolerance.
Q36:
Answer: d
Incomplete forms are well documented in which the serum bicarbonate is NORMAL
but the urine fails to acidify below 5.3 upon ammonium chloride administration.
Q37:
Answer: e
Other causes: membrano-proliferative glomerulonephritis especially type II, and
cryoglobulinemia.
Remember: systemic necrotizing vasculitides are pauci-immune and do not produce
hypocomplementemia.
Q39:
Answer: e
The work up of these causes is long and complicated and depends on the overall
clinical picture. The main presenting feature is the progressive rise of blood urea
nitrogen and creatinin within few days-weeks.
Q40:
Answer: a
It is an autoimmune disease against alpha 3 chain of type IV collagen. May present as
RPGN picture, or with lung hemorrhages with dyspnia, coughetc, or with both. IgG
anti GBM antibodies can be detected in the majority of patients.
Q41:
Answer: c
Membranous nephropathy will show granular subepithelial IgG deposition. Post
infectious and type I membranoproliferative glomerulonephritis will show
subendothelial deposits. Lupus nephritis, always positive with many pictures can be
seen depending on the type.
Q42:
Answer: c
Surprisingly, absence of hematuira portends a BAD prognosis!!! All other options are
true.
Q43:
Answer: d
a- True, n Balkan nephropathy.
b- True, in Chinese herbs nephropathy.
c- True, as well as causing proximal RTA and Fanconi's syndrome.
d- False, Hanta virus infection (also CMV, and leptospirosis ) is a cause of ACUTE
interstitial nephritis.
e- True, in analgesic nephropathy.

Q44:
Answer: d
a- True, like NSAIDS and antibiotics.
2- True, but eosinophiluria is seen up to 70% of cases.
3- True as the majorities are non-oliguric.
4- False, suggestive of a drug induced etiology.
5- True, but sometimes a tapering course of steroids is usually given to enhance
recovery although its effect is questionable.
Q45:
Answer: e
Double micturition is used in reflux nephropathy: ie bladder emptying to be followed
by another micturition after 10-15 minutes, usually used at night.
Q46:
Answer: e
A radiolucent stone per se is not an indication for intervention. Other indications: the
presence of a recurrent or intolerable pain.
Q47:
Answer: 3
Hypercitraturia is protective, while hypocitraturia is a risk factor
Q48:
Answer: e
Radiotherapy and chemotherapy are very weakly effective. IL II is used as a palliative
in metastatic disease. Remember: anemia (36%) is more common than polycythemia
(4%), and hypercalcemia due to secretion of PTH related peptide occurs only in 5% of
cases.
Q49:
Answer: e
Progesterone like drugs may be used to slow the advancement of metastatic disease
although the impact on the overall prognosis is poor .
Q50:
Answer: d
Cisplatin induces loss of magnesium through tubular dysfunction mechanism.
Q51:
Answer: d
Membrano-proliferative glomerulonephritis type II is associated with C3 nephritic
factor and partial lipodystrophy. Membraboproliferative glomerulonephritis type I is
associated with hepatitis B infection, cryoglobulinemia (with or without hepatitis C
infection), and bacterial infections. Goodpasture's syndrome is associated with HLA
DR15 (previously known as HLA DR2).
Chapter VI/ Electrolytes and Acid-Base Disturbances
Answers
Q1:
Answer: d
Water moves between different compartments by a passive process.
Plasma water=3 liters
Interstitial water =9 liters
Intracellular water =27 liters
These amounts are approximate in a healthy 65 Kg male
Q2:
Answer: e
It can be buffered by CATAIONIC proteins like albumin and hemoglobin, a tricky
one!!
Q3:
Answer: e
The first 4 factors increase the amount of potassium that is excreted in urine. Such a
simple physiology should be kept in mind, and can explain the various causes of
hypokalemia of a renal tubular origin.
Q4:
Answer: a
65% of the filtered sodium is reabsorbed there. The above facts explain the
abnormalities seen in Fanconi's Syndrome (complete proximal renal tubular acidosis
type II).
Q5:
Answer: c
65% of the filtered water is absorbed in the proximal tubule. Other options are true.
Q6:
Answer: e
Ethacrynic acid is a LOOP diuretic. Other causes: NSAIDS, calcium channel blockers
and vasodilators.
Q7:
Answer: e
Diuretic resistance is very common in clinical practice and we should know these
factors. e.g. a patient with end stage renal disease is given 20mg lasex, and then 40
mg Lasix but still no urine !!!! This is non-sense as the kidneys are already
functionless.
Q8:
Answer: e
Remember: hyponatremia with HIGH ECF volume:
A-heart failure.
B-renal failure.
C-SIADH (syndrome of inappropriate secretion of ADH).
Remember: hyponatremia with NORMAL ECF volume:
A-nephrotic syndrome
B-hypothyroidism.
C-diuretics.
D-NSAIDS.
Answer: 3
Remember: alcohol makes you urinate MORE and smoking makes you urinate LESS.
Q11:
Answer: c
Such a urinary osmolality may indicate compulsive water drinking causing water
intoxication and dilute urine, or diabetes insipidus (urinary osmolaity in SIADH is
usually around 460 mmol / Kg .i.e. inappropriately high).
Q12:
Answer: d
It is beta blockers, other causes: spironolactone, amiloride and triamterene.
Remember: hypokalemia potentiates digoxin toxicity, but in digoxin toxicity
hyperkalemia may occur.
Q13:
Answer: a
Bicarbonate infusion reduces serum potassium by 0.2-0.4 meq / L, so it is a weak
agent; anyhow, it is used in severe acidosis (but watch for circulatory overload).
Calcium resonium is used in the chronic setting. Calcium gluconate infusion is just a
cardio protective agent; it does lower the serum potassium.
Q14:
Answer: d
a- True, due to muscle fiber necrosis, and even rhabdomyolysis.
b- True, usually seen as a difficult weaning from a ventilator in the ICU setting.
c- True, due to impaired red cell membrane ATPase.
d- False, HYPERcalciuria and HYPERmagnisuria.
e- True, usually resistant to anti-arrhythmics.
Q15:
Answer: e
Volume expansion may be a cause of hypophosphatemia. You should know the
causes of hypophophatemia, they are commonly seen in MRCP examination.
Other causes: nutritional recovery syndrome, parenteral nutrition, insulin infusion,
glucose infusion, alkalosis (respiratory and metabolic), oral phosphate binders, and in
the way of treatment of diabetic ketoacidosis.
Remember: Hypophophatemia is USUALLY multi factorial and commonly co-exist
with other electrolyte imbalance.
Q16:
Answer: e
Remember: drug induced hypomagnesaemia: cisplatin, gentamycin, and loop
diuretics.
Remember; magnesium is very poorly absorbed orally, hence oral preparations are of
no use in deficiency states. Other causes: chronic diarrhea, excessive lactation,
hyperparathyroidism, primary and secondary aldosteronism, upper GIT fistula and
protein energy malnutrition.
Q17:
Answer: e
a- True, Diamox (carbonic anhydrase inhibitor).
b- True, by creating uretrosigmoidostomy (a form of ureteric diversion used in the
past).
c- True, due to HCL content.
d- True, and type I and II.
e- False, DKA in the way of recover and early (not late) uraemia , may cause normal
anion gap metabolic acidosis. Normal anion gap metabolic acidosis is one of the
favorite subjects in the MRCP (both part I and II).
Q18:
Answer: d
In ethylene glycol poisoning there is accumulation of oxalic acid and glycolic acid.
Q19:
Answer: c
Type B is caused by metformin (also sorbitol, isoniazide, and salisylates).
*Type A (also seen in any severe shock, carbon monoxide poisoning) in general is
caused by profound hypotension and or severe anemia.
*Type B (also seen in hepatic failure, severe infections, ethanol, and methanol
poisoning) in general is caused by impaired mitochondrial respiration and increased
lactate production.
Q20:
Answer: e
a- True, if excessive.
b- True, due to direct stimulation of the respiratory center +metabolic acidosis.
c- True, the clue is tetany +normal or increased PaO2 +high PH +very low PaCO2).
d- True, and pulmonary embolism.
e- False, METABOLIC alkalosis due to loss of HCL in vomitus.
Chapter VII / Endocrinology Answers
Q1:
Answer: e
a- True, and hence a sequential or a dynamic testing should be used.
b- True, and hence, a biochemical diagnosis should be done before imaging.
c- True, like adrenal adenomas versus carcinomas.
d- True, and if you suspect a hormonal deficiency then choose a stimulation test.
e- False, the usually story.
Q2:
Answer: e
As the name implies, it is an apoplexy, so the presentation is a catastrophic one.
Lateral extension may involve the 3
rd
, 4
th
and 6
th
cranial nerves. Remember: a
TSHoma is responsible for 1 % of all functioning pituitary tumors.
Q3:
Answer: d
a-True, and radiotherapy is a second line treatment.
b-True, and radiotherapy is a second line treatment.
c- True, notice that radiotherapy is more effective in children, and also used to
irradiate the pituitary to prevent Nelson's syndrome.
d- False, dopamine agonists are the first line agents even in macroadenomas unless it
is cystic and or large.
e- True, unfortunately octreotide does not cause tumor shrinkage.
Q4:
Answer: e
Pergolide is a dopamine agonist used in the treatment of hyperprolactinemia. Causes
of hyperprolactinemia are many, and it is one the favorite topics in the MRCP
examination. Other causes are: pregnancy and lactation, medications (oral
contraceptive pills, metoclopromide, reserpine, methyldopa, antipsychotics,
antidepressants, opiates and antiandrogens). However, the commonest causes are:
1- Disconnection hyperprolactinemia, as in non-functioning pituitary
macroadenomas.
2- Prolactinomas, usually microprolactinomas.
3- Primary hypothyroidism.
4- Polycystic ovarian syndrome.
Q5:
Answer: e
External irradiation is used for some macroadenomas to prevent re-growth if
dopamine agonists are stopped. All other options are true.
Q6:
Answer: e
External irradiation although causes shrinkage of the tumor, yet the growth hormone
level takes long to return to normal and there is a high risk of panhypoituitarism.
Q7:
Answer: a
Sampling should be done post exercise, not pre exercise. Other tricks are true.
Q8:
Answer: b
The 250 microgram tetracosactrin injection can be given at ANY time during the day.
Because it relies on ACTH dependent adrenal atrophy in secondary adrenal failure so
it may not diagnose acute adrenal failure secondary to acute ACTH deficiency.
Q10:
Answer: e
Unlike primary hypothyroidism, measuring TSH is not helpful because the pituitary
may secrete glycoproteins which are usually detected by TSH assay but they are NOT
BIOACTIVE. So the aim is to KEEP the T4 in the upper part of reference range
during replacement therapy with T4.
Q11:
Answer: e
a- True, and in the assessment of GH deficiency.
b- True, and in SEVERE hypopituitarism..
c- True, with signs of neuroglycopenia.
d- True, at time 0, 30, 45, 60, 90, 120 minutes post administration.
e- False, it is given as soluble insulin 0.15 u / kg intravenously.
Remember: at the end of the test, the serum GH should be >20 mu / L, and serum
cortisol >550 nmol/ L.
Q12:
Answer: d
DDAVP should not be given if you suspect a severe compulsive water drinking as this
will prevent water excretion and risks severe water intoxication if the patient
continues to drink water.
Q13:
Answer: e
pruritis (with increased sweating), palmar erythema, and spider nevi, skin
pigmentation (but vitilligo is much more common), and clubbing, all are seen in
thyrotoxicosis per se .
Q14:
Answer: c
a- True, and slightly raised serum bilirubin.
b- True, very mildly.
c- False, up to 5 % only, and almost always mild.
d- True, due to associated diabetes mellitus and lag storage.
e- True, a well documented feature.
Q15:
Answer: d
a- True, 0.2% will develop severe reversible agranulocytosis during carbimazole
therapy.
b- True, and in those dependent on voice e.g. singers.
c- True, and planned pregnancy with in 6 months.
d- False, 10% will develop TRANSIENT hypocalcaemia following surgery, and 1%
only will develop long term permanent hypocalcaemia.
e- True, thus regular follow ups are important.
Remember: clinical improvement in a patient on carbimazole is seen after 2 weeks but
the biochemical abnormalities will normalize after 4 weeks.
Q16:
Answer: c
a- True, ike coxsackie, mumps, and adeno viruses.
b- True, and byswallowing, and movement of the neck. The pain may radiate to the
jaw and ears.
c- False, the ESR is usually raised and a low titer of thyroid antibodies non-
specifically detected transiently.
d- True, but steroids are occasionally used for severe cases.
e- True, apart from oral propranolol in certain cases and no need for anti-thyroid
drugs.
Q17:
Answer: d
The combination of:
Suppressed TSH, negligible radio-iodine uptake, low serum thryroglubulin, and
greatly raised T4:T3 ratio (usually around 70:1) are diagnostic.
Q18:
Answer: d
There is NO association between post partum depression and post partum thyoriditis.
It is supposed to be an autoimmune in origin.
Q19:
Answer: d
a- True, also precipitated by surgery and radio-iodine treatment in a previously
inappropriately treated patient.
b- True, but should always be kept in mind and the patient should be rendered
euthyroid before the operation.
c- True, anti-thyroid measures are: oral or IV propranolol, oral sodium iopodate (a
radio-contrast medium will restore serum thyroid hormone levels to normal with in 2-
3 days), oral potassium iodate or Lugol's solution, oral or per rectal carbimazole.
d- False, there is NO parenteral preparation for carbimazole , if can not be given
orally , then give it rectally or by an NG tube.
e- True, and the patient should continue on carbimazole.
Q20:
Answer: e
Post ablative and primary atrophic hypothyroidism are both not goitrous.
Q21:
Answer: e
Iron deficiency anemia is a common finding in premenopausal females due to heavy
menses.
There 8 RARE but well recognized features, although SEEN in many textbooks and
QUESTIONS and BOFs, yet in clinical practice if you are in an endocrinology clinic
you will see that they are rare:
1-Myotonia.
2- Cereberallar ataxia.
3- Frank psychosis.
4- Pleural and pericardial effusions and heart failure.
5- Ileus.
6- Impotence (while in thyrotoxicosis it is common!!).
7- Galactorrhea.
8- Ascites.
Q22:
Answer: d
READ the question well "Biochemical findings that are useful in the assessment of
hypothyroidism". T3 is not a reliable test to differentiate between a euthyroid and
hypothyroid patient. Also, we may see: normochromic anemia, low voltage ECG,
bradycardia and non specific ST-T changes indicate PROLOBGED SEVERE
hypothyroidism.
Q23:
Answer: d
Remember: Subjective improvement is seen after 2-3 weeks, and the reduction in
body weight and periorbital puffiness occurs relatively rapidly, BUT restoration of
skin and hair texture and resolution of any effusion may take 3-6 MONTHS.

Q24:
Answer: e
a- True, a rare presentation rather than to be a complication of an already diagnosed
patient.
b- True, as well as raised CSF protein, so you it may be missed as a PRIMARY CNS
DISEASE.
c- True, it is not a "cold case".
d- True, and start treatment with intravenous hydrocortisone before the biochemical
diagnosis.
e- There is NO parenteral preparation of T4, BUT there is a parenteral form of T3 and
so T3 can be given intravenously.
Q25:
Answer: c
Option "c" is the classical finding in those who are non-compliant and take large
doses before the follow up visit to fool the doctor!
Q26:
Answer: e
Phneytoin is an enzyme inducer, so the dose of thyroxin may need to be increased in
hypothyroidism and concomitant use of phenytoin.
Q27:
Answer: e
Obesity is rarely due to a single gene mutation. Endocrine diseases associated with
obesity are: Cushing' syndrome, hypothyroidism, and polycystic ovarian syndrome.
Q28:
Answer: d
Follicular carcinoma and adenoma are very similar on FNA cytology, but thyroid
biopsy is needed to demonstrate vascular invasion in cases of carcinomas.
Q29:
Answer: e
Secondary tumors are rarely seen.
Q30:
Answer: b
a- True, it is an aggressive one.
b- False, it is part of MEN type II.
c- True, as there is no curative treatment.
d- This, clinically speaking it is only a tumor MARKER!
e- True, because the para-follicular cells dont trap iodine and hence it has no role in
the treatment.
Q31:
Answer: e
Even when caused by an adenoma, the tumor is rarely felt in the neck (usually located
at surgical exploration).Other options are true.
Q32:
Answer: e
Secondary bone malignancy is a cause of hypercalcemia with undetectable
parathyroid hormone level (also seen in vitamin D intoxication, milk alkai syndrome,
sarcoidosis , Addison's diseasecet. ). The 1
st
4 options are the cause of
hypercalcemia with normal or high serum parathryoid hormone level.
Q33:
Answer: d
The hypercalcemia of primary hyperparathyroidism does not respond to steroids.
High dose glucocorticoids are very effective in the treatment of malignancy-
associated hypercalcemia patients.
Q34:
Answer: a
It may caused by hypoparathyroidism and pseudohypoparathyroidism. Also may
cause seizures, and Parkinsonian like picture. Pseudopseudohypoparathyroidism
resembles pseudohypoparathryoidism pheonotypically, but it has a normal
biochemical profile.
Q35:
Answer: e
Pseudopseudohypoparathyroidism resembles pseudohypoparathyroidism
phenotypically, but it has a normal biochemical profile.
Q36:
Answer: d
pseudohypoparathyroidism is treated by vitamine D metabolites like alpha calcidol
tablets and follow up the patient with serum PTH and serum calcium.
Q37:
Answer: e
1- True, and weight gain is the commonest symptom.
2- True, so it is seen in 75% of cases.
3- True, as well as prominent hypokalemic alkalosis.
4- True, frank psychosis is uncommon.
5- False, type II fiber atrophy.
Remember:
Striae are seen in 50%
Proximal myopathy 50%
Bruising 50%
Obesity as a sign is seen in 97% of cases, so 3 % are not obese.
Q38:
Answer: c
a- True, as the stress of hospitalization may interfere with many tests and some
normal persons will even FAIL to suppress on overnight dexamehtason suppression
test (i.e. behaves like Cushing's) because the hypothalamic- pituitary-adrenal axis will
ESCAPE such suppression due to powerful endogenous stress mechanisms.
b- True.
c- False, it does not cross react and that's why it is used in the suppression test.
d- True, hence called pseudo Cushing's.
e- True, and very high levels indicate an ectopic source.
Q39:
Answer: e
Without treatment, the 5 year survival rate is 50%. All other options are true.
Q40:
Answer e
a- True, and liquorice abuse.
b- True, and congenital adrenal hyperplasia due to 11 beta hydroxylase and 17 alpha
hydroxylase deficiency.
c- True, causing hypokalemia, hypertension and low aldosterone.
d- True, and 11 beta HSD deficiency.
e- False, Glucocorticoids suppressible hyperaldosterosnism and Conn's adenoma and
idiopathic bilateral adrenal hyperplasia , all are the cause of PRIMARY
aldosteronism.
Q41:
Answer: e
Although there is an avid sodium retention, leg edema is uncommon and suggests
secondary aldosteronism or a complicating heart failure. Serum potassium is normal
in up to 70% of cases at the time of diagnosis because many patients are already
treated by salt restriction making less sodium available to be exchanged for potassium
at the distal tubule.
Q42:
Answer: c
1- True, like MEN type II, neurofibromatosis, and Von Hipple Lindau.
2- True, and predominantly elevated adrenalin indicates an adrenal tumor that is not
large enough to outgrow its blood supply.
3- False, it commonly occurs in the absence of diabetes.
4- True, indicating a high level of dopamine.
5- True, in urinary bladder tumors.
Remember: although pallor is commonly seen during the attack, occasionally
FLUSHING is seen.
Q43:
Answer: e
a- TB is the next, hence CXR should be done in all cases.
b- True, and when presents with buccal hyperpigmentation, both are highly
suggestive.
c- True, Addison's disease per se causes fasting hypoglycemia.
d-True.
e- False, it is a rare disease with an incidence of 8 new case/ million of population.
Male to female ratio is 1:2.
Q44:
Answer: d
a- True, up to 90% of cases, and a late onset form with hirsutism is a recognized
presentation.
b- True, many enzymatic deficiencies are documented.
c- True, and salt losing nephropathy and crises in neonates, usually in males.
d- False, associated with hypertension due to increased 11 deoxycorticosterone, a
powerful mineralocorticoid.
e- True, and plastic surgery has a place for the ambiguous genitalia in females
Q45:
Answer 5
Other drugs: estrogens, and GnRH analogues given for prostatic carcinomas.
Q46:
Answer: e
LH: FSH ratio more than 2.5:1.Also there is: hypertension, hyperglycemia,
hyperlipidemia, hirsutism, oligomenorrhea or secondary amenorrhea, and infertility.
Remember : PCOS is treated according to the PRERSENTING feature .eg She is
complaining of infertility, so treat the infertility, or complaining of hirsutism so treat
the hirsutism . DONT TRY TO TREAT EVERYTHING! However, weight reduction
in obese is important in the overall management.
Q47:
Answer: e
In idiopathic hisutism, female family history is very important, especially seen in
Asians or Mediterraneans.
Q48:
Answer: b
a- True, and mutation in RET proto-oncogen on chromosome 10 in type II.
b- False, hypercalcemia is the commonest presenting feature of type I, while in type
IIa it is uncommon and in type IIb it is absent.
c- True, hence family history is very important.
d- True, unlike the sporadic ones (10% bilateral).
e- True, also Cushing syndrome is uncommon in type I.
Q49:
Answer: e
a- True, next is the cecum and appendix.
b- True, due to obstruction of the mouth of the appendix.
c- True, because they have a low malignant potential.
d- True, Pulmonic stenosis, tricuspid regurgitation and right sided endocardial
fibrosis.
e- False, Cramping abdominal pain and diarrhea with flushing and wheeze are the
commonest presenting features of carcinoid SYNROMS not tumors.
Q50:
Answer: d
1- True, and steatorrhea and achlorhydria.
2- True, and skin rash and diabetes.
3- True, or watery diarrhea and multiple severe peptic ulceration.
4- False, with HYPOKALEMIA, hence called pancreatic cholera.
5- True, due to hypoglycemia.
Chapter VIII / Diabetes Mellitus Answers
Q1:
Answer :5
Nesidioblastosis is a diffuse beta cells hyperplasia and a cause of early life recurrent
hypoglycemia
Q2:
Answer 4
Concordance rate in monozygotic twins is around 40-50 %
Q3:
Answer 5
Concordance rate in monozygotic twins is almost 100 % implying strong yet
unidentified genes involved in the pathogenesis.
Q4:
Answer 5
1-MODY type I, a rare progressive early onset form
2-MODY type III. Progressive early onset
3-MODY type II , mild and relatively stable early onset type
4-very rare
5-it has a maternal inheritance, may be part of MELAS (mitochondrial
encephalopathy with lactic acidosis and stroke like episodes)
Q5:
Answer: 3
It inhibits the action of lipoprotein lipase and hence it enhances lipogensis .
Q6:
Answer: 3
No smoking is allowed and the patient should sit during the test (not walking around
to kill time)
Q7:
Answer 5
The Fastingblood glucose of more than 11.1 mmol/L on 2 or more occasions is
diagnostic for diabetes mellitus
Q8:
Answer 2
Total fat should be 30-35%
Saturated fat should be <10%
Monounsaturated fat should be 10-15%
Polyunsaturated fat should be <10%
Carbohydrates 50-55%
Total protein 10-15%
Q9:
Answer 2
Insulin does reduce post prandial glycemia and has a favorable effect on lipid profile
Q10:
Answer 5
1-and anorexia, weight loss
2-and cholestatic jaundice
3-hnece contraindicated in heart failure
4-due to salt and water retention in the short term
5-diarrhea and flatulence
Q11:
Answer 3
It causes hypothermia , MI ,TIAs
Q12:
Answer 1
Unexpected or unusual exercise may cause hypoglycemia if the original dose is not
reduced.
Other causes: malabsorption, errors in anti-diabetic agents dosing and schedules and
administration, impaired hypoglycemia awareness.
Q13:
Answer: 5
Leukocytosis is seen in DKA per se in the absence of infection.
Confusion, drowsiness and Coma occur in 10% of cases.
Q14:
Answer: 1
DIC is rare
Q15:
Answer 4
Mortality usually exceeds 40% even in best centers
Q16:
Answer 5
Venous loops and beadings are seen mainly in PRE-proliferative stage
Q17:
Answer 4
It is mainly sensory
Q18:
Answer 4
The nodular glomeruloscleorsis type is less common than the diffuse type
Q19:
Answer: 4
1-to assess the risks and benefits of treatment and complications of diabetes
2-so that it is controlled at the time of conception
3-and maternal and fetal morbidity and mortality
4-contraindicated, insulin is used
5-due to the physiological and stress changes during pregnancy
Q20:
Answer 5
Hospital admission is needed when there is a cardiovascular or a renal disease and a
poor diabetic control.
Chapter IX / Hematology Answers
Q1:
Answer: 5
1- important in dialysis patients and causes resistance to erythropoietin therapy.
2- especially the minor trait.
3- especially the minor trait, its clue is that the patient is an Asian one with normal
hemoglobin electrophoresis and normal ferritin.
4- and sideroblastic anemias :clue is a dimorphic blood picture
Remember: Microcytosis and hypochromia have a relatively narrow differential
diagnosis so you have to know it because it is a common clinical problem.
5-False, aplastic anemia causes either a normocytic or macrocytic anemia.
Q2:
Answer: 5
Remember: macrocytosis is very common in clinical practice but megaloblastic
anemia is not that common and the term macrocytosis indicates an increase in the
MCV, but megaloblastic changes are pathological changes seen in the cells mainly in
the nucleus. Fish tapeworm infestation causes vitamin B12 deficiency and hence
megaloblastic changes. Note the first 4 items cause characteristic changes in the bone
marrow.
Q3:
Answer: 5
Item 5 causes characteristic changes in the bone marrow.
Q4:
Answer: 5
Dimorphic blood picture is a common problem seen in clinical medicine. It has 4
causes in general that you HAVE TO KNOW:
1- Blood transfusion in a patient with pre-existent macro- or microcytosis.
2- In the way of recovery of a hematinic deficiency (i.e. during its replacement).
3- Combined deficiency states, simultaneously or in succession .e.g. celiac disease.
4- Primary sideroblastic anemia where there is 2 CLONES, one is the normal and the
other is the abnormal one producing the macrocytes.
In item 5, phenytoin causes macrocytosis exacerbated by pregnancy.
Q5:
Answer: 5
1- True, usually tolerated because of the rightward shift in oxyhemoglobin
dissociation curve.
2- True, regular blood transfusions are uncommonly indicated except in those with
recurrent life threatening crisis like recurrent stroke.
3- True, the objective is to reduce the HbS, and NOT TO RAISE THE PCV which if
is high, may induce stagnation and further sickeling!
4-True, unfortunately.
5- False, up to 10% of children.
Q6:
Answer: 4
1-True, be careful as there may be a co-existent iron deficiency anemia causing a
falsely low or normal HbA2.
2- Together with iron chelation therapy and daily folic acids.
3- The usually story, and then confirmed by hemoglobin electrophoresis.
4- False, may be due to severe longstanding anemia causing high out put cardiac
failure.
5-True, and profound dyserythropoiesis.
Q7:
Answer: 5
Hemolytic disease of the new born causes mainly extra vascular hemolysis.
Q8:
Answer: 4
1- Any neurological syndromes +fever +skin rash =exclude TTP.
2- True for unknown reason.
3- Purpuric rash or frank ecchymosis.
4- False, impaired coagulation systems (PT and aPPT) indicate DIC, these are normal
in pure HUS or TTP.
5- To prematurely release platelets from the bone marrow.
Q9:
Answer: 3
1- and eczema and other skin diseases.
2- and filiariasis, ascariasis.
3- False, treatment with steroids causes neutrophilia and eosinopenia.
4- usually forgotten.
5- PAN and esosinophilic granuloma.
Q10:
Answer: 5
1- and bone marrow recovery from radiotherapy.
2- and inflammatory bowel disease, SLE.
3- and myeolomonocytic leukemia.
4- and TB, typhus, brucellosis, SBE, malaria, trypanosomiasis.
5- false, causes monocytopenia.
Q11:
Answer: 5
1- as in Arabs and blacks.
2- like hepatitis A.
3- ususally associated with low platelets.
4- and other drugs like penicillamine.
5- false, causes neutrophilia which is a useful clue to differentiate it from viral
hepatitis.
Q12:
Answer: 4
The common ALL type has a good prognosis. The T cell type presents with a
mediastinal mass and a very high leukocyte count and has a gloomy prognosis.
Also t(9, 22) cytogenetic abnormality has a very poor prognosis (unlike in CML
where its presence confers a good prognosis).
Q13:
Answer: 2
Knowing the cytogentics of AML is very important, not only for the prognosis but
also to decide the future treatment like bone marrow transplantation in those with
poor cytogenetics.
M2-t(8,21)
M3-t(15,17)
M4-inv16
Also in ALL: t(1,19) in pre-B ALL, t(8,14) ALL L3 Burkitts subtype
Q14:
Answer 4
1- True.
2- Retenoic Acid Receptor Alpha.
3- True, hence a rapid development of neurtrophilia is seen.
4- False, hypergranular.
5- True and usually asymptomatic to start with.
Q15:
Answer: 4
1- True, but remember; Ph chromosome is found in 5% of childhood ALL, 25% of
adult ALL and 1% of adult AML.
2- True, due to its inhibitory effect on the BCL-ABL fusion gene product.
3- True, a useful clue.
4- False, serum vitamin B12 is high due to high vitamin B12 binding protein.
5- True, other causes of low LAP score: PNH and some cases of aplastic anemia and
MDS.
Q16:
Answer: 5
1- and 13q abnormalities, and both portend poor prognosis.
2- and warm immune hemolytic anemia and ITP like picture may be seen.
3- true.
4- as in many lymphoproliferative disorders.
5-FALSE, the doubling time is usually long and the cell turnover low.
Q17:
Answer: 5
1- as in many hematological malignancies.
2- and ESR is high and indicates an active disease.
3- and hence it is difficult to know weather these represents lymphomatous masses or
not; PET scan may be used to differentiate between them.
4- and lymphocytic predominant has good prognosis.
5- Fasle, with EBV infection.
Q18:
Answer: 5
1- true, myeloma high ESR is responsible only for 3% of cases of ESR>100.
2- keep it in mind ,on the other hand there may be no serum paraprotein, but only
urinary Bence J ohns protein.
3- true, infected stem cells may be responsible for the secretion of high level of IL-6.
4- true, as there is an overall inhibitory effect on bone osteoblasts.
5- false, only in the presence of a fracture.
Q19:
Answer: 5
Immune paresis is in favor of MM, as well as bone and renal diseases.
Q20:
Answer: 5
1- and in neonates.
2- and smoking, COPD and morbid obesity.
3- and other high affinity hemoglobins.
4- and renal cell carcinoma
5- false, a primary one.
Q21:
Answer: 5
Other causes, systemic inflammatory disorders and vasculitis, trauma, infections.
PRV may have a high platelet count but it is not a reactive process because it is part of
the panmyelosis process seen in PRV.
Q22:
Answer: 5
1- true, mainly it is a disease of elderlies.
2- like hypo granular neutrophils , abnormal neutrophil nuclear lobulation, vacuolated
erythroblasts.
3- other causes of death are from infection and bleeding.
4- and secondary MDS is seen after chemotherapy and radiotherapy.
5- false, refractory anemia with excess blasts in transformation.
Q23:
Answer: 5
1- indicates the presence of inhibitors in the patient's plasma(unlike hemophilia).
2- bilateral adrenal vein thrombosis.
3- may be confused with SLE per se.
4- and epilepsy, TIAs, stroke and multi-infarct dementia.
5- false, arterial and venous thrombosis, if hemorrage occurs it is either due to over
anticoagulation or due to severe thrombocytopenia.
Remember: the aPTT is already prolonged making heparin therapy follow up is
difficult.
Q24:
Answer: 5
1- many cases are mild and discovered later in life.
2- becauses it induces further thrombocytopenia, other types have normal platelets
count.
3- true, the only useful indication of bleeding time application.
4- but an abnormal vWf multimers.
5- some cases are autosomal recessive and usually the dominant ones have a variable
expression.
Q25:
Answer: 5
1- hence no family history is present.
2- characteristic, and the platelets may be elevated after a bleeding episode like a GIT
bleeding or due to recurrent bleedings causing iron deficiency anemia and
thrombosytosis.
3- in severe cases should be always the first line IF AVAILABLE.
4- if does not correct this indicates the development of inhibitors.
5- its activity is between 50-150%.
Q26:
Answer: 5
1- and also SLE.
2- in contrast to children.
3- and 2/3
rd
responds to splenomegally after failure of steroids.
4- its role mainly is to exclude other diseases like ALL in children, and antiplatelets
antibodies are not used routinely in the diagnosis.
5- false, such a history is usually obtained in children not adults.
Q27:
Answer: 5
Treatment with steroids (together with melphalan) helps to stabilize the disease and
may produce improvement.
Q28:
Answer: 3
HIGH serum Beta 2 microglobulin level is a bad prognostic sign.
Q29:
Answer: 5
1- for many reasons, dehydration is common.
2- about 55%, IgA 21%, light chain only 22% and others including non secretory
about 2%.
3- true, hypogammaglobulinemia is common and severe. Bone marrow failure is an
advanced stage in the course of the disease.
4- true, and may indicate the need for plasmapharesis.
5- indicates a recent fracture. MM per se does not raise serum alkaline phosphatase
due to inhibitory cytokines on osteoblastic function and hence radio isotope scan is
also normal.
Q30:
Answer: 5
1- hence symptoms of infection are common.
2- true, relatively has a slow pace.
3- can be seen when performing flow cytometry in the diagnosis.
4- and to deoxycofermycin.
5- false, splenomegally is seen in up to 90% of cases and may be massive, but lymph
node enlargement is highly unusual (unlike CLL).
Q31:
Answer: 2
1- many options are there but the treatment is mainly supportive.
2- false, chromosomes 5 and 7.
3- usually in elderly population, but in theory no age is exempted.
4- like following alkylating agents or etoposide.
5- with dysplastic, NOT megaloblastic changes.
Q32:
Answer: 3
1- true, and all other cases are secondary, like infections, tumors, drugs.
2- smudge cells are seen in CLL; the presence of spherocytes per se may be a clue to
AIHA but not its cause.
3- direct Coombs test detects IgG and complement but not IgA or IgE antibodies
hence may be falsely NEGATIVE when AIHA is associated with these antibodies.
4- true hence those with low tires may be falsely NEGATIVE.
5- yes, also seen in ITP.
Q33:
Answer: 4
1- these are usually mild. More severe cases either indicate a coexistent defect of
second different protein or coincidental polymorphism of ALPHA spectrin.
2- and indicates the need for spelnectomy.
3- as well as growth retardation in children and death of a family member from the
disease.
4- false, 25% only.
5- a must BUT may become positive after many transfusions i.e. alloimmunization!!
Q34:
Answer: 5
1- hence during the acute hemolytic episode dont assess the enzyme level as it may
become transiently normal or even high.
2- a useful clue in acute attacks.
3- so you have to give an enough HISTORY to the hematology lab so as to use this
stain.
4- especially after receiving the water soluble analogue of vitamin K.
5- false, the Caucasian and the Oriental types are the most severe when compared
with the African type.
Q35:
Answer: 4
1- true, it just provides a clue to them like hypersegmented neutrophils
2- true, but (written in some textbooks) a peripheral neuropathy state may be seen in
folate deficiency also.
3- true, anorexia ,alcohol and phenytoin can severely depress the serum level in the
absence of deficiency state; also a single meal may totally NORMALISE it in a
severely deficient patient .So Red cell folate is better than serum folate level.
4- false, may precipitate severe hypokalemia.
5- and reticulocytes percentage may reach 50% with in the first week of treatment.
Remember: Pregnancy is the commonest cause of megaloblastosis world-wide and
more likely to be seen in the context of multiple pregnancy, and multiparity.
Q36:
Answer: 5
Hemosidrinuria, hemoglobinuria, and reduced serum haptoglobin all are seen.
Q37:
Answer: 4
1- and levels down to 100 microgram/ L are still suggestive of IDA in chronic
inflammatory illnesses.
2- serum iron fluctuates by 30-50% on daily basis and even diurnal basis. It is low in
acute phase responses.
3- and in nephrotic syndrome and malnutrition .It is raised in pregnancy and oral
CCP.
4- false, less than 16 %.
5- measured by immunoassay.
Q38:
Answer: 5
They (bronchospasm, angiodema and hypotension) predominate the clinical picture.
Q39:
Answer: 5
Ensure urine out put of at least 100 ml / minute.
Q40:
Answer: 1
Item 1 is wrong because he has a clear cut precipitating factor, besides this is the first
episode, so no need to screen.
Chapter X / Rheumatology Answers
Q1:
Answer: 3
1-true, is an autoantibody.
2-but, we detect only IgM in the slide agglutination test.
3-false, does not reflect any thing apart from poor prognosis if found in high titer in
RA.
4- like slide tube agglutination, SCAT and Rose Waaler tests.
5-true, must be seropositive here.
Q2:
Answer: 5
1- as it rises rapidly and falls rapidly and hence reflects the degree of the APRs.
2- true, as it rapidly rises from a low to high levels in acute inflammatory process and
closely mirrors the degree of inflammatory.
3- true, in cases of improvement in the APR, the CRP normalizes rapidly while the
ESR LAGS behind for a variable period of time.
4- and in polycythemia rubra vera (PRV), so in these cases we measure the plasma
viscosity as a measure of APR (remember in PRV there is florid APR; yet the ESR is
very low).
5- the CRP is elevated in the presence of infection of trauma in SLE, other wise in
acute flare ups it is normal;a useful CLUE!
Q3:
Answer: 4
1-hypochromic microcytic one is seen up to 25 % but does not respond to iron
therapy. It is not an iron deficiency states, but there is defects in ferro-kinetics and
iron utilization; unfortunately may be complicated by true iron deficiency anemia eg
long term treatment with NSAIDS.
2- true, useful guide to the overall activity eg in RA.
3-true, varieties of pictures, and may be complicated by drug side effects on the blood
count and bone marrow ,so keep it in mind.
4-leukocytosis and neutrophilia eg in classical PAN.
5-true, a common cause of neutrophila.
Q4:
Answer: 4
1- eg using rodent organs or human cell lines.
2- like rheumatoid factor, low tires may be seen in healthy normal people in a good
percentage.
3-hence its name.
4- FALSE, 100% of cases; a negative titer virtually excludes drug induced lupus (here
it is mainly anti histone H2 A and B subtypes).
5- depends on the antigen preparation used in the test and whether we detect IgG or
IgM type; however the tests are unfortunately NOT standardized and liaison with
local labs is important.
Q5:
Answer: 5
1- with a fall from standing height or less.
2- also radiological evidence of osteoporosis, and premature gonadal failure.
3-remmeber, not only the disease may pose a risk but also its treatment like
corticosteroids.
4-also, clinical features of osteoporosis, like loss of height and kyphosis.
5-false ,BMI less than 19; hence obese persons are usually protected.
Remember: polycystic ovarian syndrome patients although may be infertile with
irregular or absent cycles, but the high BMI and the good amount of estrogens and
androgens protect them from osteoporosis.
Q6:
Answer: 5
1- as the isotop is taken by body bones.
2- also in primary and secondary bone tumors.
3- as the Tc-biphosphanate later localizes to areas of bone remodeling; theses LATE
images are usually taken after few hours.
4-true, the main indication in general.
5-false, it depends on the osteoblatstic activity which is inhibited in multiple
myeloma; so it is not useful here, but only if it was complicated by fractures.
Remember: although bone scan has a high sensitivity, it extremely lacks specificity
and lacks high anatomical resolution.
Q7:
Answer: 5
Other indications: reflex sympathetic dystrophy and hypertrophic osteoarthropathy;
and useful in the assessment of the extent of Pagets disease of the bone. In multiple
myeloma (MM), it depends on the osteoblatstic activity which is inhibited in MM, so
it is not useful here, only if it was complicated by fractures.
Q8:
Answer: 1
1- false, less than 20 and more than 50 years of age.
2- and a history of a major trauma.
3- and the presence of a painful spinal deformity, severe symmetrical spinal
deformity, saddle anesthesia, progressive neurological signs in the lower limbs,
sphincter dysfunction, and a sensory level
4- very important indicators.
5- and HIV, malignancy, on long term steroids...etc.
Q9:
Answer: 5
90% of cases improve after 6 weeks so it has an excellent prognosis. Other features:
Tendency for recurrent episodes, pain is limited in the back or thigh but NEVER
below the knee, and no clear cut root signs.
Q10:
Answer: 5
In general, the prognosis is reasonable with 50% recovery at 6 weeks.
Q11:
Answer: 4
1- so it is common.
2- true associations, also hyperinsulinemia.
3- true, at least 4.
4- FALSE, there is NO such features which are (with posterior apophyseal joint
involvement) indicate spinal spondylosis.
5-true, very rarely have symptoms, usually low back pain.
Q12:
Answer: 5
1- and from capsular fibrosis and intervening large osteophytes; There is palpable,
sometimes, audible coarse crepitus.
2- so called "good days and bad days".
3- also may increase upon joint movement.
4- usually one or few painful joints (rarely multiple painful areas).
5- false, NO such joint instabilities
Q13:
Answer: 5
1- ogether with trauma; usually are monoarticular.
2- like J IA .
3- and the so called endemic osteoarthritis.
4- and hemachromatosis.
5- FALSE, arthropathy is seen up to 50% of cases of acromegally (not in GH
deficiency).
Q14:
Answer: 5
1- true, you should treat appropriately to minimize disability.
2- true, they are safe and cheap.
3- true, with excellent results.
4- true, with no significant drug-drug interaction and are often effective at relieving
pain.
5- false, a common age associated phenomenon, and may precipitate acute
pseudogout attacks.

Q15:
Answer: 4
In Caucasians, the prevalence is about 1-1.5 % with female to male ratio of 3:1
Q16:
Answer: 1
HLA DR4 is the major susceptibility haplotype in most ethnic group eg found in 75%
of Caucasian patients with RA, and HLA DR4 positivity is more common in those
with severe disease. HLA DR1 is more important in Indians and Israelis while HLA
DW15 is more important in J apanese.
Q17:
Answer: 2
1- true, with infiltrations by lymphocytes, plasma cells and macrophages.
2- CD4 positive cells.
3- similar granulomatous lesions are seen in the sclera and pericardium, pleura and
lungs.
4- fibrous or bony ankylosis occurs late.
5- the regional lymph nodes draining the actively inflamed joints are frequently
hyperplasic.
Q18:
Answer: 5
1- and when rupture may simulate DVT, but remember both may coexist together as
the patient may be immobile and there is an increased risk of DVT, so be careful!!
2- usually seen in extensor body surfaces, sclera and lung.
3- Episcleritis is a benign complication but sclertits is serious.
4- seen in up to 30% of nodular seropositive patients, but constrictive pericarditis is
rare.
5- the CNS is surprisingly SPARED in RA vasculitis.
Q19:
Answer: 1
1- false, less than 1 % only.
2- and nodular seropositive disease.
3- and weight loss, recurrent infections, and sicca syndrome.
4- and normochromic anemia, defective T and B cell function and neutropenia.
5- usually between 50-70 years of age.
Q20:
Answer: 4
1- mouth ulceration and Pemphigus are both rare.
2- drug induced lupus and Goodpasture's syndrome are also rare.
3- rapidly falling platelet count, mild thrombocytopenia, and proteinuria are
indications to stop the drug and reintroduce it slowly, but if they recur then stop it for
good.
4- febrile reactions and pancytopenia are absolute indication to stop the drug
IMMEDIATELY and for GOOD.
5-and glomerulonephritis.
Q21:
Answer: 3
Marrow suppression and aplastic anemia may occur and both carry a significant
mortality.
Q22:
Answer: 2
1- an association with HLA B27.
2- peripheral asymmetrical oligoarthritis affecting the lower limbs more than the
upper.
3- but there is association with enthesopathy.
4- true.
5- like iritis (up to 20-25%), and aortitis(4%) which are the commonest extra
particular manifestations.
Q23:
Answer: 2
The peak incidence is in the 2
nd
and 3
rd
decades. The incidence is higher in Pima and
Haida Indians because of higher prevalence of HLA B27. The Kelbsiella carriage may
be responsible for joint and eyes disease flares ups.
Q24:
Answer: 3
1- occurs in 25% and conjunctivitis in 20%.
2- although extra spinal involvement is seen up to 40%.
3- false, up to 1% only; also aortitis, aortic and mitral regurgitations and cardiac
conduction defects.
4- because there will be loss of hip flexion which can not compensate for the spinal
rigidity.
5- true and the patients should receive bisphosphanates.
Q25:
Answer: 4
Male to female ratio is 15:1. May be preceded by: chlamydia urethritis; and GIT
salmonella, shigella, campylobacter, and yersinea infections.
Q26:
Answer: 2
Circinate balanitis is seen in up to 20-50% of cases, and are usually painless and
easily escape notice.
Q27:
Answer: 2
Synchronous onset of skin disease and joint disease is uncommon and seen in up to
5% of cases.
Q28:
Answer: 2
1- notice that arthritis mutilans is seen in 5% of cases only.
2- FALSE; they are found more commonly in psoriatic arthritis (85%) than in
uncomplicated psoriasis (30%).
3- true, like genitals, natal clefts and scalp.
4- but conjunctivitis is much more common.
5- they are asymmetrical, coarse, and non marginal syndesmophytes and
asymmetrical sacroiilitis.
Q29:
Answer: 3
Serum levels of uric acids are higher in males than females. It is a disease of post
pubertal males and post menopausal females.
Q30:
Answer: 5
Azapropazone is a uricosuric agent; other causes: pyrazinamide, alcohol and lead.
Q31:
Answer: 5
High dose aspirin promote urate excretion by the kidney. Other indications: gout with
a very high serum uric acid level, and hyperuricemia due to congenital enzyme
defects like HGPRTase deficiency.
Q32:
Answer: 5
1- the usual story in those on thiazides of more than 18 months duration.
2- true and the hands, not the feet, are the usual targets in those patients.
3- as 100mg / day.
4- true, followed by early mobilization.
5- NSAIDs and Colchicine are best avoided because of increased incidence and
severity of toxicities.
Q33:
Answer: 3
Hyper- not hypoparathyroidism. Other causes: hemaochromatosis and
hypophophatasia.
Q34:
Answer: 5
Tophi are the hallmark of prolonged hyperuricemia, not pseudogout, although both
may coexist!
Q35:
Answer: 5
1- true, it is not a cold case.
2- true, with appreciable morbidity.
3- inpatient management, not on an outpatient basis.
4- but the culture is positive in only 25% of gonococcal cases, while the genital tract
cultures are positive in 70% in these cases!
5-false, staph aureus.
Q36:
Answers: 4
1- true, age and female genders are risk factors.
2- no age is an exempt, but it s rare in young people.
3- only 2 consistent associations had been shown: sleep abnormality and abnormal
pain perception.
4- false, the ESR should be normal.
5- true , this is an evidence based intervention.
Q37:
Answer: 5
Tibolon is a hormone receptor modulator that acts as a partial agonist at the estrogen,
progesterone and androgen receptors, and used in the treatment of osteoporosis.
Q38:
Answer: 5
Vitamin D and calcium decrease the risk of NON-vertebral osteoporotic fractures, and
data are still not available regarding the reduction in VERTERBRAL fractures.
Q39:
Answer: 5
Only 3 medications till now have been approved to decrease the risk of NON-
vertebral osteoporotic fractures: Biphosphanates, Hormonal replacement therapy and
Calcium +Vitamin D
Q40:
Answer: 4
1- usually seen in the context of chronic renal failure; very rare now a days.
2- a congenital defect in bone specific alkaline phosphatase.
3- true, no need for calcium at all.
4- false, type I responds well to active vitamin D metabolites; type II is very resistant
(together with hypophsphatasia, are both the most difficult to treat).
5- a true diagnosis by bone biopsy.
Q41:
Answer: 5
1- indicates that the bone targeting insult occurs early in life.
2- causing bone thickening with abnormal architecture and reduced mechanical
strength.
3- in the majority of patients.
4- usually in those with limited cardiac reserve.
5- osteosarcoma is RARE and portends a very POOR prognosis and should be
suspected in any rapid increase in pain or swelling of an affected bone.
Q42:
Answer: 1
There is polyclonal B and T cell activation
Q43:
Answer: 5
1- also there may be discoid rash, photosensitive rash, livedo reticularis, lupus
panniculitis and lupus profundus , purpura , skin ulceration , nail fold infarcts, bullous
lesions .
2- but contractures may occur due to tendon inflammation and may be seen as bone
deformity (J accoud's arthropathy).
3- true, portends a poor prognosis and should be treated aggressively.
4- the effusions are exudative; rarely chest pain is due to MI.
5- false, mild neuropsychiatric manifestations (like depression or psychosis) and
seizures .
Q44:
Answer: 2
Erosive arthropathy is highly uncommon.
Q45:
Answer: 4
Pulmonary hypertension is 6 times more common in the LIMITED type than the
diffuse type.
Q46:
Answer: 3
ANA is seen in almost 100% of SECONDARY Sjogrens syndrome in RA. In the
primary one it is seen in about 50-70% of cases. Anti Ro and anti La also between 50-
70% of cases and both are more suggestive of primary Sjogren's syndrome.
Q47:
Answer: 3
The CPK is usually raised, sometimes to a very high level. Anti J o1 Antibodies
correlate more with pulmonary parenchymal involvement.
Q48:
Answer: 5
1- true, and other features reflect systemic involvement like MI, stroke, and renal
failure.
2- sometimes esosinophilia, like in Church Strauss vasculitits.
3- true, and the treatment is mainly targeted against the hepatitis B infection.
4- and sometimes methotrexate is used.
5- cANCA is found up to 90% of active Wegners granulomatosis.
Q49:
Answer: 4
Giant cell arteritis should be treated by high does of steroids like 80 mg / day
prednisolone.
Q50:
Answer: 1
Glucocorticoids cause: proximal myopathy with type II fiber atrophy, osteonecrosis,
and osteoporosis (not osteomalacia).
Chapter XI / Neurology Answers;
5 Subchapters
Subchapter A:
Q1:
Answer: 5
The word EEG does not mean epilepsy, and EEG is not used to diagnose epilepsy,
although it is the most sensitive investigation in epilepsy. Epilepsy is a clinical
diagnosis that can be solidified by EEG. Metabolic myopathies usually have normal
EMG e.g. in hypokalemic weakness; and here there are no characteristic changes on
EMG testing.
Q2:
Answer: 4
1-false, many bone artifacts would interfere with the picture.
2-false, both are excellently seen.
3-false, MRI gives better details.
4-true, but should avoided in claustrophobic people.
5-fasle, unfortunately it is highly operator dependent.
Q3:
Answer: 2
1-true, may be raised in many CNS diseases; however in many other diseases it might
be normal even in the presence of profound CNS insult like NPH, MSetc.
2-false, usually 60-70% of the blood levels, levels below 50% are always abnormal
3-true above 50 mg / dl is abnormal.
4-true, up to five cells is normal, all should be mononuclear.
5-true, oligoclonal bands indicate intrathecal synthesis of IgG, and is seen in many
CNS diseases (not only MS!!).
Q4:
Answer: 4
1- hence anticholenergics are given in upper motor neuron lesion to dampen the
bladder.
2- and alpha blockers cause retrograde ejaculation.
3- true.
4- the reverse is true, in acute lesions, retention is seen early to be followed gradually
by urgency and frequency of micturition.
5- true, usually sequamous in type.
Q5:
Answer: 2
Unlike the common belief; visual fields are actually detected much BETTER by a
neurologist rather than by an ophthalmologist (who orders formal perimetry and
doesn't know how to detect them by his own!). Any how, visual field defects are
common in clinical neurology:
1- true, and with later constriction of the peripheral field. Remember: any
papilloedema with severe visual impairment, then it is NOT papilleodema, it is
BILATERAL papillitis!
2- true, tubal vision and tunnel visions are not the SAME; tubal vision: the visual field
remains with the same constriction as we go farther from the patient , but in the tunnel
vision , it improves as we move away from the patient ( just imagine yourself looking
from inside a tube and then inside a tunnel! they are different). Tubal vision is highly
characteristic of conversion disorders. Tunnel vision is usually seen chronic simple
glaucoma.
3- ie a chiasmal lesion, also a chiasmal lesion may cause a junctional scotoma.
4- may be a case of retrobulbar neuritis e.g. multiple sclerosis. In these cases, the
patient sees NOTHING and you see NOTHING abnormal also!
5- seen in a relatively early chronic simple glaucoma.
Q6:
Answer: 5
Facial sensation is usually badly examined in clinical neurology, you should always
be efficient in it, as it may reflect and localize many CNS diseases.
1- The V1 and V2 branches of the trigeminal nerve might be affected.
2- and others CP angle lesions as in acoustic neuroma; here loss of corneal reflex is an
early feature, so use it as a screening tool!
3- C2 nerve supplies the angle of mandible; it is still a facial area!
4- true, but with a hemisensory loss in the limbs (eg pure sensory lacunar stroke).
5-keep it in mind: any neurological abnormality found on examination e.g. facial
anesthesia , then it is not an idiopathic trigeminal neuralgia.
Q7:
Answer: 4
Those who dont practice neurology think that this condition is rare; no it is not, it is
commonly seen in clinical neurology and has many causes like bilateral hemispheric
strokes , upper brain stem lesionsetc.
1- spastic Donald duck speech (as if the patient tries to speak from the back of the
mouth).
2- oropharyngeal type. i.e. with regurgitation, chocking, coughing.
3- true, very characteristic if you see it once.
4- FALSE, small spastic conical one.
5- true.
Q8:
Answer: 5
Many conditions can MIMIC brain death clinically upon examination and without
excluding them you will KILL a person legally despite the reversibility of brain
damage.
These are:
Profound shock, sedatives drug intoxication, hypothermia (core body temperature
below 32 C) and neuromuscular blockade use .Children usually resist brain damage
better than adults for the same degree of insult, so give them a chance and be
extremely careful when diagnosing brain death in a child less than 5 years old.
Dont think that the diagnosis of brain death is an easy task (seems very easy when
read in textbooks), it is a terrible responsibility, legally, ethically and medically.
Q9:
Answer: 5
Remember: the frontal lobes have many functions, motor, behavioral, personality,
urination, praxis...etc. Olfactory hallucinations are seen in temporal lobe lesions (the
frontal lobe is close to the nose and olfactory nerves but has nothing to do with
olfaction!)
Q10:
Answer: 1
1- true, may indicate a brainstem lesion like Wallenberg's lateral medullary syndrome.
2- false, although being a prominent feature, it may be absent or very mild so as to
escape detection.
3- false, central one is usually mild although it is continuous.
4- fasle, a differential diagnosis if the vertigo is extremely episodic.
5- fasle, peripheral, indicating an internal ear pathology.
Q11:
Answer: 5
You should always look at the patient while entering your room as his gait may SAY
a lot of things while the patient can't talk at all. Gait abnormalities are very common
in neurology, and a patient may be on a wheel chair HIDING his abnormal gait.
Waddling gait is suggestive proximal myopathy including osteomalacia, and rarely
bilateral long standing congenital hip dislocation.
Q12:
Answer: 4
1- the patients usually describes the pain as if it is of many hours duration, actually
speaking , it is lancinating in seconds only but the rapid repetition of many episodes
with in a short period gives a false impression of many hours duration.
2- the neurological examination, apart from certain triggering ZONES, should be
normal.
3- usually middle aged and elderly females.
4- true, carbamazepine is very effective in the majority.
5- multiple sclerosis must be excluded here.
Q13
Answer: 5
Pseudobulbar palsy is an upper motor neuron lesion including affection of the tongue
muscles; with the tongue being small conical and spastic with NO fasciculation; it can
be caused by bilateral hemispheric lacuanr strokes which does not cause a bulbar
palsy. All other options cause lower motor neuron lesion of the 12
th
cranial nerve with
atrophy and fasciculation of the tongue.
Q14:
Answer: 5
1- FALSE, sharp waves are highly specific in the appropriate clinical settings with a
false positive rate of 0.1%.
2- false, only in highly selected patients e.g. in a brain tumor patient with one attack
of seizure there is a place to start PROPHYLACTIC anticonvulsants as the pathology
and the etiology is obvious and persistent; otherwise wait for a second and recurrent
attacks (which is the definition of EPILEPSY).
4- false, prominent ones.
5-TRUE, so follow up is very important in this period with proper education of the
patient and his family.
Q15:
Answer: 5
1- true, he can be seen and investigated as an outpatient; only in highly selected
patients hospitalization is needed for assessment e.g. presentation with status
epilepticus or the need for certain inpatient investigations.
2- one of the corner stones in the diagnosis; your eyes or others eyes!!
3- true, effective and safe.
4- but urination during the attack is not that specific.
5- FALSE, his picture is typical of idiopathic generalized grand mal epilepsy. Imaging
studies are done when the clinical picture is suggestive e.g. focal features of the fits,
abnormal neurological examination, mental retardation and Todd's paresis.
Q16:
Answer: 5
1- and of hearing, or vision and usually well formed ones.
2- in simple ones, there is no impairment at all.
3- or undue familiarity.
4- true, you should always think of secondary generalization.
5- FALSE, suggests focal motor epilepsy of the motor cortex in the frontal lobe.
Q17:
Answer: 2
1- true, early there is an enlargement of the physiological blind spot which is almost
always asymptomatic. Severe impairment of vision here indicates a co-existent
pathology or it is actually a bilateral papillitis (not a papillodema).
2- FALSE, suggests an alternative diagnosis like optic neuritis.
3- with contralateral optic atrophy in Foster Kennedy Syndrome.
4- true, may indicate a rapidly progressing process.
5- true, there may be other signs also like bradycardia and hypertension.
Q18:
Answer: 5
1- true , a large percentage.
2- unfortunately.
3- a good percentage.
4- true, but the long term prognosis is better than that of ischemic stroke.
5- false, up 20 %, other 20% is the lacunar type.
Q19:
Answer: 3
Headache may be seen in supra as well as infratentorial strokes.
Q20:
Answer: 5
1- true, which results in prominent basal meningitis and cranial nerve palsies and
hydrocephalus.
2- usually the 6
th
cranial nerve and which may be bilateral.
3- yes, but sometimes no evidence of chest or military TB is seen.
4- true, and also after starting steroids, in which the CSF cells become mainly
neutrophilic, which is highly characteristic in the appropriate clinical settings.
5-false, up to 25% of cases may present explosively like pyogenic meningitis and
even the CSF study is suggestive of pyogenic cause.
Q21:
Answer: 4
Functional recovery following strokes is an important part to predict the course of the
illness and patient's education and management.
-Coma per se if deep and more than 24 hours carries a very bad prognosis.
-Non-dominant hemispheric lesions have a poor functional outcome.
- Associated hypertension if severe is also another poor prognostic factor.
- Brain stem strokes are usually devastating as many structures are PACKED closely.
Q22:
Answer: 4
1- true, usually presents with out a prodromal phase, even in a stroke like picture.
2- with raised protein and pleocytosis.
3- true, some element of meningitis is usually seen, hence the precise term meningo-
encephalitis.
4- false, unfortunately few have good prognosis, but the majority have a high
morbidity and mortality.
5-true, with no seasonal variation.
Q23:
Answer: 4
1- like HLA DR2, DW2, A3, and B7.
2- suggesting a strong environmental factor.
3- true.
4- false, a strong white matter preference.
5- true.
Q24:
Answer: 4
MS has diverse clinical manifestations, and should always be looked for in any
STRANGE neurological disease.
1- true, but in the long term, sub cortical plaques may encroach upon the cortex and
irritate it.
2- true, of sub cortical type seen in advanced cases.
3- true, dissemination in place.
4- false, usually in the range of 60-80 mg / dl; if it is above 100 mg / dl then look for
another cause.
5-true, but very rare.
Q25:
Answer: 3
Remember, MS has no diagnostic investigation; all investigations are used to support
the CLINICAL diagnosis and clinical criteria are the most important in making the
diagnosis.
1- the evoked potentials are useful to detect silent lesions to detect dissemination in
place.
2- to see CNS white matter lesions, activity, distribution, number, follow upsetc.
3- no place at all.
4- but many diseases can cause oligoclonal bands.
5- true, although poor recruitment might be seen due to the pyramidal weakness in the
limbs for example.
Q26
Answer: 1
1- idiopathic Parkinsons disease, usually asymmetrical or unilateral to start with;
pure bilateral symmetrical onset of tremor is highly atypical.
2-against the diagnosis.
3-it is a basal ganglia pathological process, not a pyramidal one.
4-? Supra nuclear palsy.
5- never an expected finding in idiopathic Parkinson's disease.
Q27:
Answer: 5
MND in general has a grave prognosis; most patients die within 2-6 years of the
diagnosis. No drug therapy till now has been shown effectively to slow down the
disease progression.
Q28:
Answer: 4
1- true, adds confusion to picture.
2- usually in adolescents.
3- true, nothing can be done to stop the degeneration.
4- AD with complete peniterance but with a late onset appearance.
5-true, the chorea may be totally absent at presentation.
Q29:
Answer 4
1-true, hence it might be used as a screening tool to irradiate these areas before any
catastrophe occurs.
2-true, then motor then schincteric.
3-yes; after sensory and motor changes.
4- false, uncommon causes, the majority of cases are due to extradural extramedullary
masses.
5- true may reach 3 grams /dl (Froin's Syndrome).
Q30:
Answer: 5
Paraplegia; means a lesion in the spinal cord or the brain; so we usually start with
examination of the lower limbs, then the trunk and then the upper limbs and then
ascend to see if there is any cranial nerve findings.
1- true as a well as superior saggittal sinus thrombosis.
2- like in MS.
3- like a vertebral fracture dislocation.
4- true, with dissociated sensory loss.
5- FALSE, flaccid asymmetrical weakness in both lower limbs.
Q31:
Answer: 3
Contralateral loss of pain and temperature sensation 2-3 segments below the level of
the lesion.
Q32:
Answer: 5
1- with loss of reflexes.
2- and may cause ugly looking hands, also scars, burns.
3- crude touch and vibration are intact to start with.
4- true, so called syringobulbia, but nystagmus may be seen in high cord lesions.
5- false, an insidious disease.
Q33:
Answer: 5
1- up 40% of cases.
2- true, and always exclude depressive pseudodementia .
3- unfortunately, only 5-10% of all cases of dementias are reversible.
4- true, may be the abnormal chromosome 21 per se is the cause.
5- false, suggests Lewy body dementia.
Q34:
Answer: 5
1- and hyperthyroidsm which may cause also a bulbar weakness old people.
2- usually with evidence of peripheral neuropathy.
3- with type II fiber atrophy.
4- look for Loosers zones, measure serum calcium and phosphate.
5- false, may cause proximal amyotrophy, but not proximal symmetrical myopathy
Q35:
Answer: 4
Duchenne gene, is the largest gene in the body.
1- true , up to 10 years is accepted.
2- calf pseudohypertrophy due to fibrofatty infiltration.
3- usually very high, a normal level is against the diagnosis.
4- false, although cardiac involvement is common, it is rarely the cause of death.
Common causes are: respiratory failure, aspiration, and acute gastric dilatation.
5- true, many explanations were given; one of them suggests that dystrophin is present
normally in brain but it s defective in those patients.
Q36
Answer: 3
1- true, a useful one, and tenslon test should be carried out to differentiate myasthenic
from cholinergic crisis but should be done in the intensive care setting.
2- true, like abdominal cramps and diarrhea.
3- false: not indicated in neonatal myasthenia, pure ocular myasthenia, myasthenia for
more than 7 years duration, congenital myasthenic syndromes, and patients above the
age of 60 years.
4- true as 33% of thymomas are malignant; the prognosis is bad even when the tumor
is completely removed
5- true; during any crisis, stop all medications and admit immediately to the ICU with
assisted ventilation and diagnose the cause of this deterioration and treat as
appropriate.
Q37:
Answer: 5
1- a preceding compylobacter infection portends a poor prognosis.
2- true, but fortunately uncommon.
3- true, you should follow up the patient; ask for dyspnea and measure the FVC
regularly.
4-true, may be subtle.
5- false, seen in up to 65% of cases, which is not a small percentage and may result in
a sudden cardiorespiratoy arrest.
Q38:
Answer: 4
After thorough investigations, up to 1/3
rd
of all cases of peripheral neuropathy, no
single cause will be found.
1- there might be hypochromic anemia, or even a hemolytic anemia.
2- measure urinary ALA and PBG during acute attacks.
3- or a diabetic autonomic neuropathy and diabetic nephropathy.
4- false, hypothyroidism NOT hyperthyroidism may be the cause; hyperthyroidism
does not cause peripheral neurpathy (PN); hypothyroidism may cause a demyelinating
PN.
5- or amyloidosis (small fiber neuropathy).
Q39:
Answer: 4
1- or may be increased during pregnancy or oral contraceptives
2- even to the shoulders
3- true.
4- false, in up to 5% of cases only; due to certain variants in connection between the
median and ulnar nerves; that is why the diagnosis is mainly clinical.
5- ?acromegally, hypothyroidism.
Q40:
Answer: 4
1- such a history is highly characteristic in elderly people.
2- due to smaller brain and stretching of the bridging veins.
3- true, may be very easily missed by CT scan especially if bilateral.
4- false, venous source; hence it is not RAPIDLY formed or explosive in nature after
the start of the bleeding.
5- true and may fluctuate in size.
NB: the classical description of a fluctuating level of consciousness of SDH is rare in
clinical practice.
Chapter XI / Neurology
Subchapter: B
Q1:
Answer: d
a- true, and the incidence and prevalence rises with age.
b- true, when adjusted for age.
c- true, it is a progressive degenerative disease of uncertain cause.
d- false. It is usually sporadic, only 5% of cases are inherited.
e- true. Neuritic plaques are extracellular deposits containingbeta-amyloid protein,
presenilin-1 and 2, alpha-1 antichymotrypsin, apolipoprotein E, alpha-2
macroglobulin and ubiquitin. Neurofibrillary tangles are intracellular deposits
containing hyperphosphorylated tau protein (a micro-tubule associated protein), and
ubiquitin.
Q2:
Answer: e
a- true, beginning at the 4
th
decade in 100% of patients. The abnormal chromosome
21 might confer this susceptibility.
b-true, with mutations in presenilin 2 and 1 respectively.
c-true, but how these APOE4 allels confer disease susceptibility is still unclear.
d-true, and presenilin 1 and 2 appears to be involved in later cleavage process.
e-false, cholinergic neurons are LOST and there is marked REDUCTION in this
enzyme level at these sites.
Q3:
Answer: e
a- true, thus there is marked reduction in the level of choline acyltransferase enzyme
in the cortex and hippocampus of those patients and that's why we give acetyl
cholinesterase inhibitors to increase the level of acetylcholine at these site.
b- true, , but how these APOE4 allels confer disease susceptibility is still unclear.
c- true, with abnormalities in amyloid beta A4 precursor protein.
d- true, and abnormal PS1 gene on chromosome 14 also is responsible for early onset
cases.
e- false, all will develop Alzheimer's disease by the 4
th
decade if they live.
Q4:
Answer: c
a- true, often noticed by the family memebers.
b- true, and then to place.
c- false, these might be prominent early manifestations and thus forcing the patient to
leave work or to give up the management of the family finances early.
d- true, with short slow shuffling steps, flexed posture with wide base and difficulty in
initiating walking.
e- true, causing the patient to become lost easily.
NB: Depression can be very prominent early in the course and may give way to a
restless agitated state.
Q5:
Answer: e
a- true of many types. Myoclonus is rare.
b- psychiatric manifestations gradually become prominent.
c- true, hemiparasis and extensor planters are rare late features.
d- true, a very difficult situation to the caregivers.
e- false, typically after 5-10 years from inanition or infection.
NB: lab investigations and brain imaging studies are used to differentiate it from other
causes of dementia and to exclude other causes of impaired cognition.
Q6:
Answer: a
a- false, till now there is NO single medication that can reverse or arrest the
progression of Alzheimer's disease. Acetyl choline replacement therapy is only
symptomatic and useful in mild cases for about 6 months and its long term benefit is
questionable.
b- true, about 5%, for a period of up to 6 months in mild early cases.
c- true, because of its excellent side effect profile and once daily administration.
d- true, but NOT idebenone, propentophyllin, Gingko biloba and acetyl L carnitin.
e- true, behavioral disturbances are very common at all stages.
Q7:
Answer: e
a- true, and can continue social , recreational and limited professional activities.
b- true, very difficult to be management at home.
c- true, by injudicious actions or decisions.
d- true, long period of suffering.
e- false, typically from inanition or infection.
Q8:
Answer: e
a- true, much earlier than Allzheimer's.
b- true, loss of recent memory is not an early or a prominent feature.
c- true, a very characteristic feature. MRI usually shows global cortical atrophy with
compensatory hydrocephalus in Alzheimer's disease.
d- true, this is virtually diagnostic.
e- false, goes with Alzheimer's disease.
NB: Familial occurrence of frontotemporal dementia has been documented and
mapped to chromosome 17.
Q9:
Answer: e
a- true, characterized by rapidly progressive dementia and startle myoclonus.
b- true, it affects the cerebral cortex, basal ganglia, cerebellum, brain stem abnd spinal
cord.
c- true, with a peak incidence of 60-64 years.
d- true, like Alzheimer's disease.
e- false, more than 1 member of a family is affected in 5-10% pf cases only. Conjugal
cases are RARE.
Q10:
Answer: b
a- true, but human to human transmission (eg by corneal transplant ) is very rare.
b- false, it is present only in the CSF but not in any other body fluid. It is also present
in the liver ,spleen , kidney, eye lung and lymph nodes.
c- true, the cellular isoform is present normally in neurons but its function is still
unkown.
d- true.
e- true, such features occur in fatal familial insomnia.

Q11:
Answer: e
a- true, may start as mild global cognitive impairment.
b- true, and may dominate the clinical picture early in the course.
c- true, as well as hallucinations, depression, labile affect, euphoria, anxiety and
delusional disorders.
d- true, may be induced by startle. Extrapyramidal , pyramidal and cerebellar signs are
also common. Visual field defects, cranial nerve palsies and seizures are less
common.
e- false, it is a rapidly progressive and aggressive fatal disease. Typically results in
coma or akinetic mutism within months.
NB: Characteristic features of new variant CJ D:
Earlier age of onset (mean age, 30 years)
More prolonged course (greater than 1 year)
Invariable cerebellar involvement.
Prominent early psychiatric manifestations
Diffuse amyloid plaques
Q12:
Answer: c
a- true, although in the appropriate clinical settings these changes are highly
suggestive , they are unfortunately transient, and these changes are totally ABSENT
in the variant form of the disease.
b- true, but non-specific at all.
c- false, it is increased in CSF samples. It is a normal brain protein that is also
increased in herpes simplex encephalitis.
d- true, rapid and easy way of diagnosis. In sporadic cases, it can be diagnosed by
immuno-detection of PrPcs in brain tissues obtained at biopsy.
e- true, a non-specific finding.
Q13:
Answer: e
a- true, especially in patients with less fulminant course and paucity of cerebellar and
extrapyramidal signs.
b- true, when subcortical involvement is prominent.
c- true, when cortical focal signs are prominent.
d- true, mean duration of illness is 7 months. Although transient improvement may
occur, it is invariably fatal.
e- false, with 1 year .
Q14: Answer: c
a- true, responds to shunting.
b- true. or secondary to meningitis or subarachnoid hemorrhage.
c- false, the dementia is oftenly mild and insidious and typically preceded by gait
disorder and incontinence.
d- true, focal cortical signs are rare.
e- true, e.g due to meningeal fibrosis and adhesions following meningitis or
subarachnoid hemorrhage.
Q15:
Answer: d
a- true, the earliest manifestation is a gait disorder.
b- true, with unsteady standing and difficulty in initiating walking in the absence of
ataxia or weakness.
c- true, and many patients may be unaware of it. Fecal incontinence is rare.
d- false, uncommon features .
e- true, indicating prominent involvement of the frontal lobes.
NB: There is no papilloedema.
Q16:
Answer: a
a- false, it either normal or LOW.
b- true, and without any obstructing mass lesion.
c- true, together with delayed clearance and failure of ascent over the cerebral
convexities.
d- true, thus ALL patients should undergo this maneuver.
e- true, like shunt infection ,subdural hematoma, and shunt malfunction that
necessitates shunt removal.
Q17:
Answer: c
a- true, up to 1/4
th
pf elderly demented patients who come to autopsy have round
eosinophilic intracytoplasmic neuronal inclusion bodies termed Lewy bodies which
are present in cerebral cortex and brainstem.
b- true.
c- false, Lewy bodies contain alpha synculein ( a protein that is also found in Lewy
bodies in Parkinson's disease) and Tau ( which is present in Alzheimer's disease and
frontotemporal dementias).
d- true, "Lewy-body variant of Alzheimer's disease" and "diffuse Lewy body disease "
and respectively.
e- true, a highly distinguishing feature.
Q18:
Answer: e
a- true, highly characteristic, together without prominent early impairment of recent
memory .
b- true, of well formed type.
c especially rigidity and bradykinesia.
d- true, or tacrine.
e- false, those patients are very sensitive to theextraspyramidal side effects of these
medications and thus medications should be avoided.
Q19:
Answer: b
a- true, usually multi-factorial .
b- false, gliomas arsing from the frontal or temporal lobes or the corpus callosum.
Although such tumors tend to infiltrate the subcortical white matter extensively , they
initially give rise to few neurological signs.
c- true, together with impaired concentration and subtle personality changes.
d- true ,or other cortical signs.
e- true, focal or generized.
Q20:
Answer: d
a- true, in general more common that opportunistic brain infections. At least 50% of
patients experience variable degree of cognitive impairment during later or the pre-
terminal course of the disease.
b- true, it is an AIDS-defining illness that might be the presenting feature.
c- true, by certain neuro-tropic strains.
d- false, HIV-1 doea not appear to replicate within neurons, astrocytes, or
oligodendrocyte IN VIVO, and the loss of these cell types is not prominent in brains
of patients with AIDS dementia complex. It thus has been suggested that neuronal
function is impared by indirect neuro-toxic mechanism. This might involve cytokines
released HIV-infected monocytes or macrophages, viral products such as gp120.
e- true.
Q21:
Answer: a
a- false, the onset is usually insidious with cognitive and behavioral symptoms lie
easy forgetfulness ,apathy, social withdrawal, and motor symptoms like unsteady gait,
leg weakness and impairment in handwriting.
b- true, may seen early .
c- true, indicating an advanced stage.
d- true , with delusions and visual hallucinations.
e- true.
NB; At pre-terminal stages most patients are vegetative.
Q22:
Answer: e
a- true, with ventricular dilatation and hyper-intense subcortical signals on T2-
weighted images.
b- true, not polymorphonuclear .
c- true, non-specific finding, seen also in syphilis, sarcoidosis, MSetc.
d- true.
e- false, the commonest CSF abnormality is an elevated protein which is seen in 65%
of cases .
NB: The CSF could be totally NORMAL.
NB: Most patients usually die with 1-9 months from aspiration or opportunistic
infections.
NB: The course could be steadily progressive or acutely exacerbated by a systemic
sepsis like pulmonary infections.

Q23:
Answer: c
a- true, it was the commonest cause of dementia and psychiatric disorders related to
neurosyphilis in the prepenicillin era.
b- true, followed by global intellectual deterioration with grandiosity , depression,
psychosis and focal weakness.
c- false, these the usual TERMINAL events.
d- true, highly characteristic.
e- true, there might be a dull or vacant facial expression.
Q24:
Answer: b
a- true, or may stabilize or even progress.
b- false, wether symptomatic or not , a persistently abnormal CSF profile indicates
failure of therapy and thus should be RE-treated.
c- true, thus follow up visits are very important.
d- true.
e- true, any how , they should receive treatment .
Q25:
Answer: c
a- true, termed J C virus.
b- true, asymptomatic infections are very common but clinical disease is VERY
RARE. In those with immune suppression, the disease occurs due to virus RE-
activation.
c- false, the virus infects the oligodendrocytes leading to diffuse and patchy
demyelination that primarily affects the white matter of the cerebral hemispheres but
also involves the brainstem and cerebellum.
d- true, leading to death within 3-6 months.
e- true, usually seen in patients with immune suppression like leukemia, lymphoma ,
systemic cancers, organ transplant recipients..etc.
Q26:
Answer: e
a- true, no systemic effect caused by the virus , but systemic manifestations of
immune suppression might be seen like chest infections and oral candidiasis in AIDS.
b- true, altered mentation together with hemiparesis and visual field defects are the
classical presenting features.
c- true, focal cortical signs are very common.
d- true.
e- false, seizures are characteristically ABSENT.
Q27:
Answer: d
a- true, uncommonly may show raised protein or raised white cells.
b- true, indicating diffuse subcortical involvement. The usually findings are:
multifocal superficial subcortical white matter lesions located near the gray-white
matter junction that LACKS mass effect and do NOT enhance following contrast
administration.
c- true, but rarely done in clinical practice. Although PCR for J C virus in the CSF
may be diagnostic , definitive diagnosis is done by brain biopsy.
d- false, results in death within 3-6 months.
e- true, as with amantadine and adenine arabinoside. Unfortunately no treatment is
effective.
Q28:
Answer: e
a- true, but the disease uncommon.
b- true, variable course.
c- true, and finally coma.
d- true, only symptomatic measures and cessation of drinking and improvement in
nutrition is advised.
e- false, the outcome is highly variable: patients may die, survive with dementia, or
recover.
Q29:
Answer: e
a- true, over many years .
b- true, due to aluminum in the dialysate fluids and may be aggravated by aluminum
containing antiacids.
c- true, an aggressive picture resulting in death within 6 months.
d- true, these changes are characteristically reversible by diazepam.
e- false, the disease is now very rare because of removal of aluminum from dialysate
fluids.
Q30:
Answer: e
a- true, should be differentiated from alcoholic cerebellar degeneration which
primarily affects gait.
b- true, and may be punctuated by episodes of acute hepatic encephalopathy.
c- true, some improvement following L-dopa or bromocryptin has been described.
d- true, but paraparesis is rare.
e- false, the CSF is almost always normal apart from raised glutamine level, although
slightly raised protein may be found.
Q31:
Answer: d
a- true, usually between the ages of 50-70 years. Prior history of head trauma is absent
in 25 % of cases.
b- true, and may be easily missed by CT scans because bilateral hematomas are
isodense with the brain and there is no midline shift. The hematomas may be
demonstrated by contrast CT scans. In few cases its demonstration requires cerebral
angiography which should always be done bilaterally. MRI is also useful.
c- true, other risk factors are : alcoholics, epilepsy , treatment with anticoagulations,
cerebral atrophy from any cause, ventricular shunts, and longterm hemodialysis.
d- false, it is the usual first feature to start with.
e- true, seizures are uncommon.

Q32:
Answer: d
a- true, after Alzheimer's disease and dementia with Lewy bodies.
b- true, from occlusion of major cerebral arteries OR small penetrating arteries in the
subcortical white matter , basal ganglia and thalamus respectively.
c- true , for example; the number of strokes, their location and the total infarct volume
required for stroke to produce dementia are still uncertain.
d- false, almost all patients are hypertensive.
e- true, and or with stepwise progression of deficits.
Q33: Answer: c
a- true, with dysarthria, dysphagia and pathological emotionality.
b- true, with bilateral extensor planters.
c- false, the reverse is true.
d- true, also should be done in those without history of hypertension. Also look for
polycythemia, thrombocytosis, and cardiogenic emboli.
e- true, to prevent recurrent strokes. Control hypertension and other risk factors and
treat any associated disease.
Q34:
Answer: c
a- true, and because depression is common and is treatable, distinguishing between
the 2 disorders is very important.
b- true, adding to more confusion to the clinical picture. When depression is being
considered, psychiatric referral should be done.
c- false, prominent affections of sleep, apatite, sexual desire and weight are in favor of
depression.
d- true, these are in favor of depression.
e- true, mood is extremely depressed in the morning hours in depression.
Chapter XI / Neurology
Subchapter C
Q1:
Answer: d
a-true, those patients when seen early after the injury, would exhibit a confusional
state in which they are unable to incorporate new memories (i.e. anterograde
amnesia).
b-true, thus without a history a trauma , it will be diagnosed as a transient global
amnesia rather than a post traumatic amnestic state.
c-true, covering a variable period of events prior to the trauma. The period of
retrograde amnesia begins to shrink gradually, with the most remote memories the
first to return.
d-false, always seen following head traumas that resulted in loss of consciousness.
Events occurring in the post traumatic confusional interval tend to be
PERMINENTLY lost. Exceptions are islands of memory, for a lucid interval between
the trauma and unconsciousness, or for a period of a fluctuating post-traumatic
confusional state.
e-true, the severity of the injury tends to correlate well with the duration of confusion
and with the extent of permanent retrograde and post-traumatic amnesia.
Q2:
Answer: e
a-true, uncommonly seen in those with short periods of unconsciousness.
b-true, because of selective vulnerability of the pyramidal neurons in" h1 sector of
Scholz".
c-true, and relative preservation of remote memory ,and thus the patient typically
appear to have an isolated disorder of short term memory.
d-true, and many patients exhibit a lack of concern about their environment and
confabulation may be very prominent in some.
e-false, a period of retrograde amnesia preceding the insult may occur.
Q3:
Answer: e
a-true, seen as an isolated syndrome.
b-true, like bibrachial paresis, cortical blindness ,or visual agnosia.
c-true, fortunately a rapid one, although the deficit may persist.
d-true, highly characteristic.
e-false, may show hypodense areas in the basal gangilia and the cerebellar dentate
nuclei.
Q4:.
Answer: b
a-true. Posterior cerebral artery supplies the medial temporal lobes, the thalami, the
occipital cortex and the upper midbrain.
b-false, the posterior cerebral artery comes form the vertebro-basilar system. The
amnestic syndrome results from bilateral damage to the inferio-medial temporal lobes.
i.e. to both hippocampi and adjacent structures like the dorso-medial thalami.
c-true, sometimes with alexia without agraphia, anomia, or sensory symptoms( like
peduncular hallucinosis).
d-true, like impaired papillary reflexes, vertical gaze palsy, or oculomotor nerve
lesions.
e-true, like many acute amnestic syndromes.
Q5:
Answer: b
a-true, or in old people with risk factors for atheroscleosis, especially a prior ischemic
event in the posterior cerebral arterial territory.
b-false, fortunately , only 10% will have recurrent attacks. TGA is a primary disorder
of short term memory that can last for minutes or days, typically for hours. The
patient appears agitated and perplexed and may repeatedly inquire about their
whereabouts, the time, and the nature of what they are experiencing.
c-true, as are remote memories and registration.
d-true, but usually gradually shrinks.
e-true, thus accounting for the patient's repetitive questions .
Q6:
Answer: c
a-true, and some may have a prominent agitation.
b-true, the patient's obvious concern about the condition distinguishes TGA from
most other organically based amnestic syndromes, but unfortunately may give rise to
a suspicion that this amnesia is psychogenic.
c-false, many patients are having risk factors for atherosclerotic disease and there may
be a history of a prior brain ischemic event.
d-true, better seen by MRI.
e-true, these are compatible with cerebral edema and may be related to spreading
depression (a wave of cellular depolarization accompanied by cellular swelling in the
brain).
Q7:
Answer: e
a-true, or by non-alcoholic individuals.
b-true, these disorders should be looked for and excluded at all possibility.
c-true, with relative preservation of remote memory and immediate recall.
d-true, especially serotonin or glutamine-mediated .
e-false, the disorder is self-limiting and no specific treatment is required, but
reduction of ethanol intake should be counseled, and thiamin should be given to treat
a possible Wernicke's encephalopathy.
Q8:
Answer: d
a-true, and in such patients, a prior psychiatric history, additional psychiatric
symptoms, or a precipitating emotional stress can often be identified.
b-true, or selective to some but not OTHER events during such a period.
c-true, an exceedingly RARE finding in organic amnesias. Despite such disorientation
to person, orientation to place and time may well be preserved.
d-false, recent memory may be LESS affected than remote memories- the REVERSE
of the pattern customarily seen in organic amnesias.
e-true, or examination after giving amobarbital sodium.
NB: Less frequent patterns of dissociative amnesia:
- Systematized amnesia: restricted to certain categories of information.
- Continuous amnesia: for events for some time in the past up to and including
the present.
- Generalized amnesia: for everything!!!
Q9:
Answer: e
a-true, but such a history is absent in up to 25% of patient and thus the diagnosis may
be challenging. Most patients are chronic heavy alcoholics, but also may be seen in
malnutrition and famines.
b-true, but how thiamin deficiency produces this effect?! It is still not settled.
c-true, and residual signs of a preceding Wernick'es encephalopathy like a horizontal
nystagmus or a mild cerebellar gait ataxia might be seen.
d-true, and long term memory is frequently affected as well but registration is intact.
e-false, the patient is classically apathetic and lacks insight into his illness. They may
attempts to reassure the physician that no impairment exists and try to explain away
their obvious inability to remember. CONFABULATION IS OFTEN BUT NOT
ALWAYS PRESENT.
NB: All patients should receive thiamin to prevent progression of the deficits,
although existing deficits are unlikely to be reversed.

Q10:
Answer: b
a-true, seen in those who recover. Often there is a total amnesia for the period of
encephalitis.
b-false, confabulation may occur and the whole syndrome may exactly resemble that
of alcoholic Korsakoff's.
c-true, like docility, indifference, flat affect and mood, inappropriate jocularity, sexual
illusions, hyperphagia, impotence, repetitive stereotyped motor activities, and the
absence of goal-oriented activities.
d-true, with or without secondary generalization.
e-true.
Q11:
Answer: a
a-false, it is a rare type.
b-true, or compressing its floor or walls from without.
c-true, may pose a diagnostic challenge.
d-true, and endocrine disturbances, visual field defects and papilloedema.
e-true, may include surgery, irradiation, or both.
Q12:
Answer: d
a-true, and presents as an amnestic syndrome that may precede the full blown picture
of the original lung tumor.
b-true, Anti-Hu antibodies may be detected up to 6% of patients in the serum and
CSF.
c-true, these changes preferentially affect the gray matter of the hippocampus,
cingulum, piriform cortex, inferior frontal lobes , insula, and amygdale.
d-false, they are VERY COMMOMN EARLY features.
e-true, and in many situations it progresses to a global dementia.
NB:
Complex partial seizures with or without secondary generalization may occur.
Depending on the underlying tumor and their auto-antibodies, other parts of the
nervous system may be involved as well and thus many other signs are seen, like
pyramidal, cerebellaretc.
Q13:
Answer: c
a-true, but non-specific findings.
b-true, but non-diagnostic.
c-false, in the medial temporal lobes.
d-true, anti Hu antibodies are the commonest and are usually associated with small
cell lung cancer, and anti Ta antibodies in testicular cancers ; in both the prognosis is
poor.
e-true, or it can progress or can remit.
NB: Korsakoff's syndrome should always be excluded, because cancer patients
are prone for malnutrition and the resulting thiamin deficiency is easily
treatable.
Chapter XI / Neurology
Subchapter D:
Headache is a very common complaint, yet it is totally non specific. Every day, we
see many patients in the neurology headache clinic (either referred from other doctors
or through a selfreferral), and the majorities of those patients are thinking that they
are suffering from a neurological illness (mainly a brain TUMOR). Proper history
taking and an efficient examination (not only neurological, but also a general medical
one ) will provide a clue or clues to the underlying CAUSE of this "HEAD PAIN" in
the majority of cases, at least will provide a crude reassuring evidence that we are not
dealing a sinister pathology. Psychological support of the patient and good
explanation about "what is going on" is very important in the management,. e.g many
patients with tension type headache do not know what is "tension" headache and thus
they change the physician repeatedly and non-fruitfully in an attempt to exclude a"
brain tumor". In this chapter I will try to cover this VERY IMPORTANT and
COMMON yet UNDER-ESTIMATED and BADLY MANAGED COMPLAINT.
Few Notes to be remembered:
1-Headache is caused by: traction, displacement, inflammation, vascular spasm, or
distension of the pain sensitive structures in the head and neck.
2-Isolated involvement of the boney skull, most of the dura, or most regions of the
brain parenchyma does not produce pain.
3-Pain sensitive structures within the cranial vault; these include:
a- Venous sinuses ( like the superior sagittal sinus).
b- Anterior and middle cerebral arteries.
c- Dura at the base of the skull.
d- Trigeminal, glossopharyngeal, and vagus nerves.
e- Proximal portion of the internal carotid artery and its branches near the circle of
Willis.
f- Brainstem periaquiductal gray matter.
g- Sensory nuclei of the thalamus.
4- Extracranial pain-sensitive structures; these include:
a- Periosteum of the skull.
b- Skin, subcutaneous tissues, muscles and arteries.
c- Neck muscles.
d-2
nd
and 3
rd
cranial nerves.
e- Eyes, ears, paranasal sinuses, teeth, oropharynx, and mucous membrane of the
nasal cavity.
5- Certain associated symptoms with headache:
a- Recent weight loss: think of cancer, giant cell arteritis, depression.
b- Fever and or chills: think of systemic infections ( bacterial , viraletc.) and
meningitis.
c- Dyspnea: think of infective endocarditis causing a brain abscess, pulmonary
encephalopathy.
d- Visual disturbances: think of ocular disorders (acute glaucoma, acute iritis),
migraine, or intracranial process affecting the optic nerve or visual pathways.
f- Nausea and vomiting: think of migraine, post-traumatic headache, intracranial
mass lesions.
g- Phtophobia: think of meningitis, subarachnoid hemorrhage, and migraine.
f- Myalgia: think of: viral syndromes, tension headache, giant cell arteritis.
g- Ipsilateral rhinorrhoea and lacrimation: typical of cluster headache.
h- Transient loss of consciousness: may accompany migraine and glossopharyngeal
neuralgia.
6- Physical signs detected when examining a patient complaining of headache:
a- Fever: think of systemic infections( viral, typhoid ..etc), CNS infections
(meningitis, brain abscess..etc).
b- Tachycardia: think of anxious patient with tension headache, systemic infections,
severe headache per se, Pheochromocytoma( ? episodic, BP, sweating ..etc), mass
lesion with early raised intracranial pressure, medications ( like vasodilators with
reflex tachycardia).
c- Blood pressure, hypertensive patient: think of an acute elevation in blood pressure
(?pheochromocytoma) or very blood pressure( ie malignant hypertension or
hypertensive encephalopathy). Hypertension per se is a risk factor for strokes
(? ICH, SAH, ischemic) but chronic uncomplicated hypertension PER SE is RARE
cause of chronic headache syndromes.
d- Hypoventilation: look for cyanosis and pulmonary encephalopathy ( ? examine
the optic disks for papilloedema).
e-Weight loss: Cachexia; think of cancer, polymyalgia rheumatica, giant cell
arteritis, chronic infections.
f- Skin: local infection of skin of the head and neck ( ?causing brain abscess,
cavernous sinus thrombosis), vasculitic lesions( ? systemic necrotizing vasculitides,
infective endocarditis, cancer associated, skin stigmata of phakomatosis (
neurofibromatosis cauing brain tumors, nevus flamus of Sturge-Weber's and
intracranial AVMs, Shagreen patches in tuberous sclerosis with intracranial
gliomas).
g- Scalp, face, and head: scalp tenderness; think of migraine, giant cell arteritis,
subdural hematoma, post-herpetic neuralgia, head trauma. Skull boney tenderness;
think of multiple myoloma, metastatic cancer, head trauma, Paget's disease. Bruit
over the orbit or skull; think of intracranial AVMs, carotico-cavernous fistulae,
aneurysm, meningioma. Tongue laceration; think of post-ictal headache.
h- Neck: Cervical muscle spasm; think of tension headache, cervical spondylosis,
spine injuries, migraine headache, cervical vasculitides, meningitis.
Q1-
Answer: e
a-true, ruptured Berry's aneurysm is responsible for up to 75% of cases.
b-true, with an equal sex incidence in general.
c-true, and ruptured AVMs are mainly seen in the 2dn to 4
th
decades of life.
d-true, but acute elevation of blood pressure (eg at orgasm) may be responsible for
their rupture.
e-false, responsible for only 10% of cases, with somewhat male preponderance.
Q2:
Answer: b
a-true, they are not TRULY congenital, and found in up to 2% of autotopsy series in
previously healthy people.
b-false, they are multiple in 20% of cases. The aneurysms are mainly located at the
major side branches at the circle of Willis.
c-true, looking for adult polycystic kidneys or aortic coarctation.
d-true, sometimes very large ones with multiple feeding vessels are found.
e-true, "mycotic" aneurysms in bacterial endocarditis; these are responsible for 2-3 of
ruptures and are mainly located at the distal middle cerebral arterial territory.
Q3:
Answer: d
a-true, and distorts the pain sensitive structures causing severe headache.
b-true, and acutely reduce the cerebral blood flow, and this together with "convulsive"
effect of the rupture, are thought to be responsible for the sudden loss of
consciousness at onset which seen in at least 50% of patients.
c-true, sub-hyaloid preretinal hemorrhages are seen in up to 20% of cases.
d-false, the subachnoid bleed is located outside the brain parynchyma. Focal cerebral
signs are UNCOMMON at onset , exception are : massive hemorrhage with extension
into the underlying brain parenchyma, large aneurysms in the middle cerebral artery,
and ruptured AVMs.
Q4:
Answer: c
a-true, it is classical but not invariable.
b-true, total absence of headache is against the diagnosis. The headache needs not be
to severe, eg mild headache is usually seen in ruptured AVMs, but should be present.
c-false, at least is seen in 50% of cases at onset. Vomiting and neck stiffness are also
common .
d-true, and during either at rest or exertion.
e-true, sentinel headache due to mild leaking or aneurysmal stretch.
NB: The most significant feature of the headache is that it is NEW, milder but
otherwise similar headaches may have occurred in the weeks prior to the acute event.
These prior headaches are probably the result of small prodromal sentinel bleeds or
aneurysmal stretch.
Q5:
Answer: c
a-true, and usually subsides slowly over the next 2 weeks.
b-true, and doubles the mortality figure.
c-false, the blood pressure rises very rapidly and may reach a very high level and a
hypertensive encephalopathy may a be diagnosed instead.
d-true, and together with signs of meningeal irritation , a diagnosis of pyogenic
meningitis is usually made.
e-true, but are usually absent in the first few hours of the ictus.
NB: Bilateral upgoing toes and VI cranial nerve palsy are common , BUT these do not
bear any relationship to the site of the ruptured aneurysm. Acute oculomotor nerve
palsy is an uncommon finding in posterior communicating artery aneurysms and
localizes the ruptured site. Ruptured AVMs also may localize the ruptured site by
giving focal cerebral hemispheric signs.
Q6:
Answer: c
a-true, will be positive in 90% of cases especially in patients with impaired
consciousness . It may show a subarachnoid bleed, intra-parenchymal extension,
intra-venticular blood or hydrocephalus, and even cerebral infarction. It is rapid and
available at many hospitals and suitable for unstable patients. However, it may not
show the aneurysm itself unless it is large enough.
b-true, these can not be seen by CT scans.
c-false, the sensitivity of the brain CT scans falls gradually day by day after the ictus (
from 90% in the first day down to less than 40% after 5 days post ictus.
d-true, also it is time consuming , not available at all centers, and has many
contraindications.
e-true, rises from less than 50% whithin the first 3 days to more than 80% 5 days post
ictus.
NB: If the initial CT scan is normal and the clinical index is high, the next step should
be lumbar puncture and CSF analysis.
Q7:
Answer: c
a-true, and usually exceeds the upper limit of the standard CSF manometers .ie above
600 mm.
b-true, and may be very difficult to differentiate it from traumatic taps in certain cases
and contains from 100000 to more than 1000000 RBC/ mm3.
c-false, it becomes xanthochromic after 6-12 hours due to break down of RBCs and
hemoglobin.
d-true, but in the same proportion to red cells as in the peripheral blood.
e-true.
NB: Blood is highly irritant to the leptomeninges, and a "chemical" meningitis may be
seen; the while cell count may be very high in the first 48 hours, and the glucose
becomes low 4 to 8 days post ictus. In the absence of such a pleocytosis, the CSF
glucose should be NORMAL.

Q8:
Answer: c
a-true, like peaked or deeply inverted T wave, short PR interval, or a tall U wave. The
cardiac troponins may be raised as well. These occur due to a massive catecholamine
release which may damage the myocardium.
b-true, to visualize the whole cerebral vasculature. The aneurysm is multiple in 20%
of cases, and AVMs may have multiple feeding vessels. Angiography can be
performed at the earliest time convenient for the radiology department personnel
(emergency angiography at the middle of the night is rarely indicated).
c-false, angiography is a prerequisite to the rational planning of surgical treatment and
is therefore not necessary for patients who are not surgical candidates eg those who
are deeply comatose.
d-true, causes are: the aneurysm may be sealed off by a clot, the aneurysm may be
very small, bleeding from a venous angioma, bleeding from a cavernous angioma, and
bleeding from a spinal source.
e-true, hence it is used mainly in screening purposes.
Q9:
Answer: d
a-true, it is seen in 20% of cases, over 10-14 days. Recurrent bleeding from AVMs is
less common in the acute period.
b-true, but rupture of a large aneurysm of then anterior or ,idle cerebral arteries may
direct the jet of blood into the brain parenchyma producing hemiparesis, aphasia, and
even transtentorial herniation.
c-true, these may develop with the first day to several weeks after the ictus, due to
impairment in CSF absorption.
d-false, these are rare and may mistaken for seizures. Seizures occur in 10% of cases
and ONLY after damage to the underlying cerebral cortex.
e-true, but all of them are uncommon.
Q10:
Answer: d
a-true, remote arteries may be affected but this is uncommon and usually not that
significant.
b-true, and peaks at day 10-14 post ictus.
c-true, the only ways to diagnose it.
d-false, it is closely related to the amount of the subarachnoid blood and thus is less
common where less blood is seen eg traumatic SAH or SAH following AVM rupture.
e-true, much more common than re-bleeding.
Q10:
Answer: d
a-true, this includes: bed rest, mild sedation, elevation of head of bed (15-20 degrees)
and analgecis.
b-true , as well as heparin.
c-true, to ensure adequate cerebral perfusion, and intravenous fluids should be used
with caution to avoid over-hydration.
d-false , hyponatremia should be treated by oral NaCl or iv 3% saline infusion rather
than fluid restriction. It may be due to cerebral salt wasting rather than SIADH.
e-true, and pharmacological intervention should be used if these fail. The objective is
to decrease the blood pressure to around 160/100 mmHg. High blood pressure
portends both an increased mortality and increased risk of re-bleeding and seizures.
.
Q11:
Answer: d
a-true, given as 60 mg, 6 times daily, for 21 days.
b-true, but this intervention is more safely performed after definitive surgical
treatment of the ruptured aneurysm.
c-true.
d-false, anticonvulsants should be given prophylactically ( e.g. pheytoin 300 mg /day)
because seizures increase the risk of re-bleeding.
e-true, to avoid brain edema.
Q12:
Answer: e
a-true, clinical grade I, II, and III. In those patients, surgery has been shown to
improve the clinical outcome.
b-true, but current evidence support an early intervention within 2 days post ictus.
This approach reduces the period at risk for re-bleeding and permits aggressive
treatment vasospasm with volume expansion and pharmacologic elevation of blood
pressure.
Q13:
Answer: e
a-true, overall , about 65% of patients will die!!!
b-true, and 25% of patients will die subsequently because of the initial hemorrhage
and its complications, and a further 20% will die of re-bleeding if the aneurysm was
not surgically treated.
c-true.
d-true.
e-false ,up to 50%.
Q14:
Answer: e
a-true, and relief with recumbency.
b-true, and comes in 1-2 days after LP and disappears within 1-2 days thereafter.
c-true, with traction upon pain sensitive structures at the base of the brain.
d-true , e.g. 22 gauge or smaller, and removing only as much as fluid needed for the
studies to be performed.
e-false , although it is self limited in the majority of cases, it responds well to caffien
sodium benzoate 500 mg infusion which can be repeated after 45 minutes if headache
persists or recurs upon standing. In resistant cases, the subarachnoid rent can be
sealed by injection of autologous blood into the epidural space at the site of the
puncture, this requires and experienced anesthesiologist.
NB: MRI in these cases will show marked enhancement of the pachymeninges at the
base of the brain and a "sagging brain" on sagittal sections.
Q15:
Answer: e
a-true, for unknown reasons.
b-true , or it may be severe and SAH-like in nature.
c-true , or it may be a post-LP like in nature ie mild dull occipital and increased by
upright position and relieved by recumbency.
d-true , all but the severe types are benign. The severe ones must be differentiated
from SAH.
e-false, marked increase in the systemic blood pressure.

Q16:
Answer: d
a-true, causing headache, diplopia and papilloedema.
b-true, due unilateral or bilateral abducens palsy. Although facial palsy may be seen
in paediatric age group( which rare) other cranial nerves involvement is against the
idiopathic variety.
c-true, however , the commonest is the idiopathic variety, next is an underlying
cerebral venous sinus thrombosis.
d-FALSE, there is what called headache-free pseudotumor cerebri. So the absence of
headache is not against the diagnosis.
e-true, and in the idiopathic variety, young woman are the usual victims with a peak
incidence in the 3
rd
decade.
Q17:
Answer: e
a-true, but be aware of headache free type ( we diagnosed a young male patient few
days ago as having this type of psuedotumor cerebri).
b-true, they are very common. Abducens nerve palsy could be uni- or bilateral.
c-true, but moderate to severe papilleodema is seen in up to 90% of cases.
d-true, the patient typically describes " a man standing in the fog" picture. Visual
obscurations are also seen.
e-FALSE, it is seen in all varieties, and hence the old term "BENIGN" intracranial
hypertension is no longer used.
Q18:
Answer: c
a-true, with no sequelae if the raised intracranial pressure was maintained in the
normal range to PREVENT secondary optic atrophy.
b-true, the most important differential diagnosis, thus it was termed "Pseudotumor
Cerebri" .Brain MRI or CT scans should be done.
c-false, typically shows slit-like ventricles and "bulging" eye globes. MRI and MRV
may also reveal a sinus thrombosis with or without venous infarctions.
d-true, any abnormal other parameter excludes the idiopathic variety.eg in septic sinus
thrombosis the CSF parameters will be abnormal like raised protein, pleocytosis..etc.
e-true, eg it might be 500 mm in the morning and 200 in the evening , but
characteristically never falls below the upper limit of normal range UNLESS the
patient is taking a treatment for it.
Tip : before few days we saw a 27 year old multiparous female with a full blown
picture yet the opening pressure was 90mm and never exceeded the normal
range, careful history taking revealed that the patient was taking Diamox in the
last 2 weeks which was given blindly by an ophthalmologist.
Q19:
Answer: e
a-true, staring with 250mg Diamox tablets 3 times daily up to 1500 mg/ day.
Furosamide can be aded as 40 mg twice daily .
b-true, very effective in these cases, given as 60-80 mg / day, and follow up the
patient.
c-true, to produce a "dural LEAK" area for the CSF to escape into the surrounding
regions.
d-true, but unfortunately shunt complications are common.
e-FALSE, unilateral procedures had been shown to protect BOTH eyes .
Q20:
Answer: d
a-true, if the patient was young ,or the pain is bilateral or alternating ,or associated
with brain stem syndromes always think of multiple sclerosis.
b-true, but in general the cause is still unknown.
c-true, isolated involvement of the 1
st
division or bilateral affection in the idiopathic
variety is very RARE.
d-FALSE, fortunately, occurrence during sleep is RARE. Prominent occurrence
during sleep may be a clue towards cluster headache.
e-true, but long term spontaneous remission is very RARE.
Q21:
Answer: c
a-true, any abnormal physical signs are against the diagnosis of the idiopathic variety,
eg may be due to multiple sclerosis or brain stem tumors. The trigger zones lie about
the cheek, nose , or mouth and the pain is "triggered" by stimulating these areas with
touch , cold application, chewing ,wind ,laughing, and teeth brushing.
b-true, a microvascular compressing "loop" is too small to be seen IF present.
c-FALSE, should always be NORMAL; a microvascular compressing "loop" is too
small to be seen IF present.
d-true, and produces an excellent response within 24 hours and may be the diagnostic
approach. If the response is not that good ,we may add pheytoin or lamotrigine.
Gasserian trigeminal ganglion ablation is NO longer used (high risk of keratopathy
and ulceration).
e-true, but because of the excellent response to carbamazepin this procedure is rarely
used in clinical practice.
Q22:
Answer: d
a-true, in general it is rare in clinical practice.
b-true, ie paroxysmal lancination, or simply it may be a continuous burning or aching
pain or discomfort.
c-true, and that's why it is triggered by swallowing ( note: trigeminal neuralgia is
triggered by chewing ).
d-false, the glossopharyngeal nerve is part of the neurogenic reflex bradycardia
pathways, so during sever pain episodes severe bradyarrhythmias may occur and thus
causing syncope( this is not seen in trigeminal neuralgia).
e-true, and somewhat younger age group is affected ( totally unlike trigeminal
neuralgia).
NB: the treatment is similar to trigeminal neuralgia ( see above).
Tip: Did you notice the differences between trigeminal neuralgia and
glossopharyngeal neuralgia!!?? Review the above questions well.
Q23:
Answer: d
a-true, the incidence increases with advancing age ( 70% above the age of 70 ), and
with thepresence of underlying immune suppression (like AIDS ,steroid usageetc)
,or malignancy( the classical one is Hodgkin's disease).
b-true, it is uncommon for the pain to persist for more than a year.
c-true, a useful clue indicating a previous attack of herpes zoster.
d-FALSE, the first division is the commonest to be affected and that's why the main
presentation is FOREHEAD headache.
e-true, together with residual scars.
Q24:
Answer: a
a-false, the 1
st
division of the trigeminal nerve when involved ,may cause a lost
corneal reflex, neuropathic corneal ulcerations, and visual loss. Remember that
impaired vision may be due to a previously associated zoster ophthalmitis including:
anterior uveitis, chorioditis, optic neuritis.
b-true, loss of corneal sensation.
c-true, with hypo-pigmentation .
d-true, and detected only by carful testing.
e-true, due to many reasons mentioned above.
Q24:
Answer: d
a-true, Aciclovir has been shown to reduce the intensity and duration of the zoster
rash and associated acute pain but NO effect at reducing the incidence of post herpetic
neuralgia. Prednisolone has been shown to reduce the intensity and incidence of the
acute pain but NO effect on post herpetic neuralgia.
b-true, but usually large dosed are needed ( up to 250 mg/ day ). Carbamazepin ,
phenytoin , and gabapentin are less effective.
c-true , or a xylocain-prilocain cream.
d-FALSE, may be very effective by depleting the pain-mediating peptides from the
peripheral nerve endings and sensory neurons, but unfortunately it is rarely tolerated.
e-true, at weekly intervals.
Q25:
Answer: d
a-true, for which no cause can be found.
b-true, and it is not paroxysmal.
c-true, and from intracranial extension of squamous cell carcinoma, or an infection at
the site of a tooth extraction.
d-false, treated by amitryptiline with or without phenelzine.
e-true, when tricyclics are not effective.
Q26:
Answer: d
a-true, at least in onset. It may be bilateral or alternating. Cluster headache is always
(100%) unilateral.
b-true, but not in all cases.
c-true, and photo and phnophobias and lassitude.
d-false, visual or other neurological abnormalities accompanying headache are seen in
up to 10%.
e-true, 75% of patients are women. A family history of migraine is seen in 50% of
cases( unlike cluster headache where a family history of a similar problem is not
seen).
Q27:
Answer: e
a-true, although a consistent mendelian pattern of inheritance has not been found
among the collective group of familial migraineurs.
b-true, and predicts a significant environmental contribution.
c-true, chromosome 19p13( associated with miss-sense mutation in a brain expressed
voltage gated P/Q calcium channel gene ) and 2 neighboring loci on chromosome 1q.
d-true, an possibly with multiple modes of inheritance and variable degrees of
penetrance.
e-false, migraine is considered a multi-genetic and multi-factorial disease.
Q28:
Answer: e
a-true, and the commonest are hemianpoic field defects,

Chapter XI / Neurology
Subchapter E:
Q1:
Answer: e
Apart from sedative drug intoxication, all other items cause confusional state+fever
(not hypothermia ) .Alcohol and sedative drug withdrawal cause fever but intoxication
with them is the cause of low body temperature Other causes of hypothermia with a
confusional state: hypothyroidism, hepatic encephalopathy, hypoglycemia.
Q2:
Answer: e
Bleeding peptic ulcer is the cause of hypotension (not hypertension). Dont forget
hypertensive encephalopathy and sedative drug withdrawal.
Q3:
Answer: c
Apart from opioid intoxication (which produces pinpoint reactive pupils), all others
can be the cause of her presentation.
Q4:
Answer: b
Hepatic encephalopathy (and prominent hyperglycemia) can be a cause of
hyperventilation not hypoventilation.
Q5:
Answer: e
Wernick's encephalopathy can cause confusional state with ophthalmoplegia and
ataxia (not a hemiparesis state) .Keep in mind that metabolic causes of
encephalopathy (like hypo or hyperglycemia, hepatic, uremicetc) can cause focal or
multifocal neurological signs that are usually fluctuating or ALTERNATING between
the right and left sides of the body.
Q6:
Answer: e
This question highlights the importance of any associated illnesses.
a-true, and hence EEG should be done which will reveal an ongoing seizure activity
in one of the temporal lobes .
b-true, always think of this possibility. A variety of intracranial hemorrhages might
occur and can precipitate a catastrophic status epilepticus.
c-true. A prolonged post-ictal confusional state is seen in the presence of underlying
metabolic or structural encephalopathy. Hence the recovery might be long giving a
wrong impression of a continued seizure activity (ie status epilepticus).
d-true , either iatrogenic (like adding an enzyme blocking medication like
Clarithromycin for a simple chest infection and enhancing phenytoin toxicity) ,or
might be self poisoning to suicide.
e-False..always take a complete history of any recent or OLD illnesses.
Q7:
Answer: c
a-or a sulphonylurease-induced hypoglycemia
b-or post traumatic epilepsy
c-false. Alcohol intake can cause: acute intoxication (with alcohol or other materials
like methyl alcohol), hypoglycemia, head injury, decompensated hepatic
encephalopathy, Wernick's encephalopathy, post-ictal state. Don't forget alcohol
withdrawal.
d-or may suggest an organic cause like hypothyroidism, Wilson's disease and
decompensated cirrhosis, B12 deficiency or a functional state. Don't forget
neuroleptic malignant syndrome in those taking conventional neuroleptics like
schizophrenic patients.
e-it may cause a variety of CNS and extra-CNS causes of confusional state.
Q8:
Answer: a
a-false, think of AIDS-dementia complex.
b-true, might be a cause of multi-infarct dementia
c-true, like bradycardia in hypothyroidism or atrial fibrillation in multi-infarct
dementia .
d-true, like hypothermia in hypothyroidism
e-true, might be due to brain tumor or subdural hematoma causing a dementing
illness.
Q9:
Answer: c
a-true, an acute confusional state which is mainly seen in non-chronic alcoholics.
b-true, although the severity and clinical features of encephalopathy correlate roughly
with blood ethanol levels. Chronic heavy alcoholics may have a very high blood level
although they don't appear to be intoxicated.
c-false, it is characteristically raised . The plasma osmolality roughly increases by 22
mOsm/L for every 100 mg/ dl of ethanol presents .
d-true. Can be differentiated by presence of ethanol odor, increased plasma osmolality
(in ethanol poisoning it is raised), blood and urinary toxicology.
e-true. Also predisposes to head injury, lung aspiration, seizures. Chronic alcoholism
increases the risk of bacterial meningitis.
Remember that the treatment is supportive only. All alcoholics should receive 100 mg
of thiamin intravenously to prevent Wernick's encephalopathy.
Q10:
Answer: d
Item d is false because confusion, IF PRESENT, is usually mild. Illusions and
hallucinations, usually visual, are seen up to 25% of cases. It usually responds to
diazepam 5-20 mg or chlordiazepoxide 20-25 mg orally every 4 hours.
Q11:
Answer: a
a-false. Usually seen within 48hours; however, in 70% of cases they occur within 7-
24 hours of abstinence.
b-true, and the interval between the first and last seizure is usually 6-12 hours up to
85% of cases . Up 40 % of patients will have ONE seizure only.
c-true. They abate spontaneously; however, diazepam or chlordiazepoxide is given
prophylactically because up to 30% of patients will develop delirium tremens.
d-unusual and atypical features are: focal fits, prolonged duration of the fits ( >6-12
hours ), more than 6 fits, status epilepticus or a prolonged post-ictal phase. In these
cases, a prompt search for pathology is required.
e-true. Always look for such an association.
Q12:
Answer: c
a-true. It is the most aggressive type with a high mortality rate, usually seen with 3-5
of abstinence and may last up to 72 hours.
b-true. Also tachycardia and sweating.
c-false. The mortality rate is 15% and mostly due to concomitant : infection ,
pancreatitis, cardiovascular collapse, or trauma .
d-true, the total requirement to produce a calm patient may exceed 100 mg/ HOUR.
e-true, like atenolol 50-100 mg/ day.
Q13:
Answer: d
a-true, with respiratory depression , hypotension, reactive pupils and hypothermia.
b-true, a characteristic feature. However, very large doses of phenobarbitone or
glutethimide may result in LARGE FIXED pupils.
c-true, with ataxia, dysarthria and hyporeflexia.
d-false. The mortality rate is low and mostly due to aspiration pneumonia (with or
without systemic sepsis, or due to iatrogenic fluid overload and pulmonary edema
).Despite severe intoxication, a patient who arrives at the hospital with adequate
cardio-pulmonary function and support should survive without any sequelae.
Remember the treatment is mainly supportive while the drug is being eliminated.
e-true, it is mainly used to increase the urinary clearance of Phenobarbital., but in
general should be avoided as it can lead to fluid overload. Hemodialysis may be used
in severe resistant cases of barbiturate poisoning or when drug elimination is impaied
by renal failure.
Q14:
Answer: d
a-true, intermediate or short acting agents are more likely to produce an withdrawal
syndrome when stopped abruptly. The syndrome is seen within 1-3 days (for short
acting agents ) and may take up to 1 week or even more to appear ( for long acting
agents ).
b-true, with confusion, agitation, seizures.
c-true, if positive , the patient should receive long acting phenobarbital orally to
maintain a calm state without signs of intoxication. In most patients it is possible to
stop it gradually after progressive decrement in the daily doses within 2 weeks.
d-false. Seizures especially the myoclonic ones should be treated aggressively with
anticonvulsants.
e-true, mainly seen in those taking very high frequent doses.
Q15:
Answer: d
a-true, as an iatrogenic overdose. Also seen as an accidental overdose in addicts, and
in suicidal attempts.
b-true, and pontine hemorrhage is a differential diagnosis .Although needle tracks and
marks might be seen, they are not diagnostic .
c-true, the test is positive if the pupils dilates and the patient regains his full
consciousness; however, when very large doses of opioids are taken or multiple drug
ingestion is present the pupils may slightly dilate.
d-false, with appropriate treatment , patients should recover uneventfully .
e-true, because nalaxone is a short acting agent.
Q16:
Answer: d
a-true, or with antidepressants and antihistamine overdose.
b-true, also flushing, urinary retention and tachycardia.
c-true, mainly used in antipsychotics or antidepressants overdose.
d-false. Symptoms usually resolve spontaneously.
e-true, although rarely needed . Physostigmine can produce severe bradycardia,
seizures and hypersalivation. Specific treatment is required when there is life
threatening cardiac dysrrythmias.
Q17:
Answer: d
a-true. Their mechanism of action involves a variable combination of inhibiting the
reuptake and or increasing the release of noradrenalin and or dopamine and thus
producing a central stimulant and peripheral sympathomimetic effects.
b-true, and cocaine can produce myocardial infarction.
c-true, either due to sudden severe hypertension, drug induced vasculitis, or rupture of
AVMs.
d-false, should be avoided , especially in cocaine induced myocardial infarctions.
Alpha blockers are useful to attack hypertension.
e-true, and thus attacking the psychotic manifestations of overdose. Because
amphetamines are longer acting than cocaine, amphetamine intoxication is more
likely to require treatment.
Q18:
Answer: c
a-true, and prominent insomnia.
b-true, changes in the mental status are usually the most striking feature. Alterations
in affect and mood may predominate the clinical picture.
c-FALSE, very rare. The presence of a prominent seizure activity should prompt a
search for another pathology or to revise the diagnosis.
d-true, there is prominent sympathetic overactivity.
e-true, and when this fails, treatment with diazepam may be of benefit.
Q19:
Answer: c
a-true, unlike other hallucinogens( eg LSD ).
b-true, also large or small pupils, horizontal and vertical nystagmus, hypertonia,
hyper-reflexia and myoclonus. There may analgesia to a surprising degree.
c-false, phenothiazines reduce seizure threshold and may produce severe hypotension.
Haloperidol can be used safely in such cases. Diazepam can be used for sedation and
treating muscle spasms.
d-true, although in some patients it may take days or even weeks.
e-true, this is especially seen in poisoning with large doses .

Q20:
Answer: e
a-true, whether accidental or intentional .
b-true, thus such patients may be wrongly diagnosed as having a serious CNS illness.
c-true, like renal or hepatic failures.
d-true. Dementia patients are very susceptible.
e-false, one of the prominent causes especially in old people.
Q21:
Answer: d
a-true, and also a "reversible" dementia.
b-true, bilateral ptosis is seen up to 65 % of cases due to low sympathetic tone of
levators.
c-true, also agitation and even frank psychosis is seen.
d-false, the most characteristic neurological finding is a delayed relaxation of tendon
reflexes, typically seen at the ankles.
e-true, and CSF pressure is occasional increased.
Q22:
Answer: e
a-true, so-called " activated crisis ".
b-true, so-called " apathetic crisis ".
c-true, because of the prominent hyper-adrenergic manifestations.
d-true, and exaggerated action tremor and hyper-reflexia are usually seen.
e-false, very rare and when prominent should cast a doubt on the diagnosis.
Q23:
Answer: e
a-true, especially in those with recurrent "hypos", or the presence of associated
autonomic neuropathy, or concomitant treatment with beta blockers.
b-true, flaccid quadriparesis is an advanced feature. Focal , multifocal or generized
seizures and myoclonus may be seen.
c-true, although a prolonged hypoglycemia at levels of 30 mg/ dl or lower invariably
leads to irreversible brain damage.
d-true, with coma, bilateral extensor planters and decorticate or decerebrate posturing.
e-false, somnolence might be seen. Tachycardia is seen with agitated delirium, but
bradycardia is seen with somnolence.
Q24:
Answer: c
a-true, while the degree of systemic acidosis does not.
b-true, focal neurological signs, focal or generalized seizures that are not responsive
to antiepileptics are commonly seen.
c-false, the mortality rate is unfortunately between 40-70% and is largely due to
failure to recognize the condition in elderly patients without prior history of diabetes
or who present who present with stroke or seizures, and to coexistent diseases.
d-true, in contrast to diabetic ketoacidosis.
e-true, impairment in consciousness ranges between very mild and subtle drowsiness
to deep and profound coma.
NB: hypersomolar non-ketotic hyperglycemia is a presenting feature of up to 40% of
cases of type II diabetes mellitus.
Q25:
Answer: d
a-true, and hyponatremia may produce focal signs by unmasking preexisting
structural brain lesions such as infarcts.
b-true, "central pontine myelinolysis syndrome" in which no treatment is available,
thus prevention is very important by avoiding rapid correction of hyponatremia.
c-true, but the tendon reflexes are usually normal .
d-false, they are very rare ( unlike hypocalcemia in which seizures are very common
and might be the only presenting feature).
e-true, the overall clinical picture is due to calcium-induced increase in the
depolarization threshold of nerve and muscle with consequent under-excitability.
Q26:
Answer: e
a-true, thus, hypocalcemic patients demonstrate a positive Chvostek's sign and
Trousseau's sign both of which indicate a hyperexcitability state.
b-true, this is especially seen in children. Hypocalcemic fits can sometimes be very
resistant to anticonvulsants.
c-true, also basal ganglia calcification and Parkinsonian features.
d-true, or induced by Trousseau's sign.
e-false, chorea may be seen. Also, irritability, depression, hallucinations, frank
psychosis all might be seen.
NB: The following 3 questions highlight the importance of knowing what Wernicke's
encephalopathy is. Every few days, the A/E department consults us to see certain
patients with a variety of neurological findings, these patients then prove to have
Wernicke's, although they give thiamine BLINDLY and the patient usually improves,
we noticed that many junior and senior house officers lack much information about
Wernicke's. I tried to cover some useful aspects about this VERY COMMON
neurological disease (we see it almost every day in the A/E department). You should
also know the prognosis if it.
Q27:
Answer: c
a-true, there is also small blood vessel proliferation and small petechial hemorrhages.
The most commonly affected areas are: mammilary bodies, periaqueductal grey
matter, cerebellar vermis, and occulomotor, abducens and vertibular nuclei. How
thiamine deficiency produces these effects is still not clear.
b-true, the commonest ocular manifestations are nystagmus and unilateral or bilateral
lateral rectus weakness or paralysis.
c-FALSE, ataxia primarily affects GAIT, limb ataxia is highly uncommon, as is
dysarthria.
d-true, up to 80%; remember alcoholism can attack many targets ( some are
asymptomatic).
e-true, it is uncommon cause of coma, but a very common cause an acute confusion.
Mental status examination reveals GLOBAL confusion with a prominent impairment
of immediate recall and recent memory.
NB: CSF analysis is usually NORMAL, although mild increase in protein (<90 mg/dl)
might be seen. An increased opening pressure or pleocytosis or low glucose should
prompt a search for other or additional disease.
Q28:
Answer: d
Wernicke's encephalopathy can have many ocular findings which can easily escape
detection. Medicine books usually mention " ophthalmoplegia and nystagnmus", but
there is no further details regarding these findings.
There are 9 ocular findings in general:
1-External rectus weakness or paralysis ( uni- or bilateral ).
2-Nystagmus, horizontal or a combined horizontal and vertical one.
3-Internuclear ophthalmoplegia, a common finding!
4-Conjugate gaze palsy or weakness.
5-Ptopsis.
6-Retinal hemorrhages.
7-Optic neuropathy.
8-Involvement of vestibular focusing mechanisms.
9-Small miotic reactive pupils ( a very rare finding but worthy to mention). Very
subtle anisocoria and sluggish pupillary reaction might be seen. However, the pupils
are usually spared.
Q29:
Answer: e
a-true, a very important thing tot remember.
b-true, together with confusion , should start to improve within 1 week.
c-true, they totally disappear in 40% only, others (60%) will be left with residual gait
ataxia and horizontal nystagmus.
d-true, up to 75 % of cases after recovery from Wernicke's, thus long term follow ups
are important.
e-FALSE, external ophthalmolegia, VERETICAL nystagmus and confusion
SHOULD BE ENTIRELY REVERSIBLE. Failure of these to reverse back to normal
should prompt a search for other or additional disease.
Q30:
Answer: c
a-true, can produce confusion, depression ,aggression, agitation, frank psychosis with
hallucinations.
b-true, but clear cut sensory LEVEL is against the diagnosis. Also, Lhermitte 's sign,
distal paresthesias, gait ataxia, all might be the presenting feature.
c-FALSE, loss of ankle jerk indicates peripheral neuropathy. Remember both might
present together and can cause a combination of exaggerated knee jerks, up going
planters with loss of ankle jerks.
d-true, any other abnormal profile should cast a doubt on the diagnosis.
e-true, vitamin b12 should assessed in the differential diagnosis of any unexplained:
cognitive impairment , myelopahty, and peripheral neurpatrhy, whether anemia is
present or not.
NB: Neurological abnormalities present for more than a year are less likely to correct
with treatment. Encephalopathy may begin to clear within 24 hoursafter the first
vitamin B12 injection, but full neurological recovery, when it occurs, may take
several months.

Q31:
Answer: e
a-true, that might gradually progress to stupor and coma.
b-true, as in other organ failure, asterixis ( which is actually a negative myoclonus)
indicates impairment of the parietal postural control mechanism.
c-true, and focal or multi-focal neurological signs that might fluctuate in severity and
type might occur.
d-true, it is the most SPECIFIC CSF finding. Also, an elevated opening pressure, mild
pleocytosis, and increased protein all might be seen Xanthochromia is seen when the
total serum bilirubin exceeds 4-6 mg/dl.
d-false, as in any metabolic encephalopathy, the EEG shows diffuse slowing with
triphasic complexes
NB: The prognosis in hepatic encephalopathy is most closely correlated with the
severity of hepatpocellular damage than neurological dysfunction.
Q32:
Answer: d
a-true. CSF acidosis is rare and cerebral edema is NOT a factor.
b-true, adding more confusion to the clinical picture. Motor manifestations are: coarse
tremor, asterixis, myoclonus and tetany. Focal or generalized seizures or focal signs
all are common.
c-true, and this may wrongly suggest an infectious meningitic process.
d-false, as in any severe encephalopathy ,and which may be diagnosed as seizures.
e-true, should always be kept in mind. The treatment should be directed against the
renal failure in its cause. Control of seizures and hypertension is very important.
Q33:
Answer: d
a-true, it may begin during hemodialysis or as long as 24 hours after hemodialysis. It
is a rare complication of maintenance hemodialysis in patients with chronic stable
renal failure.
b-true and a rapid correction of systemic acidosis may also exacerbate CSF acidosis
as CO2 diffuses into the CSF.
c-true, sometimes before any clinical change has occurred and showing paroxysmal
activities with spikes and sharp waves.
d-false, fortunately these severe abnormalities are uncommonly seen.
e-true, and prevention is much more important by correcting uremia more gradually
or using briefer periods of dialysis at reduced rate of blood flow.
Q34:
Answer: d
a-true, the confusional state ranges from mild drowsiness to deep coma.
b-true, unlike other encephalopathies complicating other organ failures.
c-true, hypereflexia is uncommon.
d-false, one of the commonest mistakes in clinical practice. Bicarbonate infusion
increases the production of CO2 which subsequently diffuses into the CSF and
worsens the CSF acidosis leading t paradoxical clinical deterioration while systemic
acidosis is improving.
e-true. Treatment involves intubation and mechanical ventilation.
Q35:
Answer: e
a-true, a very aggressive process.
b-true, and on the surface of the brain.
c-true, and their surface may show ependymal exudate or granular ependymitis
d-true, and thus focal signs like hemiparesis can be seen.
e-false, cranial nerve palsies are in general either due to vasculitis or nerve
compression due to basal fibrosis, both of which are very common.
Q36:
Answer: d
a-true, surprisingly such history is usually absent, thus the diagnosis needs a high
index of suspicion.
b-true, but ALL might be ABSENT.
c-true, and can wrongly be attributed to a mass lesion.
d-false, can be a prominent feature.
e-true, and thus increasing the already increased morbidity.
Q37:
Answer: c
a-true, but may show a clot upon standing due to high protein content.
b-true, this early polymorphonuclear pleocytosis may wrongly suggest a pyogenic
cause.
c-false, it may reach very high levels ,especially in those with spinal subarachnoid
block.
d-true, it was used in the past, but nowadays it has been shown that it is too non-
specific.
e-true, unlike pyogenic meningitis (where the CSF glucose might reach ZERO mg/dl
!).
Q38:
Answer: c
a-true, the inflammatory response is associated with the release of inflammatory
cytokines like IL-1, LL-6 and TNF alpha and thus promoting the permeability of the
blood brain barrier, vasogenic cerebral edema , changes in cerebral blood flow and
perhaps direct neuronal toxicity.
b-true, and more marked in the basal cisterns with N. meningitides.
c-FALSE, actual bacterial invasion of the underlying brain is very rare. The burden of
the pathological process is within the leptomeninges with secondary changes in the
underlying brain.
d- true, and thus many complications can alter the clinical course.
e-true, that is why meningitis is an aggressive disease.
Q39:
Answer: d
a-true, and few may presents as a stroke like pattern within 1 day .
b-true, it is usually absent in neonates and old people and in those with deep coma.
Thus its absence is NOT against the diagnosis.
c-true, and may be easily mistaken for a brain abscess.
d-FALSE, it is seen in 50-60% of cases of N. meningitidis infections.
e-true, but in clinical practice the full syndrome is often NOT present.
Q40:
Answer: b
a-true, leucopenia may indicate and immune suppressed state or an over-whelming
infections and hence portending a very bad prognosis.
b-FALSE, fortunately, it can be isolated in 40-90% of cases ( while CSF culture is
positive in 80% of cases only ! ).
c-true, indicating a wide spread inflammatory process. Granular ependymitis may be
seen.
d-true, and focal changes may indicate the presence of focal cerebritis, brain abscess
or scarring.
e-true, always look for a parameningeal focus of infection and remote sites of
infection like pulmonary abscess.
Q41:
Answer: d
a-true, and the CSF appearance ranges from slightly turbid to grossly purulent.
b-true, predominantly polymorphonuclear. The presence of predominantly
mononuclear pleocytosis is consistent with Listeria infections. CSF white cell count
more than 50000 cells/ ml almost always indicates the presence of a burst brain
abscess as cause.
c-true, and may be immeasurably low ie almost ZERO mg/ dl.
d-FALSE, it is positive in 60-80% of cases. In TB meningitis , acid fast smears are
positive in 20% of cases only.
e-true, a useful tool. CSF culture is positive in 80% of cases only.
Q42:
Answer: d
a-true, and is based upon the patient's age and predisposing factors.
b-true, although it is suggested to be given to children less than 2 months of age and
to adults with positive CSF gram stain and features of raised intracranial pressure.
Steroids has been shown to decrease hearing loss and neurological sequelae in
children with H. influenzae meningitis.
c-true, and the CSF pleocytosis and the proportion of polymorphonuclear cells should
decrease with 3 days.
d-FALSE, 20mg /kg once daily for 4 days. In N. meningitides meningitis it is given as
20mg/kg twice daily for 2 days. You should know these as they are totally different.
e-true, and a vaccine is also available for certain strains of N. meningitides and is
recommended for military recruits, college students and travelers to areas of ongoing
epidemics.
Q43:
Answer: d
a-true, case fatality rate may exceed 25%.
b-true, Waterhouse-Friderichsen syndrome.
c-true, as is a delay at initiating treatment.
d-false, BAD prognosis.
e-true.
Q44:
Answer: a
a-false, it is seen within 2 years of a primary syphilitic infections and affects men
more than women.
b-true, it is usually asymptomatic.
c-true. The cranial nerves most frequently affected in descending order are: facial,
acoustic, oculomotor, trigeminal, abducens and optic, but other cranial nerves may be
involved as well.
d-true.
e-true, even when asymptomatic to prevent the development of more serious CNS
complications.
Q45:
Answer: a
a-false, although CSF VDRL is positive in 50% of cases only, the blood FTA
treponemal tests are almost always positive.
b-true. These are not present in normal CSF.
c-true, and the opening pressure may be normal or slightly elevated.
d-true, although acute syphilitic meningitis is a self limiting illness with littile or no
swquelae, it is should be treated to prevent the future development of tertiary
neurosyphylis.
e-true, and another course of treatment should be given if the CSF cell count or
protein remains elevated.
Q46:
Answer: d
a-true, there is no seasonal variation. The mortality rate ranges between 40-70% and
depends on many factors.
b-true, the age group is higher than that of other causes of viral encephalitides in
general.
c-true, while HSV type II affects neonates when passing through an infected birth
canal. HSV type II in adults usually produces viral meningitis rather than viral
encephalitis.
d-FALSE, characteristically involves the medial temporal and inferior frontal lobes(
unilateral or bilateral affection) .
e-true, intranuclear inclusions may beseen in neurons and glia. Patients who recover
may shoe cystic necrosis of the involved areas.
Q47:
Answer: d
a-true, but does NOT reliably implicate HSV as a cause of encephalitis .
b-true, may cause coma with few days.
c-true, the diagnosis can be easily missed early and may be wrongly diagnosis as an
acute psychosis.
d-false. May be seen
e-true, reflecting the predilection of HSV for LIMBIC structures.
Q48:
Answer: c
a-true, CSF white cell count usually between 50-100 cell/ml.
b-true, it is one of the causes of hemorrhagic CSFs. The glucose is usually NORMAL.
c-FALSE, the virus generally can not be isolated form the CSF but the virus DNA can
be detected in the CSF by PCR in some cases.
d-true, and focal paroxysmal activity in the form of spikes and waves might be seen.
e-true, but usually they show abnormalities in one or both temporal lobes and the
changes may extend to frontal and parietal lobes.
NB; the CSF is totally normal in 2% of cases.
Q49:
Answer: e
a-true, it can complicate many systemic cancers, in descending order: ALL, non-
Hodgkin's lymphoma, malignant melanoma, AML, breast carcinoma , Hodgkin's
lymphoma , lung carcinoma, GIT carcinomas and sarcomas .
b-true, but may be the presenting manifestation or occur after many years of illness (
sometimes in apparently cured patients ).
c-true, with nausea , vomiting , seizures, and gait abnormalities.
d-true, indicating involvement at multiple levels of the neuraxis.
e-false, primary brain tumors may be associated with meningeal gliomatosis , and
medulloblastomas and pineal tumors have a particular propensity for meningeal
dissemination.

Q50:
Answer: b
a-true, and the cytology is positive in 55 % of cases in general.
b-false, large volumes of fluid and repeated CSF samples increase the diagnostic
yield. The detection of malignant cells by cytological examination is the objective.
c-true, beta2 microglobulin, beta glucuronidase and LDH isozyme-V are sensitive
markers.
d-true, should always be kept in mind.
e-true, and pleocytosis is present in 60% of cases .
NB: Untreated , neoplastic meningitis typically results in death within 2 months.
NB: In neoplastic meningitis: ophthalmopleagia(30%),facial weakness(25%),
papileodema (10%),
Q51:
Answer: e
a-true, with or without pre-existent hypertension.
b-true, lower levels may suffice to produce the picture in previously normo-tensive
people.
c-true, and thus renal function should always be investigated.
d-true, may present in stroke like pattern.
e-false, it develops over a period of several hours to days.
NB: pathophysiologically; there is diffuse cerebral vasospasm, impairment of cerebral
autoregulation mechanisms of blood flow and increased blood coagulability. The net
result is diffuse microinfarcts and petechial hemorrhages that are mainly seen in the
brain stem, and to a lesser extent in the subcortical grey and white matter regions.

Q52:
Answer: e
a-true, the most helpful and specific sign.
b-true, with or without papilloedema.
c-true, and elevated protein might also be seen.
d-true, indicating areas of brain edema and these changes are reversible upon
successful drug therapy.
e-false, renal failure appears to increase the risk of developing hypertensive
encephalopathy , thus blood urea and electrolytes should be estimated in all cases.
NB: hypertensive encephalopathy is a diagnosis of exclusion.
Q53:
Answer: d
a-true, without producing hypotension.
b-true, and thus the infusions of antihypertensives should be carefully monitored to
produce these targets.
c-true, indicating over treatment and hypotension.
d-false, it can given either by intravenous infusions or by repeated boluses.
e-true, but prompt treatment usually produces FULL clinical recovery.
Q54:
Answer: e
a-true, commonly seen between the age of 10 to 40 years.
b-true ,50% in average.
c-true, and also thrombocytopenia.
d-true, causing small microinfacrts and discrete hemorrhages.
e-false, true cerebral vasculits is very RARE.
Q55:
Answer: e
a-true, the cognitive dysfunction includes acute confusional state, schizophreniform
psychosis, depression and mania.
b-true, but may be focal as well.
c-true, also , diplopia, ptosis, hemiparesis, paraparesis and chorea all are less common
findings.
d-true, although anti ribosomal P protein is supposed to be associated with lupus
psychosis, but still controversial issue.
e-false, the EEG is usually diffusely slowed or there are focal abnormalities.
Q56:
Answer: d
a-true, but in practice, cerebral lupus is by far much more common.
b-true, also, infections, severe anemia, lupus endocarditis , metabolic and electrolyte
disturbances.
c-true, and if the patient is not receiving steroids, steroids should started at doses
between 60-80 mg/dl.
d-false, it has NOT shown to adversely affect the overall prognosis.
e-true.
Q57:
Answer: d
a-true, after ingestion of the larvae of the pork tapeworm.
b-true, obstruction to the CSF pathways by intraventricular cysts, or inflammation
causing basilar meningitis.
c-true, also myelopathy and vague personality changes.
d-false, typically shows lymphocytic pleocytosis with many esosinophils. The
opening pressure might be high due cerebral mass lesions or very low in cases of
spinal subsrachnoid block.
e-true, also may show intracerebral calcification or ventricular enlargement.
Q58:
Answer: e
a-true, however , patients with seizures and either meningitis or one or more
noncalcified cysts should be treated. Intraventicular, subarachnoid and racemose cysts
respond very poorly to treatment.
b-true, as these can produce obstructive hydrocephalus.
c-true, by thus removing a mass effect.
d-true, might be the only option.
e-false, calcified cysts DO NOT require treatment.
Q59:
Answer: c
a-true, thus prophylaxis is very important.
b-true, and causes occlusion of small cerebral blood vessels.
c-false, it usually produces a global confusional state and seizures. Focal signs are
RARE.
d-true, with raised protein and mononuclear pleocytosis.
e-true, 20-50% and may reach 80%in cases complicated by coma and seizures.
Q60:
Answer: e
a-true, also there may cerebral infarction related to cerebral vaculitis and vernticular
enlargement due to communicating hydrocephalus.
b-true, might be also seen with actinomycosis and aspirgillus infections.
c-true, may also resemble brain abscesses and neurosyphilis.
d-true, like systemic cancer, diabetes, HIV , immune suppression, organ
transplantationetc.
e-false, intravenous amphotericin B which should be used for 3 months.
Unfortunately the treatment is long and expensive.
Endof Neurology Chapter.
Chapter XII / Infectious diseases Answers
Q1:
Answer: 4
1- genralized lymphadenopathy may be seen.
2- but unfortunately needs long time in special culture media with special.
environment. Also it is hazardous to the lab workers, so it is NOT used in almost all
cases.
3- or a pseudotumor cerebri like picture.
4- false, very common.
5- true, especially in highly endemic areas, and 2 mercapto ethanol test is used in this
situation.
Q2:
Answer: 5
1-unlike other salmonella species
2-also palpable spleen and rose spots are seen in the 2
nd
week
3-but are highly suggestive when seen
4-about 90% positivity and then decreases there after
5-false, the main indication is in those who were exposed to antibiotics
Q3:
Answer: 5
1- should be done if malaria is suspected clinically.
2- and to count the parasitemia.
3- true, may cause hypoglycemia and cardiac toxicity and cinchonism.
4- and 3
rd
trimester of pregnancy.
5- false, are of no benefit at all, actually it should be avoided.

Q4:
Answer: 5
1- true, like sewage workers or farmers.
2- true, urine culture becomes gradually positive in the second phase.
3- true, and may cause significant morbidity and mortality.
4- true.
5- there is blood neutrophil leukocytosis which is very important in differentiating
leptospirosis from severe viral hepatitis infections.
Q5:
Answer: 4
1- other sources from skin abscess, conjunctivitis.
2- true, it is not a bacteremic infection; it is exotoxin mediated.
3- actually one of the diagnostic criteria.
4- false, up to 10%.
5- and usually necessitates the use of pressors.
Q6:
Answer: 5
1- usually through a bite of an infected animal.
2- the, the virus is present in the saliva, although it infects the CNS.
3- true, very few exceptions are documented world wide, yet they are left with severe
neurological disability.
4- and so-called the dumb type found in 20 % of cases only; the other 80% is called
the furious type with many characteristic phobias.
5- false, highly variable.
Q7:
Answer: 5
1- RNA one.
2- true after an incubation period of 1-3 weeks.
3- true, and circulatory collapse.
4- considered to be high
5- false, Ribavirin can be used and may decrease the mortality rate if given with in the
1
st
week.
Q8:
Answer: 4
1- from monkeys through mosquitoes.
2- 3-6 days.
3- leukopenia is seen.
4- true, but not a specific picture.
5- false, no specific treatment, only supportive.
Q9:
Answer: 3
Till now there is no vaccine!!!
Q10:
Answer: 4
1- The disease is mainly seen in farmers and vets.
2- but usually other features are seen also.
3- true, and its treatment is usually prolonged; may be for 1 year.
4- false, the drug of choice.
5- true, liver granulomas might be seen, but usually these are asymptomatic.
Q11:
Answer: 4
1- may be mite, flea, tick and louse born.
2- in the skin, brain, lung.
3- true.
4- false, no such a feature.
5- or chloramphenicole.
Q12:
Answer: 5
1- true, actually many patients are not able to recall it.
2- true.may disappear before being noticed by the patient.
3- may be bilateral.
4- Borrelia burgdoferi.
5- false, asymmetric large joint oligoarthritis.
Q13:
Answer: 3
1- but involves the palms and soles.
2- True, rarely severe.
3- False, FTA absorption and TPHA are much more sensitive and specific.
4- and condylomata lata are seen in the genital areas.
5- True.
Q14:
Answer: 5
In general it is milder and shorter illness when compared with S. typhi infection.
Q15:
Answer: 5
1- so-called cholera sicca, causing large fluid loss and sequestration in the bowel.
2- true, a common infecting source is usually there.
3- can be seen under the microscope (comma shaped).
4- true, it is not an invasive disease.
5- false, painless diarrhea!
Q16:
Answer: 2
1- lymphogranuloma venereum, Chlamydia trachomatis serovars L1, 2, 3.
2- primary, not secondary!
3- usually type 2 viruses.
4- chanchroids.
5- granuloma inguinale.
Q17:
Answer: 3
1- true, if seen in a female child always think of sexual abuse which his VERY
RECENT.
2- true, but may be mild.
3- false, can be cured by single dose of antibiotics like oral 500 mg ciprofloxacin once
only; but because Chlamydia infections may be co-associated, so we give tetracycline
for 10 days; always CONTACT SEXUAL PARTNERS.
4-true, the usually source of SPREADING of infection to males.
5-true, do you think that this infection makes them stop sexual contact?
Q18:
Answer: 3
Erythromycin is useful in the treatment of Legionares disease but not in the
prophylaxis; besides, there is no man to man transmission of infection.
Q19:
Answer: 5
1- like gp 160 and 45.
2- unlike the western population in which P carinii is much more common.
3- the progression can be slowed by AZT therapy.
4- self limiting in healthy persons.
5- false, usually asymptomatic, unlike the pulmonary one which portends a very
aggressive course.
Q20:
Answer: 5
1- similar to mitochondrial myopathy.
2- and many NRTIs can cause liver dysfunction and lactic acidosis and pancreatitis.
3- of many types of lipodystrophy.
4- and some may cause severe allergic reaction that needs drug stoppage for good.
5- nail pigmentation is characteristically seen with zidovudine.
Q21:
Answers: 3
Low hypoxemia ratio portends a poor outcome. Other predictors of a poor outcome:
delayed diagnosis, low CD4 positive count, and hypoalbuminemia.
Q22:
Answer 5:
Also they have a greater risk of developing military and disseminated TB, and of
developing a second episode of TB from exogenous infection as demonstrated by
isolate typing.
Q23:
Answer: 1
Option 1 is the typical established picture which is seen in 80% of cases; later an
ARDS like picture will be seen. Atypical appearances are seen in up to 20% of cases;
in addition to above, a nodular appearance is also seen.
Q24:
Answer: 4
1- prior to HAART, almost 25% of HIV patients with a CD4 positive count of less
than 100/mm3 developed this grave complication.
2- and some may be totally asymptomatic and discovered by fundoscopic
examination.
3- and from toxoplasma retinitis, progressive outer retinal necrosis syndrome,
syphilis, and Pneumocystis.
4- No recovery of vision occurs in the affected areas and there is always a risk of
retinal detachment because of retinal necrosis.
5- but is the most sight threatening.
Q25:
Answer: 5
1- it is a highly contagious zoonotic protozoal enteric pathogen.
2- and usually severe and persistent large volume painful water diarrhea.
3- and rarely can cause acalculous cholecystitis, sclerosing cholangitis, pneuomonitis.
4- or by using a modified acid fast satin on stool specimens in 90% of cases.
5- may respond to paramomycin and azithromycin.
Q26:
Answer: 4
1- and acquisition is probably through respiratory of GIT tract where colonization
occurs and precedes the disseminated disease in 2/3
rds
of cases.
2- but the reticulo-endothelial system bears the major burden of the infection.
3- and CT scans usually reveal an enlarged intra abdominal and mediastinal lymph
nodes.
4- false, together with lymphadenopathy are commonly seen clinically.
5- or positive cultures of a liver biopsy or a bone marrow aspirates.
Q27:
Answer: 2
1- the highest count produced of all helmintic infestations.
2- the microfilariae survive for 2-3 years, and the adult worms for 10-15 years in
general.
3- producing lymphatic filariasis.
4- transmitted by the day biting fly Chrysops.
5- the River Blindness, transmitted by a Simulium fly.
Q28:
Answer: 5
Ascarisis can cause eosinophlia. Other tropical infections not associated with
eosinophilia: Leprosy, TB, Tapeworms, Typhoid fever and brucellosis, ameabiasis
and ARBO viral infections.
Q29:
Answer: 4
Other causes: Hydatid disease, liver flukes, filariasis, and hook worms.
Q30:
Answer: e
Cysticercosis does not produce a malabsorption state.
Q31:
Answer: 4
Oriental sore is caused by Leishmania tropica. Other options are true. Please review
textbooks for furthere details of this topic.
Q32:
Answer: 5
Infections during the 1
st
month of solid organ transplantation generally are not due to
immune suppression; they are the infections seen commonly in post- operative
patients. Nocardia infections indicate a prominent immune suppression in those
patients and are seen between 2-6 months following solid organ transplantation.
Q33:
Answer: 5
Staph aureus infections are characteristically seen during the 1
st
4 weeks post
transplantation in the form of skin wound infection, catheter related infection, and
chest infection. During the 2
nd
to 6
th
months following solid organ transplantation the
infections are of those seen in immune com[promised patients: in addition to the
above; EBV, varicella zoster, papova viruses (J C and BK), adenoviruses, toxoplasma,
nocardia, and pneurmocystis carinii.

Q34:
Answer: 1
Fecal leukocytes indicate an invasive pathogen; cholera is not an invasive infection
and hence no fecal leurkocytes are found on stool examination. All other options are
true.
Q35:
Answer: 4
Chagoma is due to local replication of Trypanosoma cruzi at the site reduviid bug
bite.
Q36:
Answer: 5
Having a cat at home does not confer an increased risk of skin candidiasis; other
infections that might be seen: Pasteurellosis (usually a bite wound), Cat scratch
disease, Tularemia, hook worms, and rabies.
Q37:
Answer: 4
A tricky question. Causes of an acute false positive RPR test: Pregnancy, recent
immunization, febrile illnesses. Causes of chronic false positive RPR test: Chronic
infections, autoimmune diseases, and iv drug addicts. In false positive testing, the tire
is usually below 1:8.
Q38:
Answer: 5
The first 3 options are the cause of a false positive monosopt test; option 4 is the
cause of a truly positive test; option 5 has nothing to do with monospot tests.
Q39:
Answer: 4
Guillain Barre syndrome is a cause of acute inflammatory demyelinating
polyradiculopathy; so no increase in muscle tone is noted. Causes of lock jaw apart
from tetanus:
1- Any inflammatory lesion in the mouth, pharynx, cheeks, or external auditory
canal (like Ludwig's angina, peritonsillar abscess, tooth abscess).
2- Malignancies; sarcoma of the jaw (squamous cell carcinoma of the oral
cavity).
3- Conversion disorders; hysterical tetanus.
4- Mechanical problems; jaw dislocation, jaw ankylosis.
5- Phenothiazines; part of dystonic reactions.
6- Strychnine poisoning; a late sign.
7- Encephalitides.
Q40:
Answer: 1
The grove sign is due to adenopthy above and below the inguinal ligament; said to
pathognomonic to LGV in the appropriate clinical setting.

Chapter XIII / Immunology Answers
Q1:
Answer 4
1-truehence IgA deficiency may cause chronic or repeated sinu-plumonary and
GIT infections
2-agglutinating one
3-hence the passive neonatal immunity and false positive neonatal screening for many
infections (actually from the mother!!)
4-type I .a famous thing
5-yes.actually its function is still not that 100% ..Has a very low serum level.
Q2:
Answer 3
1-also certain IgG subclasses in MMin Waldenstrom's it is the IgM
2-truevery higher than asthma .useful clue when suspecting ABPA
complicating asthma
3-IgM not IgG
4-true.it is not that rare ..
5-true...Many dont know that
Q3:
Answer 5
1-or IgM. IgA activates the alternative pathway
2-and hence family tree is important in the male relatives
3-true..including recurrent meningitis
4-true.? may be due to inability to remove immune complexes
5-fasle 10% only is of qualitative defect. The majorities are of a quantitative defect
Q4:
Answer: 5
1-considered to be a primary not a secondary phenomenon
2-also, gold and penicillamine
3-and nephritic syndrome
4-also CLL and MM
5-false associated with hypergammmaglobulinemia mainly of IgG type
Q5:
Answer: 5
1-true..and the spleen also may be enlarged in 30% of cases
2-and IgM may be normal or low or even raised
3-true ..The T cells are usually functionally immature
4-and there is no specific HLA association
5-fasle increased risk of selective IgA deficiency
Q6:
Answer 5
Chronic active hepatitis is associated with hypergammaglobinemia is not a
complication of HYPOgammaglobinemia
Q7:
Answer: 5
1-hence no defective cell medicated immunity
2-true ..due to defective maturation in the bone marrow
3-true, usually below 50 mg / dl
4-usually with repeated chest infection
5-faslefortunately rare
Q8:
Answer: 4
1-truecheck their relatives for CVID!!
2-because of the resulting gut hypomotility may be an additional risk factor for
bacterial overgrowth.
3-trueas blood or blood product transfusion may cause severe anaphylactic
reactions
4-fasle.around 3-4% ..And may cause false negative IgA anti-endomycium
antibodies testing .hence we check both IgA and IgG there to overcome this
problem.
5-true.due to the disease per se or its treatment with gold and penicillamine
Q9:
Answer: 5
1-Depending on certain chemokine receptors and associated ligands
2-hence the hypergammaglobinemia
3-trueusually under estimated
4-true..many complicated mechanisms
5-false .it is CD4 positive and infected early in the course ..Hence the name
M (macrophage) strains of the virus
Q10:
Answer 5
1-unlike the widely distributed class I
2-truelike the blood ABO antigen
3-trueshort arm
4-if you dont know this .this is a disaster!!!
5-fasle..supposed to be protective against type I diabetes, unlike HLA DR3/4.
Chapter XIV / Psychiatry Answers
Q1:
Answer 5
Remember ,dementia patients are liable to be confused for many reasons, like fever
per se, infections, electrolyte disturbance, medication side effect..etc ..so the past
history is very important on the other hand e.g. chronic subdural hematoma
may have fluctuated level of consciousness and gradually ending with dementia
.so it is difficult task to differentiate actually in practice..EASILY written in
the books !!.
Q2:
Answer 3
1-and TSH, FSH and LH are reduced
2-cold extremities, lanugos hair remember in anorexia nervosa there is
starvation and stress.
3-false .against the diagnosis
4-and loss of libido also. The loss of weight must be more than 25% of the original
weight or the current weight is blow 25% of the normal for age and sex
5-truebulimia usually starts later
Q3:
Answer 4
1-flase ..Usually they maintain weight
2-typically post pubertal but later than anorexia nervosa peak
3-falserarely required
4-true, to be followed by self induced vomiting, diuretic or laxative abuseor the
patient may enter a phase of prolonged dieting after these recurrent binge eating
episodes
5-false.suggests anorexia nervosa
Q4:
Answer: 5
1-chronic insidious one has a bad prognosis
2-true .the presence of poor job records indicates poor prognosis
3-true..also absence of family history and catatonic variety has good prognosis
4-true..Schizoid one portends a bad prognosis
5-false.confers a bad prognosis
Q5:
Answer 4
1-the patient feels worthless, helpless and hopeless
2-others gain weight
3-anhedonia
4-false ..some patients it is very prominent so called agitated depression
.remember ..In severe depression, psychotic symptoms may be prominent and
must differentiated from schizophrenia with affective symptoms
5-other have late insomnia
Q6:
Answer 5
1-true..the older generation like chrolpromazine is more effective on the positive
symptoms
2-true.many doctors think that one of them might be superior to others
.they only differ in the cost , availability and side effect profile but as an
effect ,all are the same
3-true, those with weak anticholinergic side effects have more risk of extrapyramidal
features, eg trifluperazine
4-true.especially haloperidol (Haldol)
5-false, some tricyclic antidepressants may be used here.
Q7:
Answer 5
1-true.many new anti depressants may have varieties of mechanisms
2-true..and also old men may have occult prostatism or glaucoma, so be careful
3-very important in old people as it may increase the risk of fall
4-true.so be patient
5-false ..In these cases, electro-convulsive therapy should be used immediately
Q8:
Answer 5
1-and cardiomyopathy
2-always ask about daily alcohol intake, may worsen already present hypertension
3-may cause chronic pancreatitis
4-interfers with tubular uric acid excretion
5- Alcohol does not cause a specific skin rash per say ..but repeated unexplained
skin bruises may be a clue to repeated falls from intoxication
Q9:
Answer 1
1-although the risk of suicide per se is much more in females but the success in that
attempted suicide event is much more in males, especially old ones
2-true..and when writing a plan for it
3-or other chronic physical disability
4-true.many females use self poisoning as a parasuicide to have others
attention
5-also when separated from the spouse
Q10:
Answer 5
ECT usually given under general anesthesia twice or thrice / week for 3 weeks .some
patients may need maintenance ECT in the long term .ECT has many contraindication
like aortic aneurysm,. Recent fractures, recent MI, unstable cervical spine., brain
tumors causing raised intracranial pressure ..but because of the general
anesthesia ,it is nowadays safe .
Chapter XV / Dermatology Answers
Q1:
Answer: 4
1- so these should be avoided.
2- true, it will exacerbate the skin disease, if severe join disease is present try
methotrexate which is effective against both skin and joint disease.
3- like the genitals and the natal clefts, so always examine these areas especially in
join diseases with out any apparent skin involvement (so you will think of another
diagnosis like rheumatoid factor negative rheumatoid arthritis); the patient might be
shy from telling you that these areas are involved.
4- false, seen in children, and some children may develop chronic plaque type upon
reaching adulthood.
5- true with many HLA associations like HLA DW6.
Q2:
Answer: 1
1- false, very irritant and should be avoided at these sites.
2- true, also oral retinoids are used in severe cases.
3- true.
4- to prevent relapse upon steroids withdrawal.
5- true, avoid chloroquin in joint disease.
Q3:
Answer: 5
Isotretinoin is used in the treatment of severe scaring acne. Other agents that might
cause this rash: chlorinated hydrocarbons, estrogenic steroids, oils, and tar.
Q4:
Answer: 5
1- false, middle age.
2- false, against the diagnosis; they are more suggestive of acne vulgaris.
3- false, the treatment of choice; sometimes prolonged for 1 year.
4- are prominent part in the clinical picture.
5- but with no comedons.
Q5:
Answer: 4
1- true, by attacking the scalp but it is uncommon, so-called lichen planus pillaris.
2- true, usually seen as white lace-like lesions on the buccal mucosa, but the
ulcerative variety is rare and is clinically challenging.
3- true, think of another disease; but always ask if the patient is taking an antipruritic
agent! The patient may say it is not pruritic!
4- false, self limiting usually within 1-2 years, leaving a prominent hyperpigmentation
on the involved skin areas.
5-true.
Q6:
Answer: 5
Dont think that pruritis is due to skin diseases only; many diseases in medicine can
produce disabling pruritis. Rifampicin has been shown to be effective against pruritis
in selected patients.
Q7:
Answer: 5
The word "characteristic" means something you have to look for the diagnosis and
absence of itching in such cases either means that your diagnosis is wrong or the
patient is taking a medication against it (so always ask about these anti itching
medications here).
Q8:
Answer: 4
1- true, useful clue when seeing a patient with rigidity, dystonias, and liver
impairment.
2- true, and may not be seen in the fingers because they disappear faster than the toes,
so examine the toes after this long period.
3- true. and may be short also, so-called brachynychia.
4- nail painting!!
5- true, indicating an established uremia, a useful sign to differentiate from acute renal
failure.
Q9:
Answer: 4
1- and usually resistant to treatment
2- true .the HIV patients are very sensitive for many drug reactions, like Septrin
rashes
3- so take biopsy and examine the mouth
4- false, it is not an AIDS defining illness.
5- true, always keep it in mind.
Q10:
Answer: 2
1- true, a slowly growing tumor.
2- false, distant metastases are extremely rare, the tumor is locally invasive and
destructive.
3- true, may also look like malignant melanoma.
4- radiotherapy is an acceptable alternative in certain areas.
5-true.
Q11
Answers: 5
The 1
st
4 options are true; diabetes per se does not confer an increase risk for
malignant melanoma development.
Q12:
Answer: 2
1- true, but the nodular variety is not preceded by that phase.
2- false, seen mainly in sun exposed areas ( usually the face) of old people.
3- true, and mostly seen in Caucasians.
4- any suspicious change in a melanocytic nevus prompt careful work up.
5- true, but are extremely rare.
Q13:
Answer: 4
1- the reverse is true.
2- tumors at certain sites like the leg generally are considered to be less aggressive
than tumors of the head and neck.
3- false , it is a reliable predictor of prognosis in stage I.
4- 70% is the correct figure.
5- true, about 90% 5 year survival rate.
Q14:
Answer: 4
1- 50% of those patients (especially heart and kidney recipients) will develop this
cancer.
2- a scaring genetic syndrome affecting the skin.
3- in or around the ulcer; also in chronic skin sinuses.
4- pale skin is a risk factor (like malignant melanoma).
5- and exposure to UV radiation and X ray irradiation.
Q15:
Answer: 4
1- benign tumors affecting old people.
2- partial thickness skin dysplasia.
3- full thickness skin dyspalsia.
4- true, but the behavior is totally different. They are totally benign.
5- they have nothing to do with sebum or sebaceous glands. Better to be called a basal
cell papilloma.
Q16:
Answer: 4
1- this is a common disorder, with repetitive picking or fidding of the prximal nail
fold of the skin.
2- and follow up is indicated to see "movement" of the underlying discoloration.
3- and in nail trauma and infective endocarditis.
4- false, indicates a chronic repetitive skin itching and rubbing with the nails.
5- usually few nails are infected.
Q17:
Answer: 5
1- erythema nodosum.
2- by granulomatous deposits.
3- lupus pernio.
4- or nodules or plaques ( sarcoid plaques).
5- not a manifestation of this multi-systemic granulomatous disease.
Q18:
Answer: 4
Human herpes type 8 infection is linked to the development of Kaposi's sarcoma;
herpes simplex infection can cause eryhtema multiforme.
Q19:
Answer: 2
1- or associated with many systemic illnesses like inflammatory bowel disease,
rheumatoid arthritisetc.
2- false, there are no diagnostic changes on skin biopsy and hence the diagnosis is a
primarily clinical one.
3- but after successful treatment of the associated disease relapses are uncommon and
intermittent.
4- or systemic steroids, sulphazalazine, dapsone, and cyclosporine.
5- together with pain relief and dressing.
Q20:
Answer: 4
Diffuse skin hyperpigmentation is a striking feature; the 1
st
3 options are seen mainly
in light exposed areas.
Q21:
Answer: 4
Phenylketonuria is cause of fait skin and hair. Other causes of pale skin in the absence
of anemia: vitiligo, oculocutaneous albinism, and panhypopituitarism.
Notice that any chronic illness can cause generalized hyperpigmentation.
Q22:
Answer: 5
1- Beta blockers.
2- Lithium.
3- Chloroquin.
4- Positive Koebner's phenomenon.
5- None of the antiasthma medications have been shown to exacerbate psoriasis.
Q23:
Answer: 5
The 1
st
4 options are the mainstay in the management of eczema in general. No place
at all for regular use of cyclosporine.
Q24:
Answer: 5
OCC are used in the treatment of hirsutism.
Q25:
Answer: 5
1- for a suspected contact allergic dermatitis.
2- to support a diagnosis of atopic eczema, and to detect a specific environmental
allergen or allergens ( animal danders, house dust mitesetc.).
3- same indications for IgE testing ,but less commonly performed.
4- useful in any susoected secondary infections which are common.
5- TB skin lesions are infectious in nature, not eczematous!
Q26:
Answer: 4
1- superinfections are very common unfortunately and add more to the burden of the
disease, particularly with staph areus, herpes simplex (which may cuase a severe
diffuse skin rash called eczema herpeticum); human papilloma virus and molloscum
contagiosum infections are both common especially with the use of topical steroids.
2- and behavioral disturbances.
3- resulting in poor school records.
4- patients with atopic eczema have an increased incidence of food allergy,
particularly to eggs, cow's milk, soya, wheat, and fish. Those foods usually cause
immediate urticarial lesions rather than exacerbation of the eczema per se.
5- because of the breaks in the skin barrier.
Q27:
Answer: 5
Minoxidil is a cause of hirsutism, and that's why minoxidil shampoos are used by bald
people.
Q28:
Answer: 5
Hirsutism is usually familial and racial; and some degree of hirsutism is expected
after menopause. The 1
st
4 options prompt a search for an underlying cause.
Q29:
Answer: 5
Alopecia areata can cause localized or diffuse NON-scarring alopecia.
Q30:
Answer: 5
Androgenetic alopecia can cause localized and generalized non-scarring alopecia.
Notice that discoid lupus can cause localized and diffuse scarring alopecia.
Q31:
Answer: 2
1- and involvement of the oral mucosa is seen in 100% of cases which may predate
the skin manifestations.
2- oral mucosa is involved in 60% of cases, and the target antigen is BP-220 (part of
hemidesmosomes).
3- frank blisters are uncommonly seen; itching is severe and we may see only
excoriations.
4- but may respond to cyclophosphamide or methotrexate.
5- and oral mucosa involvement is rare, the target antigen is type XVII collagen and
BP-180.
Q32:
Answer: 4
Porphyria cutanea tarda can cause skin blistering, increased fragility, scars, milia,
hyperpigmentation and hypertrichosis, but it does not involve the mouth mucosa.
Q33:
Answer: 5
Other photosensitive dermatoses: chronic actinic dermatitis, SLE, herpes simplex,
certain porphyrias, and medication induced (photo-toxic and photo-allergic reactions).
Q34:
Answer: 5
Porhyria cutanea tarda causes generalized hyperpigmentation. Other causes of
erythroderma: cutaneous T cell lymphoma, eczema, and lichen planus.

Q35:
Answer: 5
The so called obstetric cholestasis, together with the 1
st
4 options, are specific causes
of pruritis encountered only in pregnancy.
Chapter XVI / Genetics Answers
Q1:
Answer: 3
1- a secondary phenomenon to hypogonadism.
2- many other karyotypes are also seen.
3- false, the reverse is true, due to hypogonadism and delayed epiphyseal closure.
4- true, unlike Turner's syndrome.
5- usually very mild and does not affect performance that much.
Q2:
Answer: 5
1- true, renal anomalies are common which may be the cause also.
2- Turner's syndrome has left sided cardiac lesions, unlike Noonan's syndrome which
has right sided cardiac lesions.
3- true. usually transient.
4- true, watch for side effects like gall stones.
5- false, only to induce the appearance of secondary sexual characteristics; they are
totally and irreversibly infertile.
Q3:
Answer: 5
The Word "familial" almost always indicates an autosomal dominant inheritance;
FMF is an exception! Inborn errors of metabolism are usually autosomal recessive.

Q4:
Answer: 5
Post-date pregnancy state is not associated with elevated serum AFT.
Q5:
Answer: 4
1- true, abortion still may occur even in experienced hands.
2- true, when the size of the uterus is large enough but still it is considered to be a
LATE investigation; CVS can done much earlier so that intervention can be done
early also.
3- true, as well as many enzymes assessment.
4- false, under ultrasound guidance.
5- by finding an elevated level of 17 hydroxy progesterone in the amniotic fluid.
Chapter XVII / Toxicology Answers
Q1:
Answer: 5
Digoxin toxicity is enhanced by HYPOkalemia, and its self causes hyperkalemia at
toxicity; other factors that enhance its toxicity are hypocalcaemia, renal impairment,
ischemic cardiopmyopathy, chronic lung disease, and acidosis.
Q2:
Answer: 5
Indication of bicarbonate infusion here: long QT interval, severe hypotension , severe
acidosis, life threatening cardiac dysrrhytmias, and signs of severer CNS toxicity like
seizures. The objective is to raise the blood PH to a level of 7.45-7.55 with serum K
being in the upper range of normal reference.
Q3:
Answer: 3
1- but the red cell free protoporphyrins are increased.
2- due to chronic interstitial nephritis and renal tubular acidosis.
3- indicates chronic exposure, mainly due to deposition of sulphides and irritation of
the near by gum.
4- but the PBG is normal; acute intermittent porphyria is one of the differential
diagnoses.
5- and peripheral motor neuropathy is mainly seen in adults with chronic exposure.
Q4:
Answer: 5
1- true, and acidosis may very severe.
2- true, but it is irreversible if there was a late presentation with delayed treatment.
3- mainly seen in the early phase between 30minutes -12 hours of poisoning.
4- true, peritoneal dialysis also may be used.
5- false, does not combine with it.
Q5:
Answer: 5
1- check PT, PTT, and fibrinogen.
2- in rhabdomyolysis or prolonged seizures.
3- true, drinking too much water and SIADH.
4- also, shock, malignant ventricular dysrrhythmias, aortic dissection. Hypertension is
common and when hypotension is seen, thinks of these complications.
5- one of the causes of unexplained intracranial hemorrhage in young people.
Q6:
Answer: 5
1- true, whenever the clinical setting is suggestive, like a patient found unconscious in
his closed garage.
2- and there is low PaO2; so this discrepancy may be a clue.
3- the rose pink color of the skin is rare antemortem; cyanosis is much more common
4-true, as it may further impair the release of oxygen to tissues; so use diazepam to
control seizures.
5- false, in smokers the level of carboxyHb may be up to 15%. Indications of
hyperbaric O2: pregnancy, severe neurological impairment, coma at any time,
carboxyHB above 40% (some centers give it when it is above 20%).
Q7:
Answer: 4
1-true, but diplopia, ataxia, and coarse tremor are common.
2-true, usually caused rapid IV infusion causing asystole.
3-also any drug that competes with pheytoin albumin binding sites.
4-false, of no use at all.
5-true, look for any precipitating cause and measure serum phenytoin.
Q8:
Answer: 4
1- also by NSAIDS, renal failure, ACE inhibitors, and diarrhea.
2- causing hypothyroidism. Also may cause hyperparathyroidism with high PTH level
(i.e. like primary hyperparathyroidism).
3- and coma, up going planters.
4- false, fine tremor is commonly found at therapeutic levels; but coarse irregular one
indicates poisoning.
5- above 3-3.5 mmol / L.
Q9:
Answer:3
1- true, an acute confusional state which is mainly seen in non-chronic alcoholics.
2- true, although the severity and clinical features of encephalopathy correlate roughly
with blood ethanol levels. Chronic heavy alcoholics might have a very high blood
level although they don't appear to be intoxicated.
3- false, it is characteristically raised . The plasma osmolality roughly increases by 22
mOsm/L for every 100 mg / dl of ethanol presents.
4- true. Can be differentiated by presence of ethanol odor, increased plasma
osmolality (in ethanol poisoning it is raised), blood and urinary toxicology.
5-true. Also predisposes to head injury, lung aspiration, seizures. Chronic alcoholism
increases the risk of bacterial meningitis.
Remember that the treatment is supportive only. All alcoholics should receive 100 mg
of thiamin intravenously to prevent Wernick's encephalopathy.
Q10:
Answer: 4
1- true, with respiratory depression , hypotension, reactive pupils and hypothermia.
2- true, a characteristic feature. However, very large doses of phenobarbitone or
glutethimide may result in LARGE FIXED pupils.
3- true, with ataxia, dysarthria and hyporeflexia.
4- false. The mortality rate is low and mostly due to aspiration pneumonia (with or
without systemic sepsis), or due to iatrogenic fluid overload and pulmonary
edema.Despite severe intoxication, a patient who arrives at the hospital with adequate
cardio-pulmonary function and support should survive without any sequelae.
Remember the treatment is mainly supportive while the drug is being eliminated.
5- true, it is mainly used to increase the urinary clearance of Phenobarbital., but in
general should be avoided as it can lead to fluid overload. Hemodialysis may be used
in severe resistant cases of barbiturate poisoning or when drug elimination is impaired
by renal failure.
Q11:
Answer: 4
1- true, intermediate or short acting agents are more likely to produce a withdrawal
syndrome when stopped abruptly. The syndrome is seen within 1-3 days (for short
acting agents) and may take up to 1 week or even more to appear (for long acting
agents).
2- true, with confusion, agitation, seizures.
3- true, if positive , the patient should receive long acting phenobarbital orally to
maintain a calm state without signs of intoxication. In most patients it is possible to
stop it gradually after progressive decrement in the daily doses within 2 weeks.
4- false. Seizures especially the myoclonic ones should be treated aggressively with
anticonvulsants.
5- true, mainly seen in those taking very high frequent doses.
Q12:
Answer: 4
1- true, as an iatrogenic overdose. Also seen as an accidental overdose in addicts, and
in suicidal attempts.
2- true, and pontine hemorrhage is a differential diagnosis .Although needle tracks
and marks might be seen, they are not diagnostic .
3- true, the test is positive if the pupils dilates and the patient regains his full
consciousness; however, when very large doses of opioids are taken or multiple drug
ingestion is present the pupils may slightly dilate.
4- false, with appropriate treatment , patients should recover uneventfully .
5- true, because nalaxone is a short acting agent
Q13:
Answer: 4
1- true, or with antidepressants and antihistamine overdose.
2- true, also flushing, urinary retention and tachycardia.
3- true, mainly used in antipsychotics or antidepressants overdose.
4- false. Symptoms usually resolve spontaneously.
5- true, although rarely needed . Physostigmine can produce severe bradycardia,
seizures and hypersalivation. Specific treatment is required when there is life
threatening cardiac dysrrythmias.
Q14:
Answer :4
1- true. Their mechanism of action involves a variable combination of inhibiting the
reuptake and or increasing the release of noradrenalin and or dopamine and thus
producing a central stimulant and peripheral sympathomimetic effects.
2- true, and cocaine can produce myocardial infarction.
3- true, either due to sudden severe hypertension, drug induced vasculitis, or rupture
of AVMs.
4- false, should be avoided , especially in cocaine induced myocardial infarctions.
Alpha blockers are useful to attack hypertension.
5- true, and thus attacking the psychotic manifestations of overdose. Because
amphetamines are longer acting than cocaine, amphetamine intoxication is more
likely to require treatment
Q15:
Answer: 3
1- true, and prominent insomnia.
2- true, changes in the mental status are usually the most striking feature. Alterations
in affect and mood may predominate the clinical picture.
3- FALSE, very rare. The presence of a prominent seizure activity should prompt a
search for another pathology or to revise the diagnosis.
4- true, there is prominent sympathetic overactivity.
5- true, and when this fails, treatment with diazepam may be of benefit.
Q16:
Answer: 3
1- true, unlike other hallucinogens (eg LSD).
2- true, also large or small pupils, horizontal and vertical nystagmus, hypertonia,
hyper-reflexia and myoclonus. There may analgesia to a surprising degree.
3- false, phenothiazines reduce seizure threshold and may produce severe
hypotension. Haloperidol can be used safely in such cases. Diazepam can be used for
sedation and treating muscle spasms.
4- true, although in some patients it may take days or even weeks.
5- true, this is especially seen in poisoning with large doses .
Q17:
Answer: 5
1- true, whether accidental or intentional.
2- true, thus such patients may be wrongly diagnosed as having a serious CNS illness.
3- true, like renal or hepatic failures.
4- true. Dementia patients are very susceptible.
5- false, one of the prominent causes especially in old people.
Q18:
Answer: 1
1- false. Usually seen within 48hours; however, in 70% of cases they occur within 7-
24 hours of abstinence.
2- true, and the interval between the first and last seizure is usually 6-12 hours up to
85% of cases . Up 40 % of patients will have ONE seizure only.
3- true. They abate spontaneously; however, diazepam or chlordiazepoxide is given
prophylactically because up to 30% of patients will develop delirium tremens.
4- unusual and atypical features are: focal fits, prolonged duration of the fits ( >6-12
hours ), more than 6 fits, status epilepticus or a prolonged post-ictal phase. In these
cases, a prompt search for pathology is required.
5- true. Always look for such an association.
Q19:
Answer: 3
1- true. It is the most aggressive type with a high mortality rate, usually seen with 3-5
of abstinence and may last up to 72 hours.
2- true. Also tachycardia and sweating.
3- false. The mortality rate is 15% and mostly due to concomitant: infection,
pancreatitis, cardiovascular collapse, or trauma.
4- true, the total requirement to produce a calm patient may exceed 100 mg/ HOUR.
5- true, like atenolol 50-100 mg/ day.
Q20:
Answer: 4
Item "4" is false because confusion, IF PRESENT, is usually mild. Illusions and
hallucinations, usually visual, are seen up to 25% of cases. It usually responds to
diazepam 5-20 mg or chlordiazepoxide 20-25 mg orally every 4 hours.

Вам также может понравиться